You are on page 1of 234

 

ANSWER:  
10-1 Sequences as Functions Yes
 

Determine whether each sequence is arithmetic 3. 1, 2, 4, 8, 16


no.                                                                           
                                                                              
1. 8, –2, –12, –22 SOLUTION:  
  Subtract each term from the term directly after it.
 
SOLUTION:   2 4 8 16
Subtract each term from the term directly after it. 2 4 8
8 -2 -12 -22
 
-10 -10 -10 There is no common difference.
  Therefore, the sequence is not arithmetic.
The common difference is –10.  
Therefore, the sequence is arithmetic.
  ANSWER:  
No
ANSWER:    
Yes
  4. 0.6, 0.9, 1.2, 1.8, ...
 
2. –19, –12, –5, 2, 9 SOLUTION:  
  Subtract each term from the term directly after it.
SOLUTION:    
Subtract each term from the term directly after it. 0.6 0.9 1.2 1.8
  0.3 0.3 0.6
-19 -12 -5 2 9  
7 7 7 7 There is no common difference.
Therefore, the sequence is not arithmetic.
 
 
The common difference is 7.
Therefore, the sequence is arithmetic. ANSWER:  
  No
ANSWER:    
Yes Find the next four terms of each arithmetic
  sequence. Then graph the sequence.
 
3. 1, 2, 4, 8, 16
5. 6, 18, 30, …
 
 
SOLUTION:  
SOLUTION:  
Subtract each term from the term directly after it.
Subtract each term from the term directly after it.
 
 

 
 
There is no common difference.
The common difference is 12.
Therefore, the sequence is not arithmetic.
Therefore, the sequence is arithmetic.
 
 
ANSWER:   To find the next term, add 12 to the last term.
No Manual - Powered by Cognero 30 + 12 = 42
eSolutions Page 1
42 + 12 = 54
 
54 + 12 = 66
4. 0.6, 0.9, 1.2, 1.8, ... 66 + 12 = 78
 
ANSWER:  
10-1No
Sequences as Functions  
 

Find the next four terms of each arithmetic 6. 15, 6, –3, …


sequence. Then graph the sequence.  
 
5. 6, 18, 30, … SOLUTION:  
  Subtract each term from the term directly after it.
 
SOLUTION:   15 6 ...

Subtract each term from the term directly after it. -9 -9
 
 
6 18 30
The common difference is –9.
12 12 Therefore, the sequence is arithmetic.
   
The common difference is 12. To find the next term, add –9 to the last term.
Therefore, the sequence is arithmetic.  
  –3 + (–9) = –12
To find the next term, add 12 to the last term. –12 + (–9) = –21
30 + 12 = 42 –21 + (–9) = –30
42 + 12 = 54 –30 + (–9) = –39
54 + 12 = 66  
66 + 12 = 78 Graph the sequence.
   
Graph the sequence. ,- 1" r. .
 
L~ y
,- t
-
.. rr:-10 1 2
• .. s 6 71
-70 • -1.
-1f
-
..,..,
-24
.,., ,._3::
•A
-.....--
.,.,
-36
I

. ·-
In
1A

1 4 t 7 I
 
ANSWER:  
  –12, –21, –30, –39
ANSWER:  
,- 1·
,- -- r.
#

42, 54, 66, 78


 
-10
'-
-1.
1 2
• .. s 6 71

L y -tf
--
..,.., -24
• -3:
70
60
-
-.3:
.,.,
..
--·-  
.....
-1· 7. –19, –11, –3, …
·-  
0 1 3 .. 6 7 11

  SOLUTION:  
Subtract each term from the term directly after it.
 
6. 15, 6, –3, …
 
SOLUTION:  
eSolutions Manual - Powered by Cognero
  Page 2
Subtract each term from the term directly after it. The common difference is 8.
  Therefore, the sequence is arithmetic.
10-1 Sequences as Functions
   

7. –19, –11, –3, … 8. –26, –33, –40, …


   
SOLUTION:   SOLUTION:  
Subtract each term from the term directly after it. Subtract each term from the term directly after it.
   
-19 -11 ...
-.) -26 -33 -40
8 8 -7 -7
   
The common difference is 8. The common difference is –7.
Therefore, the sequence is arithmetic. Therefore, the sequence is arithmetic.
   
To find the next term, add 8 to the last term. To find the next term, add –7 to the last term.
   
–3 + 8 = 5 –40 + (–7) = –47
5 + 8 = 13 –47 + (–7) = –54
13 + 8 = 21 –54 + (–7) = –61
21 + 8 = 29 –61 + (–7) = –68
   
Graph the sequence. Graph the sequence.
   

- 3(i 1
-~4
_J1C
-10
-r
1
I
I
2 3 4 S 6 7x

-- e
-18 I
12 -2(;
-3C ,,
-4C
-10 1 2t4SfiX
I
-12
-~
fo-6C '
-18-
I
-7C
I
-
   
ANSWER:   ANSWER:  
5, 13, 21, 29 –47, –54, –61, –68
   
-~Cl'
., I
-24
--11c I
-18 -10 1 2 3 4 S 6 7x
I
-12 -2C
- t -3C
-4C
- ,,
-10 1 2t4S6iX
-SC
-112 fo-6C '
-18
I
-7C
I
-
   
8. –26, –33, –40, … 9. FINANCIAL LITERACY  Kelly is saving her
  money to buy a car. She has $250, and she plans to
save $75 per week from her job as a waitress.
SOLUTION:  
 
Subtract each term from the term directly after it.
a. How much will Kelly have saved after 8 weeks?
 
 
b. If the car costs $2000, how long will it take her to
eSolutions Manual - Powered by Cognero Page 3
save enough money at this rate?
   
The common difference is –7.
a. $850
 
10-1 Sequences as Functions b. 24 wk
   

9. FINANCIAL LITERACY  Kelly is saving her Determine whether each sequence is


money to buy a car. She has $250, and she plans to geometric. Write yes or no.
save $75 per week from her job as a waitress.  
  10. –8, –5, –1, 4, …
a. How much will Kelly have saved after 8 weeks?  
 
SOLUTION:  
b. If the car costs $2000, how long will it take her to
Find the ratio of the consecutive terms.
save enough money at this rate?
   
-5 S -1 I 4
---4
SOLUTION:   -8 8 -5 5 -I
a. Given a 0 = 250, d = 75 and n = 8.  
Since the ratios are not same, the sequence is not
After 8 weeks, she will have 250 + (8 × 75) or $850.
geometric.
 
 
b. Given a n = 2000.
  ANSWER:  
Find n. No
a,, = all + ml  
2000 - 250 + 1175
7511-1750 11. 4, 12, 36, 108, …
II;:.:, 24  
  SOLUTION:  
So, it will take about 24 weeks to save $2000. Find the ratio of the consecutive terms.
   
ANSWER:   12 -3 36 -3 108 -3
a. $850 4 12 36
   
Since the ratios are the same, the sequence is
b. 24 wk
geometric.
   
Determine whether each sequence is ANSWER:  
geometric. Write yes or no.
Yes
   
10. –8, –5, –1, 4, …
  12. 27, 9, 3, 1, …
SOLUTION:    
Find the ratio of the consecutive terms. SOLUTION:  
  Find the ratio of the consecutive terms.
 

 
Since the ratios are not same, the sequence is not  
geometric. Since the ratios are the same, the sequence is
  geometric.
 
ANSWER:  
No ANSWER:  
  Manual - Powered by Cognero
eSolutions Yes Page 4
 
11. 4, 12, 36, 108, …
 
 
ANSWER:   Since the ratios are the same, the sequence is
10-1Yes
Sequences as Functions geometric
   
To find the next term, multiply the previous term by
12. 27, 9, 3, 1, … 3
  .
2
SOLUTION:    
Find the ratio of the consecutive terms. 21xI _ !!. _ 40.5
  2 2
9 3 243
~xI= =60.15
27 3 9 3 3 3 2 2 4
  243xl= 729 =91.125
Since the ratios are the same, the sequence is 4 2 8
geometric.  
  Graph the sequence.
 
ANSWER:   y
Yes
  7:
:
13. 7, 14, 21, 28, …
  4:,_
SOLUTION:  
1::
Find the ratio of the consecutive terms.
0 1 2 3 4 5 6 7 8 9X
 
21 3 28 4  
~=2
7 14 2 21 3 ANSWER:  
  40.5, 60.75, 91.125
Since the ratios are not the same, the sequence is not  
geometric. y
  -
7:
ANSWER:   ,A

No ~A

  4:;

Find the next three terms of each geometric --1:


sequence. Then graph the sequence.
  0 1 2 3 4 5 6 7 8 9X
14. 8, 12, 18, 27, …  
 
SOLUTION:   15. 8, 16, 32, 64, …
 
Find the ratio of the consecutive terms.
  SOLUTION:  
-
I ')
-:::;.-
".) 18
-;::;-
3 27 3
-;-
Find the ratio of the consecutive terms.
8 2 12 2 18 2  
 
Since the ratios are the same, the sequence is
geometric  
  Since the ratios are the same, the sequence is
To find the next term, multiply the previous term by geometric.
.  
eSolutions Manual - Powered by Cognero To find the next term, multiply the previous termPage
by 5
  2.
 
 

16. 250, 50, 10, 2, …


10-1  Sequences as Functions  
SOLUTION:  
15. 8, 16, 32, 64, … Find the ratio of the consecutive terms.
   
50 I 10 I 2 I
SOLUTION:   -=- -=-
250 5 50 5 10 5
Find the ratio of the consecutive terms.  
  Since the ratios are the same, the sequence is
"'")
.,_ - 2 64 geometric
~='
8 - 16
---
"')
s:
'>
 
  To find the next term, multiply the previous term by
Since the ratios are the same, the sequence is
geometric. .
5
   
To find the next term, multiply the previous term by I 2
2. 2x---
5 5
 
2 I 2
64x 2 = 128 -x---
5 5 25
128x 2 = 256
2 I 2
256x 2 = 512 -x-=-
25 5 125
   
Graph the sequence.
Graph the sequence.
   
. . .450 y I

-•
..... 22., ... y
..... 420
..... 36()
. . .i--
..
1-300 ..... 17:
..... 240 -1s:
..... 180 H2:
~120 -1 ·--
H6() - '.., --

 
-10 1234567X
-- -
t
x
-10 1 2 3 4 ~
ANSWER:    
128, 256, 512 ANSWER:  
 
2 2 2
-480
I y ' s·zs ' 12s
..... 420  
..... 36()
-300 1., y
.....22.
-240 ' -2:.-~
--180 -11:
~120 -1s:
H6()
-10
'
1 2 34567X -,- 75..
-12:

  ~-

16. 250, 50, 10, 2, … -·


'--

  :.10 1 2 3 4 ) ,X
 
SOLUTION:  
Find the ratio of the consecutive terms.
eSolutions Manual - Powered by Cognero 17. 9, –3, 1, ,… Page 6
 
 
 
ANSWER:  
10-1 Sequences as Functions
 
 
8 y,
-
17. 9, –3, 1,
 
3
,…
- 6
,...._ 4
,...._ 2
SOLUTION:   '
-10 1 2 3 5 x
Find the ratio of the consecutive terms. I
  --4
1--6
+»..
1--8
- 3 I I
-=-- -=-- -=--
9 3 -3 3 1 3  
 
Since the ratios are the same, the sequence is Determine whether each sequence is
geometric. arithmetic, geometric, or neither. Explain your
  reasoning.
To find the next term, multiply the previous term by  
18. 5, 1, 7, 3, 9, …
.  
"'.)
  SOLUTION:  

(-±)x(-±)-i 5
-4 6
7
-4
3
6
9

ix(-±)=- 2\  
There is no common difference.
Therefore, the sequence is not arithmetic.
(--')x(-! )=_!_
27 3 81  
Find the ratio of the consecutive terms.
 
 
Graph the sequence. I I 3 3
  i. 7 ~=3
5 5 I 7 7 3
 
- 8 y

-- 6
4
Since the ratios are not the same, the sequence is not
geometric.
- -10
2
1 2 3
'
5 x
 
ANSWER:  
I
--4 Neither; there is no common difference or ratio.
1--6
1--8
 
I
  19. 200, –100, 50, –25, …
ANSWER:    
SOLUTION:  
To find the common difference, subtract any term
  from the term directly after it.
 

 
There is no common difference.
Therefore, the sequence is not arithmetic.
eSolutions Manual - Powered by Cognero
  Page 7
Find the ratio of the consecutive terms.
 
 
ANSWER:  
ANSWER:  
Neither; there is no common difference or ratio. Geometric; the common ratio is .
10-1  Sequences as Functions
 

19. 200, –100, 50, –25, … 20. 12, 16, 20, 24, …


   
SOLUTION:   SOLUTION:  
To find the common difference, subtract any term To find the common difference, subtract any term
from the term directly after it. from the term directly after it.
   
200 - I 00 50 - 25 12 16 20 24
-300 150 -75 4 4 4
   
There is no common difference. The common difference is 4.
Therefore, the sequence is not arithmetic. Therefore, the sequence is arithmetic.
   
Find the ratio of the consecutive terms. Find the ratio of the consecutive terms.
   
-100
----- I 50
-~-- I -25 I 16 4 20 5 24 6
200 2 100 2 50 2
  12 3 16 4 20 5
 
I
The common ratio is . Since the ratios are not the same, the sequence is not
2 geometric.
   
Since the ratios are the same, the sequence is
geometric. ANSWER:  
  Arithmetic; the common difference is 4.
 
ANSWER:  
I Determine whether each sequence is
Geometric; the common ratio is . arithmetic. Write yes or no.
2
   
I I I I
20. 12, 16, 20, 24, … 21. 
2 3 4 5
   
SOLUTION:   SOLUTION:  
To find the common difference, subtract any term Subtract any term from the term directly after it.
from the term directly after it.  
  I
-2 3 4 5
I
  6 12 20
The common difference is 4.  
Therefore, the sequence is arithmetic.
There is no common difference.
  Therefore, the sequence is not arithmetic.
Find the ratio of the consecutive terms.  
 
ANSWER:  
No
   
Since the ratios are not the same, the sequence is not
geometric. 22. –9, –3, 0, 3, 9
  Manual - Powered by Cognero
eSolutions
  Page 8

ANSWER:   SOLUTION:  
 
ANSWER:  
ANSWER:  
No
10-1  Sequences as Functions No
 

22. –9, –3, 0, 3, 9 2 5 8 11
  24.  ----
9'9'9'9""

SOLUTION:    
Subtract any term from the term directly after it. SOLUTION:  
  Subtract any term from the term directly after it.
.,
-9 +» 0 3 9  
3
 
6 3 6
-29 -59 -98 -119
There is no common difference.
Therefore, the sequence is not arithmetic. -I -I -I
3 3 3
   
I
ANSWER:   The common difference is - .
No 3
  Therefore, the sequence is arithmetic.
 
23. 14, –5, –19, …
ANSWER:  
 
Yes
SOLUTION:    
Subtract any term from the term directly after it.
  Find the next four terms of each arithmetic
14 -5 -19 sequence. Then graph the sequence.
 
-19 -14
25. –4, –1, 2, 5,…
   
There is no common difference.
Therefore, the sequence is not arithmetic. SOLUTION:  
  Subtract any term from the term directly after it.
 
ANSWER:  
No
 
 
The common difference is 3.
24.  Therefore, the sequence is arithmetic.
   
To find the next term, add 3 to the last term.
SOLUTION:    
Subtract any term from the term directly after it. 5+3=8
  8 + 3 = 11
11 + 3 = 14
14 + 3 = 17
 
Graph the sequence.
 
 
The common difference is .
Therefore, the sequence is arithmetic.
 
eSolutions Manual - Powered by Cognero Page 9
ANSWER:  
Yes
 
ANSWER:  
10-1Yes
Sequences as Functions  
 

Find the next four terms of each arithmetic 26. 10, 2, –6, –14, …
sequence. Then graph the sequence.  
 
25. –4, –1, 2, 5,… SOLUTION:  
  Subtract any term from the term directly after it.
 
SOLUTION:   10 2 -6 -1-1
Subtract any term from the term directly after it. -8 -8 -8
 
 
-4 -I 2 S
The common difference is –8.
3 3 3 Therefore, this sequence is arithmetic.
   
The common difference is 3. To find the next term, add –8 to the last term.
Therefore, the sequence is arithmetic.  
  –14 + (–8) = –22
To find the next term, add 3 to the last term. –22 + (–8) = –30
  –30 + (–8) = –38
5+3=8 –38 + (–8) = –46
8 + 3 = 11  
11 + 3 = 14 Graph the sequence.
14 + 3 = 17  
 
Graph the sequence.
I
1-
A y'
  -10
I
1 2 ' 4 5 6 7x
I y -12 0
-18 t
-15 -1:

---
-12 ' -24
I
-9 -3~
-36
6
3 ' 1--42
I I •
-10 1 1
3 4 5 6 7x  
I

 
---6
I ' ANSWER:  
–22, –30, –38, – 46
 
ANSWER:   IA 1, I
1-
8, 11, 14, 17
 
I y
-10
-12
I
1 2 ' I
4 5 6 7X

-18 t
~
-1e
-1s
-12
I
-24 '.
-- 9
6
.__ 3 '
'
1
-3~
·-36
-42
-, I

-10 1 1
3 4 5 6 7x  
....
I"'
I
v
' 27. –5, –11, –17, –23, …
   
SOLUTION:  
26. 10, 2, –6, –14, … Subtract any term from the term directly after it.
  Manual - Powered by Cognero
eSolutions   Page 10

SOLUTION:  
Subtract any term from the term directly after it.
10-1 Sequences as Functions
   

27. –5, –11, –17, –23, … 28. –19, –2, 15, …


   
SOLUTION:   SOLUTION:  
Subtract any term from the term directly after it. Subtract any term from the term directly after it.
   
-5 -11 -17 -23 -19 -2 15
-6 -6 -6 17 17
   
The common difference is –6. The common difference is 17.
Therefore, the sequence is arithmetic. Therefore, the sequence is arithmetic.
   
To find the next term, add –6 to the last term. To find the next term, add 17 to the last term.
   
–23 + (–6) = –29 15 + 17 = 32
–29 + (–6) = –35 32 + 17 = 49
–35 + (–6) = –41 49 + 17 = 66
–41 + (–6) = –47 66 + 17 = 83
   
Graph the sequence. Graph the sequence.
   
I y I y
'-105
-10 2 3 4 5 6 7X

- •v

-10
I
,, ~5 ~
'
-15 rvv
-20 I--
~5
-25
-3~
' -.__ 3015
-35
-10 1 I 34567X
-40
~-45

 
I
~
  '
ANSWER:  
ANSWER:   32, 49, 66, 83
–29, –35, – 41, – 47  
  I y
y '-105
I •v
-10
I
2 3 4 5 6 7 X
- 75 ~
j

~-10 rvv
-15 I--
~5
- zo ,V
'
- 30
-25 ' .__ 15
-3"
-10 1 ' 34567X
f--35
·-40
-45
I
~   '
  I 4 7
29. 
5 5 5
28. –19, –2, 15, …
   
SOLUTION:  
SOLUTION:  
eSolutions Manual - Powered by Cognero Page 11
Subtract any term from the term directly after it.
Subtract any term from the term directly after it.
   
 
 
ANSWER:  
29. 
10-1 Sequences as Functions
 
 
SOLUTION:   4 y
Subtract any term from the term directly after it. '
  3 '
4 7 •
2
5 5 5
-3 -3 1 ,,
5 5
  0 '
1 2 3 4 5 6 7 8X
...,
.J
The common difference is .  
5
Therefore, the sequence is arithmetic. 2 4
  30.  -3. --3. --3
3  
To find the next term, add  to the last term.
5
  SOLUTION:  
7 3 Subtract any term from the term directly after it.
-+-=2  
5 5
-23 I 4
3 13
2+-=- 3 3
5 5
-I -I
13 3 16
-+-=-  
5 5 5
The common difference is –1.
16 3 19
-+-:;;;- Therefore, the sequence is arithmetic.
5 5 5  
  To find the next term, add −1 to the last term.
Graph the sequence.  
 
4 y -~-I=-_?_
3 3
' _ _?__I = _.!2_
3 '
3 3
2 _.!2__1 =-13
3 3
1 ,, 13
---J:;;;--
16
' 3 3
0 1 2 3 4 5 6 7 8X  
  Graph the sequence.
 
ANSWER:  
y
.,g ~ !2 '
-· 5. 5. 5 0 I
x
  '
'
'
'

eSolutions Manual - Powered by Cognero ANSWER:   Page 12

 
b. On what day will Mario first row an hour or
more?
 
10-1 Sequences as Functions c. Is it reasonable for this pattern to continue
  indefinitely? Explain.
 
ANSWER:  
SOLUTION:  
7 10 13 16
--.--.--'1-- a. Given a 1 = 5, d = 1.5 and n = 18.
3 3 3 3
   
,y Find a 18.

- a,, -a,+ (11- l)d


0 I
x
I
a,ll = 5+ (18-1)1.5
'
, =30.S
 
' I Therefore he will row for 30 minutes and 30 seconds
on the 38th day.
 
  b. Given a 1 = 5, d = 1.5 and a n = 60.
Find n.
31. THEATER There are 28 seats in the front row of a
a,, - a1 + (11- l)d
theater. Each successive row contains two more
seats than the previous row. If there are 24 rows, 60 = 5 + (11-1)1.5
how many seats are in the last row of the theater? 1.511 = 56.5
 
II:::: 38
SOLUTION:    
Given a 1 = 28, d = 2 and n = 24. Mario will first row an hour or more on the 38th day.
 
 
c. Sample answer: It is unreasonable because there
Find a 24. are only so many hours in the day that can be
  dedicated to rowing.
a,, -a1 + (11- l)d  
a2, =28+(24-1)2 ANSWER:  
=74 a. 30 minutes and 30 seconds
   
b. on the 38th day
ANSWER:  
 
74
c. Sample answer: It is unreasonable because there
 
are only so many hours in the day that can be
dedicated to rowing.
32. CCSS SENSE-MAKING  Mario began an exercise
program to get back in shape. He plans to row 5  
minutes on his rowing machine the first day and
Determine whether each sequence is
increase his rowing time by one minute and thirty
geometric. Write yes or no.
seconds each day.
   
33. 21, 14, 7, …
a. How long will he row on the 18th day?
 
 
 
b. On what day will Mario first row an hour or
more? SOLUTION:  
  Find the ratio of the consecutive terms.
c. Is it reasonable for this pattern to continue  
indefinitely? Explain.
 
eSolutions Manual - Powered by Cognero Page 13
SOLUTION:    
Since the ratios are not the same, the sequence is not
a. Given a 1 = 5, d = 1.5 and n = 18.
geometric.
  Since the ratios are the same, the sequence is
c. Sample answer: It is unreasonable because there geometric.
are only so many hours in the day that can be
10-1dedicated
Sequencesto rowing.
as Functions ANSWER:  
  Yes

Determine whether each sequence is 36. 162, 108, 72, …


geometric. Write yes or no.  
 
SOLUTION:  
33. 21, 14, 7, …
  Find the ratio of the consecutive terms.
   
108 2 72 2
SOLUTION:   162 3 108 3
Find the ratio of the consecutive terms.  
  Since the ratios are same, the sequence is geometric.
14
-;::;-
2 7
-;::;-
 
21 3 14 2
  ANSWER:  
Since the ratios are not the same, the sequence is not Yes
geometric.  
 
I I I
37.  -.--.1.-- ....
ANSWER:   2 4 2
No  
 
SOLUTION:  
Find the ratio of the consecutive terms.
34. 124, 186, 248, …  
  1
-4 __ l
SOLUTION:   l - 2
Find the ratio of the consecutive terms. 2
 
186 3 248 4 _1_1 = -4
124 2 186 3 -4
  l
Since the ratios are not the same, the sequence is not -2 = _l
geometric. 1 2
   
ANSWER:   Since the ratios are not same, the sequence is not
geometric.
No
   
ANSWER:  
35. –27, 18, –12, …
No
SOLUTION:    
Find the ratio of the consecutive terms.
18 2 -12 2 38. –4, –2, 0, 2, …
-=-- ... -;:::;--  
-27 ..> 18
Since the ratios are the same, the sequence is SOLUTION:  
geometric. Find the ratio of the consecutive terms.
ANSWER:    
Yes

36. 162, 108, 72, …  


  Manual - Powered by Cognero Since the ratios are not same, the sequence is not
eSolutions Page 14
geometric.
SOLUTION:    
Find the ratio of the consecutive terms.
   
ANSWER:   ANSWER:  
10-1No
Sequences as Functions No
   

38. –4, –2, 0, 2, … Find the next three terms of the sequence.


  Then graph the sequence.
 
SOLUTION:   39. 0.125, –0.5, 2, …
Find the ratio of the consecutive terms.  
 
-2 I 0 2 SOLUTION:  
-=O - = undefined
--t 2 -2 0 Find the ratio of the consecutive terms.
   
Since the ratios are not same, the sequence is not -0.5 aa-4
')
---aa-4
geometric. 0.125 -0.5
   
ANSWER:   Since the ratios are same, the sequence is geometric
 
No
To find the next term, multiply the previous term with
 
−4.
Find the next three terms of the sequence.  
Then graph the sequence. 2x(-4)--8
  -8x(-4)=32
39. 0.125, –0.5, 2, …
32x(-4)- -128
 
 
SOLUTION:   Graph the sequence.
Find the ratio of the consecutive terms.  
 
-- 40
20
y
4

  -10 1 3 - 5 6 7X
I
Since the ratios are same, the sequence is geometric --40
  --60
--80
To find the next term, multiply the previous term with
-100
−4. -120
  I ••
 
ANSWER:  
– 8, 32, –128
   
Graph the sequence. ...._ 40 y
  - 20

-10 3 - 5 6 7X
I
1--40
""-60
---80
-100
-120 4
I

  40. 18, 12, 8, …
eSolutions Manual - Powered by Cognero   Page 15
ANSWER:  
– 8, 32, –128 SOLUTION:  
 
ANSWER:  
10-1  Sequences as Functions

 
40. 18, 12, 8, … ,y
  h8
h6
SOLUTION:   h4
h2
Find the ratio of the consecutive terms. ho
  8
12 2 8 2 6 ,Ii
-=- -=- 4
18 3 12 3
2
 
Since the ratios are same, the sequence is geometric.
,, 1 2 3 4 s 6 7 ax
   
To find the next term, multiply the previous term by
2 41. 64, 48, 36, …
..
.
.)
 
  SOLUTION:  
2 16 Find the ratio of the consecutive terms.
8x---
3 3  
16 2 32 48 3 36 3
-x-;-
3 3 9 64 4 48 4
32 2 6-t  
-x-=- Since the ratios are same, the sequence is geometric.
9 3 27
   
Graph the sequence. To find the next term, multiply the previous term by
  3
.
y 4
~8  
~6
h4 36xi _ 27
h2 4
~o 3 81
8 27x-;;;;-
6 ,. 81
4 ..
3 243
4 -x-;;;;-
2 4 4 16
In 1 2 3 4 s 6 7 ax  
  Graph the sequence.
 
ANSWER:   .y
80
70
60
  50
40 '
30 •
20
10
0 1 2 3 4 5 6 7 ax
 
ANSWER:  

eSolutions Manual - Powered by Cognero Page 16


   
41. 64, 48, 36, …
10-1  Sequences as Functions
 
ANSWER:  
81 243 ANSWER:  
?7--
- '4, 16 1024
192. 256. - .. -
  .)

80 ,  
70 ·'
--aoo
,
60 ,...35v
50 -soo
40 I ..... 250
30 -2vv
20 ,-..150
10 i-1 __
0 1234567 6X - ..... s:
J
  Jo 1 4 7X
 
42. 81, 108, 144, …
 
43. 
SOLUTION:  
 
Find the ratio of the consecutive terms.
  SOLUTION:  
108 4 144 4 Find the ratio of the consecutive terms.
81 3 108 3  
 
Since the ratios are same, the sequence is geometric
 
To find the next term, multiply the previous term by  
Since the ratios are same, the sequence is geometric
.  
3
To find the next term, multiply the previous term by
 
3.
144 x :!_ = 192  
3
192x~-256
3
4 1024  
'l56x-=--
- 3 3 Graph the sequence.
   
Graph the sequence.
 
-
41,.,. y
,-.. IUV

,...35u
'"""3uv
..... 25.,
..... 2~
.... ,so
t-luv
...- Sv
I
J ll 1 2 3 4 _§_ 7 x  
 
ANSWER:  
ANSWER:  
eSolutions Manual - Powered by Cognero 27, 81, 243 Page 17
 
10-1 Sequences as Functions
   

I 44. 1, 0.1, 0.01, 0.001, …


43.  ~.1.3.9 ....  
.)

  SOLUTION:  
SOLUTION:   Find the ratio of the consecutive terms.
Find the ratio of the consecutive terms.  
0.001
  Q:.!_ = 0.1 0.01 =0.1 --=0.1
" 9 I 0.1 0.01
I " .) "
-=j
I
- =.) -=3  
I 3
3 Since the ratios are same, the sequence is geometric.
   
Since the ratios are same, the sequence is geometric To find the next term, multiply the previous term by
  0.1.
To find the next term, multiply the previous term by  
3. 0.00 I x 0.1 = 0.000 I
  0.000 I x 0.1 = 0.0000 I
9x 3 = 27 0.0000 Ix 0.1 = 0.000001
27x3 =81  
81x3=243 Graph the sequence.
   
Graph the sequence. I y
..... 0.9
 
'-0.8
I .y
'-225 '-0.7
-200 --0.6
'-175 ..... 0.5
'-150 --0.4
-125 --0.3
..... 0.2
'-100
..... 75 ' '-0.1
I
..... 50 -10 1 2 3 4 6 x
- 25
'  
-10 1 ' 3 4 I I 7X
  ANSWER:  
0.0001, 0.00001, 0.000001
ANSWER:    
27, 81, 243 l .y
  --0.9
'-0.8
-225
l y • ..... 0.7
1-0.6
r200
-175 --o.s
-150 '-0.4
'-125 --0.3
'-100 '-0.2
- 75
..... 50
'-?,1
-10 1 2 3 4 6 x
..... 25
'  
-10 )34567X
  Determine whether each sequence is
arithmetic, geometric, or neither. Explain your
44. 1, 0.1, 0.01, 0.001, … reasoning.
   
eSolutions Manual - Powered by Cognero Page 18
45. 3, 12, 27, 48, …
SOLUTION:    
Find the ratio of the consecutive terms.
ANSWER:  
Neither; there is no common difference or ratio.
10-1 Sequences as Functions  
 

Determine whether each sequence is 46. 1, –2, –5, –8, …


arithmetic, geometric, or neither. Explain your  
reasoning.
  SOLUTION:  
45. 3, 12, 27, 48, … Subtract each term from the term directly after it.
   
-2 -5 -8
SOLUTION:   -3 -3 -3
Subtract each term from the term directly after it.
 
 
The common difference is –3.
3 12 27 48 Therefore, the sequence is arithmetic.
9 15 21  
  To find the common ratio, find the ratio of the
There is no common difference. consecutive terms.
   
Therefore, the sequence is not arithmetic. -2 -5 5 -8 8
---2
  I 2 2 5 5
To find the common ratio, find the ratio of the  
consecutive terms. Since the ratios are not same, the sequence is not
  geometric.
12 -4 27 9 .JS 16  
3 12 4 27 9
  ANSWER:  
Since the ratios are not same, the sequence is not Arithmetic; the common difference is –3.
geometric.  
 
47. 12, 36, 108, 324, …
ANSWER:    
Neither; there is no common difference or ratio. SOLUTION:  
 
Subtract each term from the term directly after it.
 
46. 1, –2, –5, –8, …
 
SOLUTION:    
Subtract each term from the term directly after it. There is no common difference.
  Therefore, this sequence is not arithmetic.
 
To find the common ratio, find the ratio of the
consecutive terms.
   
The common difference is –3.
Therefore, the sequence is arithmetic.
 
To find the common ratio, find the ratio of the  
consecutive terms. The common ratio is 3.
   
Since the ratios are same, the sequence is geometric.
 
  ANSWER:  
Since the ratios are not same, the sequence is not
eSolutions Manual - Powered by Cognero Geometric; the common ratio is 3. Page 19
geometric.  
   
 
ANSWER:  
ANSWER:   Geometric; the common ratio is 3.
10-1Arithmetic;
Sequencestheas common difference is –3.
Functions  
   
47. 12, 36, 108, 324, … 2 2 2 2
  48. 
5 25 125. 625
SOLUTION:    
Subtract each term from the term directly after it. SOLUTION:  
  Subtract each term from the term directly after it.
12 36 108 324 2 2 2 2
24 72 216 5 r_:, 125 625
  8 8 8
There is no common difference. 25 125 625
Therefore, this sequence is not arithmetic.  
  There is no common difference.
To find the common ratio, find the ratio of the Therefore, this sequence is not arithmetic.
consecutive terms.  
  To find the common ratio, find the ratio of the
36 -3 108 -3 324 -3 consecutive terms.
12 36 108  
  2
- 125 I
The common ratio is 3. -..,-=-:-
  - - 25 :,
Since the ratios are same, the sequence is geometric.  
  I
The common ratio is .
5
ANSWER:    
Geometric; the common ratio is 3. Since the ratios are the same, the sequence is
  geometric.
   
ANSWER:  
48. 
  Geometric; the common ratio is .
5
SOLUTION:    
Subtract each term from the term directly after it.
49. 
 
SOLUTION:  
  Subtract each term from the term directly after it.
There is no common difference.  
Therefore, this sequence is not arithmetic.
 
To find the common ratio, find the ratio of the
consecutive terms.
   
The common difference is .

  Therefore, the sequence is arithmetic.


 
The common ratio is .
eSolutions Manual - Powered by Cognero
To find the common ratio, find the ratio of the Page 20
consecutive terms.
   
Since the ratios are the same, the sequence is
ANSWER:   ANSWER:  

Geometric; the common ratio is . Arithmetic; the common difference is .


10-1 Sequences as Functions
   

50. 6, 9, 14, 21, …


49.   
  SOLUTION:  
SOLUTION:   Subtract each term from the term directly after it.
Subtract each term from the term directly after it.  
  6 9 14 21
3 5 7
-25 3 -72 4
 
There is no common difference.
-2I -2I -2I Therefore, the sequence is not arithmetic.
   
To find the common ratio, find the ratio of the
The common difference is -2I . consecutive terms.
 
Therefore, the sequence is arithmetic. ....)
9 ....) 14 14 21
  -=- -=- -=-
To find the common ratio, find the ratio of the 6 2 9 9 14 2
consecutive terms.  
  Since the ratios are not same, the sequence is not
geometric.
-=-
.,
.) 6
7
~=-
7., 4
-=-
8  
5 5 3 6 7 6
2 2 ANSWER:  
  Neither; there is no common difference or ratio.
Since the ratios are not same, the sequence is not  
geometric.
  51. READING Sareeta took an 800-page book on
vacation. If she was already on page 112 and is
ANSWER:  
going to be on vacation for 8 days, what is the
Arithmetic; the common difference is - . minimum number of pages she needs to read per day
2 to finish the book by the end of her vacation?
   
50. 6, 9, 14, 21, … SOLUTION:  
  The number of pages to be read is 800 – 112 or 688.
 
SOLUTION:   The minimum number of pages to read per day is
Subtract each term from the term directly after it.
  688 -86 .
8
 
 
 
There is no common difference. ANSWER:  
Therefore, the sequence is not arithmetic. 86 pg/day
   
To find the common ratio, find the ratio of the
consecutive terms. 52. DEPRECIATION Tammy’s car is expected to
  depreciate at a rate of 15% per year. If her car is
currently valued at $24,000, to the nearest dollar,
how much will it be worth in 6 years?
 
  Manual - Powered by Cognero
eSolutions Page 21
Since the ratios are not same, the sequence is not SOLUTION:  
geometric. Substitute 0.15, 6 and 24000 for r, t and P in
   
ANSWER:   ANSWER:  
10-186 pg/day as Functions
Sequences about 13,744 km
   

52. DEPRECIATION Tammy’s car is expected to 54. REASONING Explain why the sequence 8, 10, 13,
depreciate at a rate of 15% per year. If her car is 17, 22 is not arithmetic.
currently valued at $24,000, to the nearest dollar,  
how much will it be worth in 6 years?
  SOLUTION:  
Sample answer: The consecutive terms do not share
SOLUTION:   a common difference. For instance, 22 – 17 = 5,
Substitute 0.15, 6 and 24000 for r, t and P in while 17 – 13 = 4.
P(l-r)'  then evaluate.  
  ANSWER:  
24000( I - 0.15)" = 24000( 0.85}6 Sample answer: The consecutive terms do not share
a common difference. For instance, 22 – 17 = 5,
;,:;9052
while 17 – 13 = 4.
   
The worth of the car will be about $9052 after 6
years. 55. OPEN ENDED Describe a real-life situation that
  can be represented by an arithmetic sequence with a
common difference of 8.
ANSWER:  
 
$9052
  SOLUTION:  
Sample answer: A babysitter earns $20 for cleaning
53. CCSS REGULARITY  When a piece of paper is the house and $8 extra for every hour she watches
folded onto itself, it doubles in thickness. If a piece of the children.
paper that is 0.1 mm thick could be folded 37 times,  
how thick would it be?
  ANSWER:  
SOLUTION:   Sample answer: A babysitter earns $20 for cleaning
the house and $8 extra for every hour she watches
Given a 0 = 0.1, n = 37 and r = 2. the children.
   
Find a 37.
56. CHALLENGE The sum of three consecutive terms
  of an arithmetic sequence is 6. The product of the
terms is –42. Find the terms.
(Ir: 0.2(2}" I SOLUTION:  
=13745895350 Let x be the first term in the arithmetic sequence.
  Therefore, the next two terms should be x + d and x
+ 2d.
The thickness would be about 13,744 km.
 
ANSWER:  
about 13,744 km
 

54. REASONING Explain why the sequence 8, 10, 13,


17, 22 is not arithmetic.
 
SOLUTION:  
Sample answer: The consecutive terms do not share
eSolutions Manual - Powered by Cognero Page 22
a common difference. For instance, 22 – 17 = 5, Sample answer:
while 17 – 13 = 4. Let d = 5.
  Therefore, the terms are –3, 2, 7.
ANSWER:   Let d = 5.
Sample answer: A babysitter earns $20 for cleaning Therefore, the terms are –3, 2, 7.
the house and $8 extra for every hour she watches
10-1the children. as Functions
Sequences ANSWER:  
  –3, 2, 7

56. CHALLENGE The sum of three consecutive terms 57. ERROR ANALYSIS  Brody and Gen are
of an arithmetic sequence is 6. The product of the determining whether the sequence 8, 8, 8,… is
terms is –42. Find the terms. arithmetic, geometric, neither, or both. Is either of
them correct? Explain your reasoning.
SOLUTION:    
Let x be the first term in the arithmetic sequence.
Therefore, the next two terms should be x + d and x B"oJ.y
+ 2d.
.r + x + d + x + 2,/ = 6
Tue seq1>.e'o\ce \.\<"S ....
3x+3d =6 co""'""'o\\ J.,~eYe'o\Ce ~ O.
x+d=2 ~(1) -r\.,\e seq1>.e'o\ce is "'"'i-\.\~nc.
x(x+<l)(xT2d)= 42
x{2){2+ d)- -42
qe«
2x+.w/ =-21 ~ (2) Tiu U4fUfflU w a. C6HUf(4'(, rAiuJ

2x(l)-{2) (2-x)d = 25 tf t. Tiu U4fUfflU k jHH<eiric.


d1 -25
d=±5
Sample answer:  
Let d = 5. SOLUTION:  
Therefore, the terms are –3, 2, 7.
Sample answer: Neither; the sequence is both
ANSWER:   arithmetic and geometric.
 
–3, 2, 7
 
57. ERROR ANALYSIS  Brody and Gen are
ANSWER:  
determining whether the sequence 8, 8, 8,… is
arithmetic, geometric, neither, or both. Is either of Sample answer: Neither; the sequence is both
them correct? Explain your reasoning. arithmetic and geometric.
   

58. OPEN ENDED Find a geometric sequence, an


arithmetic sequence, and a sequence that is neither
geometric nor arithmetic that begins 3, 9,… .
 
SOLUTION:  
Sample answer: geometric: 3, 9, 27, 81, 243, … 
arithmetic: 3, 9, 15, 21, 27, … neither geometric nor
arithmetic: 3, 9, 21, 45, 93, ...
 
ANSWER:  
Sample answer: geometric: 3, 9, 27, 81, 243, … 
arithmetic: 3, 9, 15, 21, 27, … neither geometric nor
  arithmetic: 3, 9, 21, 45, 93, ...
 
SOLUTION:  
Sample answer: Neither; the sequence is both 59. REASONING If a geometric sequence has a ratio r
arithmetic and geometric. such that , what happens to the terms as n
 
  Manual - Powered by Cognero increases? What would happen to the terms if
eSolutions Page 23
?
ANSWER:    
Sample answer: Neither; the sequence is both
ANSWER:   fraction. When , the absolute value of the
Sample answer: geometric: 3, 9, 27, 81, 243, …  terms will increase and approach infinity because
arithmetic: 3, 9, 15, 21, 27, … neither geometric nor they are continuously being multiplied by a value
10-1arithmetic:
Sequences 3, as
9, 21, 45, 93, ...
Functions greater than 1.
   

59. REASONING If a geometric sequence has a ratio r 60. WRITING IN MATH Describe what happens to
such that , what happens to the terms as n the terms of a geometric sequence when the
increases? What would happen to the terms if common ratio is doubled. What happens when it is
halved? Explain your reasoning.
?
 
 
SOLUTION:  
SOLUTION:  
Sample answer: When the value of r is doubled, a 2
Sample answer: If a geometric sequence has a ratio
doubles, a 3 quadruples, a 4 is multiplied by 8, a5 is
r such that , as n increases, the absolute value
4
of the terms will decrease and approach zero multiplied by 2 or 16, and so on. So, the new terms
because they are continuously being multiplied by a are (I n = (I n · 2"- I . When the value of r is halved, the
fraction. When !rl ~ I , the absolute value of the ,,_,
terms will increase and approach infinity because
they are continuously being multiplied by a value
new terms are ()
a,,= a,,· ~ .

greater than 1.  
  ANSWER:  
ANSWER:   Sample answer: When the value of r is doubled, a 2
Sample answer: If a geometric sequence has a ratio doubles, a 3 quadruples, a 4 is multiplied by 8, a5 is
r such that , as n increases, the absolute value 4
multiplied by 2 or 16, and so on. So, the new terms
of the terms will decrease and approach zero
because they are continuously being multiplied by a are Cln - a" · 2" I. When the value of r is halved, the
,,_,
fraction. When , the absolute value of the
terms will increase and approach infinity because
new terms are a., =«: ~() .
they are continuously being multiplied by a value  
greater than 1.
  61. SHORT RESPONSE Mrs. Aguilar’s rectangular
bedroom measures 13 feet by 11 feet. She wants to
60. WRITING IN MATH Describe what happens to purchase carpet for the bedroom that costs $2.95 per
the terms of a geometric sequence when the square foot, including tax. How much will it cost to
common ratio is doubled. What happens when it is carpet her bedroom?
halved? Explain your reasoning.  
 
SOLUTION:  
SOLUTION:   2
The area of the bedroom is 13 × 11 or 143 ft .
Sample answer: When the value of r is doubled, a 2  
doubles, a 3 quadruples, a 4 is multiplied by 8, a5 is 143 x 2.95 = 421.85
4  
multiplied by 2 or 16, and so on. So, the new terms
It costs $421.85 to carpet the bedroom.
are . When the value of r is halved, the  
new terms are . ANSWER:  
$421.85
 
 
ANSWER:  
62. The pattern of filled circles and white circles below
Sample answer: When the value of r is doubled, a 2 can be described by a relationship between two
doubles, a 3 quadruples, a 4 is multiplied by 8, a5 is variables.
4  
multiplied by 2 or 16, and so on. So, the new terms
eSolutions Manual - Powered by Cognero Page 24
are . When the value of r is halved, the

new terms are .


  Option C is the correct answer.

ANSWER:   ANSWER:  
10-1$421.85
Sequences as Functions C
   

62. The pattern of filled circles and white circles below 63. SAT/ACT  Donna wanted to determine the average
can be described by a relationship between two of her six test scores. She added the scores correctly
variables. to get T, but divided by 7 instead of 6. Her average
  was 12 less than the actual average. Which equation
0 could be used to determine the value of T?
0
0 0 • 0  
oeo
O O • O• O
oeo

F   6T+12 =1T
 
O O G T _ T-12
  7- 6
Which rule relates w, the number of white circles, to  
f, the number of dark circles? H T +12=6 T
A U ;;.J/ 1 7
 
 
J T _ T-12
I
B f=-11'-1 6- 7
. 2  
  K
C ,r=2/ +I
 
 
I SOLUTION:  
D .f·;-11"
3 Donna’s average was 12 less than the actual
  average.
 
SOLUTION:   T T
The relation between w and f is II'"' 2/ + I . That is, -=--12 .
7 6
Option C is the correct answer.  
Rewrite the equation.
ANSWER:  
 
C
 

63. SAT/ACT  Donna wanted to determine the average  


of her six test scores. She added the scores correctly Option H is the correct answer.
to get T, but divided by 7 instead of 6. Her average  
was 12 less than the actual average. Which equation
could be used to determine the value of T? ANSWER:  
  H
F    
 
64. Find the next term in the geometric
G
sequence
 
H  
  A
J
 
 
B
K
 
 
C
SOLUTION:  
eSolutions Manual - Powered by Cognero Page 25

Donna’s average was 12 less than the actual  


average.
   
ANSWER:   ANSWER:  
10-1HSequences as Functions D
   

64. Find the next term in the geometric Solve each system of equations.
9 27  
sequence s.6.2·s····· y=S
65.  y·' "'X•+9
'
 
II
A  
8
  SOLUTION:  
27 Substitute 5 for y in the quadratic equation and solve
B 16 for x.
 
 
9 52 =x1 +9
C 4 x1 = 16
  .\'= ±.t
81
 
D 32
The solutions are (±4. 5).
   
SOLUTION:   ANSWER:  
Find the common ratio. (:t4.5)
 
6 9 ., 27  
-
8
-34 ~=
6
3
4
_8=3
9 4
2
  66. 
J
The common ratio is 4.  
  SOLUTION:  
The next term in the geometric sequence is Solve the linear equation for y in terms of x.
27 3 81
 
8 4 32.
 
Therefore, option D is the correct answer.  
  Substitute x + 1 for y in the quadratic equation and
ANSWER:   solve for x.
 
D
 

Solve each system of equations.


 
65. 

 
 
SOLUTION:  
By the Zero Product Property:
Substitute 5 for y in the quadratic equation and solve  
for x.
 

eSolutions Manual - Powered by Cognero   Page 26


Substitute the values of x in the linear equation and
solve for y.
 
ANSWER:  
ANSWER:  
10-1  Sequences as Functions (–4, –3), (3, 4)
 
y-x=I 3x-8/
66.  '
.r +y- , =25 67. 
8/-2x2 = 16
   
SOLUTION:   SOLUTION:  
Solve the linear equation for y in terms of x. 2
Substitute 3x for y in the second equation and solve
 
for x.
y-x=I
).T-2X1 = 16
y=x+I
-2x2+3x-16~0
 
Substitute x + 1 for y in the quadratic equation and x -3±J32 -4(-2)(-16)
solve for x.
2(-2)
 
x1 +(x+ 1)2 = 25 -3±~
x=
x2 + x2 + I + 2x = 25  
2x2 + 2.r - 24 = 0 Since the radicand is negative, there is no solution.
 
2(/ +.r-12) =0
ANSWER:  
2(x+ 4)(x-3) = 0
no solution
   
By the Zero Product Property:
  Write each equation in standard form. State
.T+4=0 or x-3=0 whether the graph of the equation is a parabola,
x =-4 or x=J circle, ellipse, or hyperbola. Then graph the
equation.
 
 
Substitute the values of x in the linear equation and
solve for y. 68. 
   
y=x+I and y=x+I  
)'.a.3+1 and y=-4+1 SOLUTION:  
)' = 4 and y=-3 Divide the equation by 6 on both sides.
   
The solutions are (–4, –3) and (3, 4).
 
ANSWER:    
(–4, –3), (3, 4) The equation is in the standard form of circle.
   

67. 

 
SOLUTION:  
2
Substitute 3x for y in the second equation and solve
for x.
eSolutions Manual - Powered by Cognero Page 27
 
ANSWER:  
 
ANSWER:  
10-1no solution as Functions
Sequences  
 

Write each equation in standard form. State 69.  4/-x2 +4-0


whether the graph of the equation is a parabola,
 
circle, ellipse, or hyperbola. Then graph the
equation. SOLUTION:  
  >
4 r-x·+4=0
>

68.  6x2 +6y2 -162 4 ,,2 -xl = -4


 
x:' y >
  ---=I
4 I
SOLUTION:    
Divide the equation by 6 on both sides. The equation is in the standard form of hyperbola.
   
.y
6x:' +6y- ' = 162
x·> + y-' =27 i-
.,,.
  '·
The equation is in the standard form of circle.
0 .i
  ,~
' ' ...
8 y -
4
I
 
-8 4 0 4 6X
7 ANSWER:  
.
~-4
hyperbola;
18

 
 
!Y
ANSWER:   ,._ '~
2 2 ,
circle; x + y = 27
 
0 x
., .....
.,
81Y

4
-
'
I

-8 4 0 4 8X  

.
-4
70. 
18
I
 
  SOLUTION:  

69. 
 
SOLUTION:  
 
The equation is in the standard form of circle.
eSolutions Manual - Powered by Cognero   Page 28

 
10-1 Sequences as Functions
 

70.  x~ + / + 6y + I J - 40 Graph each function.


   

SOLUTION:   71.  j "( r ) =~-,....,--"76


· {x-2)(x+3)
x2 + y2 + 6y + I J = 40  
x?+/+6y=27
SOLUTION:  
x2-y2 +6.r+9=27+9 (x-2)(x+3)=0
x1 +(y+3}2 =36 x=2 or x=-3
   
The equation is in the standard form of circle. The vertical asymptotes are at x = 2 and x = –3.
   
1 Since the degree of the numerator is less than the
4
degree of the denominator, the horizontal asymptote
is at y = 0.
-8
J
-4 0 '
4 ex
 
f(~
-4
, 2
6 \
1./ I ...
 
-6 -4 (l

2 f- '
4
. /I

ANSWER:  
I ,c~ - (.<- 2~x+ 3) I
14
2 2 I I I I I
circle; x + (y + 3) = 36
   
1
ANSWER:  
~ f(~

-6
'
... 4 0 '
4 81
2
\
-4
I I/ I ...
' • ' -6 -4 i
I
(l

"
4
. /I
I

I
14
2 f-
f(~ • (x- 2~x+ 3) I
Graph each function. I I I I I
   
71. 
72. 
 
SOLUTION:    
SOLUTION:  

 
The vertical asymptotes are at x = 2 and x = –3.
 
Since the degree of the numerator is less than the  
eSolutions Manual - Powered by Cognero Page 29
degree of the denominator, the horizontal asymptote The vertical asymptote is at x = 2.
is at y = 0.  
  Since the degree of the numerator is less than the
10-1  Sequences as Functions
 
~
-.> l.l -36
72.  f(x)- (x-2)l 73.  f(x)-. +6
x
   

SOLUTION:   SOLUTION:  
xi -36
(x-2)? =0 /(x)- x + 6
x-2=0
(.~ + 6)(x - 6)
x=2 = ·· x+6
 
The vertical asymptote is at x = 2. =x-6
   
Since the degree of the numerator is less than the xi -36
The graph of /(x)= r+6  is same as the graph of
degree of the denominator, the horizontal asymptote
is at y = 0. f(x) = x – 6.
   
f(.t) -3 ._j.
f(.t) ; ;
.J. 'l t(.-, = (x- 21f

. 0
, I!: -
..:: -& -4 0
,c x
~

' ' ~
I /{JC)• .!!.:2! .
K+6 1(3
(_

- ~ f

-
,~ c -1 2,
.J
   
ANSWER:   ANSWER:  
f(.t)~ ~
f{.-, = (x-
-3
21f
J 4
t(.t)-,-,

0 I!: -
..:: -a -4 0
,
ll. x
D t.
~
.l
Kl 36
.J. -4 ,
-~ ((JC)• K+6
~,, ~

~ I -::;_12
~
   

73.  74. HEALTH A certain medication is eliminated from


the bloodstream at a steady rate. It decays according
  to the equation y = ae
–0.1625t
, where t is in hours.
SOLUTION:   Find the half-life of this substance.
 
SOLUTION:  
Substitute 0.5a for y and solve for t.
 

  Manual - Powered by Cognero


eSolutions Page 30

The graph of  is same as the graph of


ANSWER:  
y = 0.5x + 1
10-1 Sequences as Functions  
 

74. HEALTH A certain medication is eliminated from


the bloodstream at a steady rate. It decays according 76. passes through
–0.1625t
to the equation y = ae , where t is in hours.  
Find the half-life of this substance.  
 
SOLUTION:  
SOLUTION:  
3
Substitute 0.5a for y and solve for t. Substitute --. 2 and for m, x1 and y 1 in the
  4 2
point-slope form of a line.
O.Sa = ae--0 162i,
 
e0.16251 =2
r-y1 aa.m(x-x1)
0.16251 "' In 2
ln2 y-2I 4(x-2)
3
aa-

I "'
0.1625 3 3 I
"'4.27
y=--x+-+-
4 2 2
  3
The half-life of the substance is about 4.27 hours. y=--x+2
  4
 
ANSWER:  
ANSWER:  
about 4.27 hours
  3
.
11=--x+2
4
Write an equation of each line.  
 
75. passes through (6, 4), m = 0.5 77. passes through (0, –6), m = 3
   
SOLUTION:   SOLUTION:  
Substitute 0.5, 6 and 4 for m, x1 and y 1 in the point- Substitute 3, 0 and –6 for m, x1 and y 1 in the point-
slope form of a line. slope form of a line.
   
y-y1 -m(x-.,·1) y- Y, "'111(.r-x1}

y-4 "'o.s(x-6) y-(-6) = 3(x-0)


y=O.Sx-3+4 y = 3.r-6
y = O.Sx+ I
 
ANSWER:  
ANSWER:  
y = 3x – 6
y = 0.5x + 1
 
 

78. passes through (0, 4),


76. passes through  
  SOLUTION:  
 
Substitute , 0 and 4 for m, x1 and y 1 in the point-
SOLUTION:  
eSolutions Manual - Powered by Cognero Page 31
slope form of a line.
Substitute for m, x1 and y 1 in the  
 
ANSWER:  
ANSWER:  
= 3x – 6 as Functions
10-1ySequences
   

I
78. passes through (0, 4), Ill= - 79. passes through (1, 3) and
4
   
SOLUTION:   SOLUTION:  
Find the slope of the line.
Substitute , 0 and 4 for m, x1 and y 1 in the point-
4  
slope form of a line. = Y1 - Y1
  111
x2-XJ
y-y1 =m(.r-x1)
I
I
- -3
y-4 =-(x-0) 2
4 8-1
I -7
y=-x+4 =-
4 14
  I
=--
ANSWER:   2
 
I
.l'--X+4
..)
Substitute
I
, 1 and 3 for m, x1 and y 1 in the point-
  2
slope form of a line.
 
79. passes through (1, 3) and Y-Yi = m(x-x1)
 
y-3 = - 2I (x-t)
SOLUTION:  
Find the slope of the line. I I
y=--x+-+3
  2 2
I 7
y=--x+-
2 2
 
ANSWER:  
I 7
y---x+-
2 2
 

80. passes through (–5, 1) and (5, 16)


 
 
SOLUTION:  
Substitute , 1 and 3 for m, x1 and y 1 in the point- Find the slope of the line.
slope form of a line.  
 

eSolutions Manual - Powered by Cognero Page 32

 
ANSWER:  

10-1 Sequences as Functions


 

80. passes through (–5, 1) and (5, 16)


 
SOLUTION:  
Find the slope of the line.
 
Ill=
)' l - y I

16-1
=
s-(-s)
J
=-
2
 
3
Substitute - , 5 and 16 for m, x1 and y 1 in the point-
2
slope form of a line.
 
y-y 1 =m(x-x,)
3
y-16 =
2(.r-5)
3 15
y=-x--+16
2 2
3 17
r=-x+-
. 2 2
 
ANSWER:  
3 17
v=-x+-
. 2 2
 

eSolutions Manual - Powered by Cognero Page 33


 
ANSWER:  
10-2 Arithmetic Sequences and Series 233
 

Find the indicated term of each arithmetic Write an equation for the nth term of each
sequence. arithmetic sequence.
   
1. a 1 = 14, d = 9, n = 11 3. 13, 19, 25, …
   

SOLUTION:   SOLUTION:  
19 – 13 = 6
ti,,=''•+ (11- l)tl 25 – 19 = 6
a11 =14+(11-1)9  
= 104 The common difference d is 6.
   
a 1 = 13
ANSWER:  
 
104
«,» (II+ (11- l)t/
 
a., = 13+(11-1)6
2. a 18 for 12, 25, 38, … = 13+611-6
=611+ 7
 
 
SOLUTION:  
25 – 12 = 13 ANSWER:  
38 – 25 = 13 a n = 6n + 7
   
The common difference d is 13.
 
a 1 = 12 4. a 5 = –12, d = –4

   
(I,,=<,, +(11-l}<I SOLUTION:  
a,8=12+(18-1)13 a,, =a1 +(11-l)d
=233 a, =a, +(5-1)(-4)
  -12=a1-16
ANSWER:   a, =4
233  
  Use the value of a 1 to find the nth term.
 
Write an equation for the nth term of each
arithmetic sequence. a., =4+(11-1)(-4)
  =4-4n+4
3. 13, 19, 25, … a,, =-4n+8
   
SOLUTION:  
ANSWER:  
19 – 13 = 6
25 – 19 = 6 a n = –4n + 8
   
The common difference d is 6.
  Find the arithmetic means in each sequence.
a = 13  
1 Manual - Powered by Cognero
eSolutions Page 1
5. 
 
 
   
ANSWER:   ANSWER:  
a n = –4n + 8Sequences and Series
10-2 Arithmetic –1, 2, 5
   

Find the arithmetic means in each sequence. Find the sum of each arithmetic series.
   
5.  6. __z _ _._: __..2._.42 7. the first 50 natural numbers
   
SOLUTION:  
SOLUTION:  
Here a 1 = 6 and a 5 = 42.
Sso = 50(1 0) +/
  50x51
(I,, = ll1 + (11 - 1 )t/ = 2
a5 = a, +(5 l)d = 1275
42= 6+4c/  
d =9
ANSWER:  
 
Therefore, the missing numbers are (6 + 9) or 15, 1275
(15 + 9) or 24, (24 + 9) or 33.  
 
8. 4 + 8 + 12 + … + 200
ANSWER:    
15, 24, 33
  SOLUTION:  
8– 4=4
-4. ? . ? . ? .8 12 – 8 = 4
6. 
 
---  
The common difference is 4.
  a, = 4. a,. = 200. d = 4
SOLUTION:  
Here a 1 = –4 and a 5 = 8.
  (Ill= ''1 + (11- l)d
 
"· = "• + (11- l)d 200=4+(11-1)-1
a5 =a, +(5-l)d 411 =200
8~-4+4d II= 50

,I= 3
 
  Find the sum of the series.
Therefore, the missing numbers are (–4 + 3) or −1,  
(–1 + 3) or 2, (2 + 3) or 5. ~ - ("• +a,
•>,, - II
2
,)
 
ANSWER:   s,. = so( ;oo)
4+
–1, 2, 5
  =5100

 
Find the sum of each arithmetic series.
  ANSWER:  
7. the first 50 natural numbers 5100
   
SOLUTION:  
9. a 1 = 12, a n = 188, d = 4
 
eSolutions Manual - Powered by Cognero Page 2
SOLUTION:  
   
ANSWER:   ANSWER:  
5100
10-2 Arithmetic Sequences and Series 1995
   

9. a 1 = 12, a n = 188, d = 4 Find the first three terms of each arithmetic
series.
 
 
SOLUTION:   11. a 1 = 8, a n = 100, S n = 1296
a,. =a, +(11-l)d  
188=12+(n-1)4 SOLUTION:  
411 = 180
n=45
S =
" II
(a, +a,.)
2
 
Find the sum of the series. 1296 = 11( S+ ~00)
 
24
S = 11(a,
"
+a,,)
2
 
II=

a,, =a1 +(u-l}d


s., =45(12+2188) I 00 = 8 + (24- l)d
=4500 23d =92
  d=4
 
ANSWER:   Therefore, the first three terms are 8, (8 + 4) or 12,
4500 (12 + 4) or 16.
   

10. a n = 145, d = 5, n = 21 ANSWER:  


  8, 12, 16
 
SOLUTION:  
a,1 =a1 +(n-l}d 12. n = 18, a n = 112, S n = 1098
145=a, +(21-1)5  
a, = 45
SOLUTION:  
 
Find the sum of the series.
 
S _-II (a, +a,,)
--
" 2

Si, =2{ 45~145)

=1995
   

ANSWER:  
1995
 

Find the first three terms of each arithmetic  


series. Therefore, the first three terms are 10, (10 + 6) or
  16, (16 + 6) or 22.
11. a 1 = 8, a n = 100, S n = 1296  
eSolutions Manual - Powered by Cognero Page 3
 
ANSWER:  
SOLUTION:   10, 16, 22
 
ANSWER:  
ANSWER:  
8, 12, 16
10-2 Arithmetic Sequences and Series 10, 16, 22
 
 

..,
12. n = 18, a n = 112, S n = 1098 12
13. MULTIPLE CHOICE Find ~:::<3k + 9).
 
SOLUTION:    
A 45
S"-11 - (",+",.)
2
 
B 78
1098: 18 o+ll2)
I
 
( 2 C 342
(11 =5  
2 D 410
a,= 10  
  SOLUTION:  
,,,, = <1, + (11-1),1 There are 12 – 1 + 1 or 12 terms, so n = 12.
112= 10+(18-l)d  
17d=l02 a, =3(1)+9 or 12
d=6 "" =3(12)+9 or45
   
Therefore, the first three terms are 10, (10 + 6) or Find the sum.
16, (16 + 6) or 22.  

( a,+a,,)
 
$ =II --
ANSWER:   " 2
10, 16, 22 s,, =12 ( 12 +45)
  2
=342
 
13. MULTIPLE CHOICE Find
Option C is the correct answer.
   
A 45
ANSWER:  
 
C
B 78
 
 
C 342 Find the indicated term of each arithmetic
  sequence.
D 410  
  14. a 1 = –18, d = 12, n = 16
SOLUTION:    
There are 12 – 1 + 1 or 12 terms, so n = 12. SOLUTION:  
 

   
Find the sum.
  ANSWER:  
162
eSolutions Manual - Powered by Cognero
  Page 4

15. a 1 = –12, n = 66, d = 4


   
ANSWER:   ANSWER:  
C
10-2 Arithmetic Sequences and Series –129
   

Find the indicated term of each arithmetic 17. a 15 for –5, –12, –19, …
sequence.
 
 
14. a 1 = –18, d = 12, n = 16 SOLUTION:  
  –12 – (–5) = –7
–19 – (–12) = –7
SOLUTION:    
,,,, = a1 + (11- l)tl The common difference d is –7.
 
"•• =-18+(16-1)12
a 1 = –5
=162
   
ll,, = lJ1 +(11-l)tl
ANSWER:  
"" =-5+(15-1)(-7)
162
  =-103
 

15. a 1 = –12, n = 66, d = 4 ANSWER:  


  –103
 
SOLUTION:  
ll,, =a, +(11-l)t.f 18. a 10 for –1, 1, 3, …

""" =-12+(66-1)4  
=248 SOLUTION:  
  1 – (–1) = 2
3– 1=2
ANSWER:    
248 The common difference d is 2.
   
a 1 = –1
16. a 1 = 9, n = 24, d = –6
 
 
ti,, =Cl1 + (11- l)tJ
SOLUTION:   a,o=-1+(10-1)2
t,,, =c11 +(11-l)ct
=17
"2, =9+(24-1)(-6)  
=-129 ANSWER:  
 
17
ANSWER:    
–129
  19. a 24 for 8.25, 8.5, 8.75, …
 
17. a 15 for –5, –12, –19, …
SOLUTION:  
  8.5 – 8.25 = 0.25
SOLUTION:   8.75 – 8.5 = 0.25
–12 – (–5) = –7  
–19 – (–12) = –7 The common difference d is 0.25.
  Manual - Powered by Cognero
eSolutions   Page 5
The common difference d is –7. a 1 = 8.25
   
 
ANSWER:  
ANSWER:   a n = 11n + 13
17
10-2 Arithmetic Sequences and Series  
 

19. a 24 for 8.25, 8.5, 8.75, … 21. 31, 17, 3, …


   

SOLUTION:   SOLUTION:  
8.5 – 8.25 = 0.25 17 – 31 = –14
8.75 – 8.5 = 0.25 3 – 17 = –14
   
The common difference d is 0.25. The common difference d is –14.
   
a 1 = 8.25 a 1 = 31

   
,,,, =<1, +(11-l)tl
,,,, =tt1 +(n-l)tf
a; =31+(11-1)(-14)
C/24 = 8.25 + (24 -1)0.25
=31-1411+14
=14
  =-1411+45
 
ANSWER:  
14 ANSWER:  
  a n = –14n + 45
 
Write an equation for the nth term of each
arithmetic sequence. 22. a 9 = 45, d = –3
 
20. 24, 35, 46, …  
  SOLUTION:  
SOLUTION:   <111 =a1 +(11-l)d
35 – 24 = 11 a9 =a, +(9-1)(-3)
46 – 35 = 11
45=a1-24
 
The common difference d is 11. a, =69
   
a 1 = 24 Use the value of a 1 to find the nth term.
   
a,, =tt1 +(11-l)cl + (n- 1)(-3)
t1,, = 69

a,, =24+(11-1)11 = 69-311 + 3


=24+ 1111-I I a; =-311+ 72
=1111+13  
 
ANSWER:  
ANSWER:   a n = –3n + 72
a n = 11n + 13  
 
23. a 7 = 21, d = 5

21. 31, 17, 3, …  
  SOLUTION:  
SOLUTION:  
eSolutions
17 Manual
– 31 =- –14
Powered by Cognero Page 6

3 – 17 = –14
 
   
ANSWER:   ANSWER:  
a n = –3n + 72
10-2 Arithmetic Sequences and Series a n = 0.25n + 11
   

23. a 7 = 21, d = 5 25. a 5 = 1.5, d = 4.5


   
SOLUTION:   SOLUTION:  
011 =a1 +(n-1)tl t111=a1+(11-l)tl
a, =a, +(7-1)5 a$ =a, +(5-1)(4.5)
21 =a, +30 l.5=a1 +18
"• = -9 a,= -16.5
   
Use the value of a 1 to find the nth term. Use the value of a 1 to find the nth term.
   
t1,, =-9+(n-1)5 t1,, =-16.S+(n-1}(4.5)
=-9+5n-5 = -16.5 + 4.Sn-4.5
a,, =Sn-14 a,,= 4.Sn-21
   
ANSWER:   ANSWER:  
a n = 5n – 14 a n = 4.5n – 21
   

24. a 4 = 12, d = 0.25 26. 9, 2, –5, …


   

SOLUTION:   SOLUTION:  
2 – 9 = –7
a; =a1 +(11-l)tl –5 – 2 = –7
a, =a, +(4-1)(0.25)  
12=a, +0.75 The common difference d is –7.
 
a, = I 1.25
a1 = 9
 
Use the value of a 1 to find the nth term.  
a,, =9+(n-1)(-7)
 
a,,= I 1.25+(n-1)(0.25) =9-7n+7
a,,= -7n+ 16
= 11.25 + 0.2511-0.25
 
a,, =0.25n+ 11
  ANSWER:  
a n = –7n + 16
ANSWER:  
a n = 0.25n + 11  
  27. a 6 = 22, d = 9
25. a 5 = 1.5, d = 4.5  
  SOLUTION:  

SOLUTION:  

eSolutions Manual - Powered by Cognero Page 7


   
ANSWER:   ANSWER:  
a n = –7n + 16 a n = –2n + 8
10-2 Arithmetic Sequences and Series
   

27. a 6 = 22, d = 9
29. 
 
 
SOLUTION:  
a,, =a1 +(11-l)d SOLUTION:  
t111 =<11 +(n-1)cl
a6 =a, +(6-1){9)
22=a, +45 a., =t,, +(15-l)(f)
a1 = -23
 
28
7=a, +-
3
Use the value of a 1 to find the nth term.
7
  (II:--
3
ti,, =-23+(11-1)(9)  
=-23+9n-9 Use the value of a 1 to find the nth term.
a,, =911-32  
 
ANSWER:  
a,, =-f +(11-1)(f)
7 2 2
a n = 9n – 32 =--+-11--
3 3 3
 
. =-::;11-J
2 '
28. a 8 = –8, d = –2
l/11
,
 
 
ANSWER:  
SOLUTION:  
2
a11 =<11 +(n-1)d tt,,=~n-3 ,
a3=a, +(8-1)(-2)  
-8=a, -14
30. –12, –17, –22, …
a, =6  
 
Use the value of a 1 to find the nth term. SOLUTION:  
–17 – (–12) = –5
 
–22 – (–17) = –5
a,, =6+(11-1)(-2)  
=6-2n+2 The common difference d is –5.
a,,= -2n+8  
  a 1 = –12
 
ANSWER:  
a n = –2n + 8
 
 
29. 
ANSWER:  
 
a n = –5n – 7
SOLUTION:    
eSolutions Manual - Powered by Cognero Page 8

31. 
 
ANSWER:  
ANSWER:  
10-2 Arithmetic Sequences and Series a n = –5n – 7
   

30. –12, –17, –22, …


  31. 

SOLUTION:    
–17 – (–12) = –5 SOLUTION:  
–22 – (–17) = –5
(111 = {11 + (11- l}t/
 
The common difference d is –5.
a, =a, +(3-1)(~)
 
a 1 = –12 4
--=u,+1
  5
a,, =-12+(n-1)(-5)
=-12-511+5
 
a,, =-5n-7
Use the value of a 1 to find the nth term.
 
 
ANSWER:  
a n = –5n – 7
«: =-~+(11-1)(~)
  9 1 1
=--+-11--
5 2 2
31.  a
1
=-11--
23
" 2 10
   
SOLUTION:   ANSWER:  
1 23
a =-n--
" 2 10
 

32. CCSS STRUCTURE  José averaged 123 total 
pins per game in his bowing league this season. He
is taking bowling lessons and hopes to bring his
average up by 8 pins each new season.
   
Use the value of a 1 to find the nth term. a. Write an equation to represent the nth term of
  the sequence.
 
b. If the pattern continues, during what season will
José average 187 per game?
 
c. Is it reasonable for this pattern to continue
indefinitely? Explain.
 
 
SOLUTION:  
ANSWER:   a. Given d = 8 and a 1 = 123.
Find the nth term.

 
eSolutions Manual - Powered by Cognero Page 9

32. CCSS STRUCTURE  José averaged 123 total 
pins per game in his bowing league this season. He  
 
ANSWER:   c. Sample answer: No; there are a maximum of 300
points in a bowling game, so it would be impossible
10-2 Arithmetic Sequences and Series for the average to continue to climb indefinitely.
   

32. CCSS STRUCTURE  José averaged 123 total  Find the arithmetic means in each sequence.


pins per game in his bowing league this season. He  
is taking bowling lessons and hopes to bring his 33.  24. ? , ? . ? . ? ,-1
average up by 8 pins each new season.
 
 
a. Write an equation to represent the nth term of SOLUTION:  
the sequence. Here a 1 = 24 and a 6 = –1.
 
 
b. If the pattern continues, during what season will
(111 :a1 +(11-l)tf
José average 187 per game?
  a6 =a, +(6-l)d
c. Is it reasonable for this pattern to continue -I =24+5d
indefinitely? Explain. d=-5
   
SOLUTION:   Therefore, the missing numbers are (24 – 5) or 19,
a. Given d = 8 and a 1 = 123. (19 – 5) or 14, (14 – 5) or 9, and (9 – 5) or 4.
 
Find the nth term.
a,, =a1 +(n-l)cl ANSWER:  
«, =123+(11-1)8 19, 14, 9, 4
 
a,,=115+8n
 
b. Substitute 187 for a n and solve for n. 34.  -6, ? . ? . ? . ? ,49 ]
 
187=115+811
&n= 72 SOLUTION:  
n=9 Here a 1 = –6 and a 6 = 49.
Therefore José’s average will be 187 pins per game  
th
in the 9 season. a,, =<11 +(11-l)tl
 
a6 =a, +(6-l)d
c. Sample answer: No; there are a maximum of 300
points in a bowling game, so it would be impossible 49=-6+5d
for the average to continue to climb indefinitely. d=II
   
Therefore, the missing numbers are (–6 + 11) or 5,
ANSWER:   (5 + 11) or 16, (16 + 11) or 27, and (27 + 11) or 38.
a. a n = 115 + 8n  
 
ANSWER:  
b. 9th season
5, 16, 27, 38
   
c. Sample answer: No; there are a maximum of 300
points in a bowling game, so it would be impossible 35. 
for the average to continue to climb indefinitely.
   
SOLUTION:  
Find the arithmetic means in each sequence.
  Here a 1 = –28 and a 6 = 7.
33.   
 
eSolutions Manual - Powered by Cognero Page 10

SOLUTION:  
Here a 1 = 24 and a 6 = –1.
   
ANSWER:   ANSWER:  
5, 16, 27, 38 Sequences and Series
10-2 Arithmetic 75, 66, 57, 48
   

35.  -28, ? . ? . ? , ? . 7 37.  -12. ? . ? . ? . ? , ? .-66


   
SOLUTION:   SOLUTION:  
Here a 1 = –28 and a 6 = 7. Here a 1 = –12 and a 7 = –66.
   
,,,, = ,,, + (11- l}d ,,,, =<ti +(11-l)<l
a6 =a, +(6-l)d a, =a, +(7-l)d
7=-28+5d -66 =-12+6d
d=1 d=-9
   
Therefore, the missing numbers are (–28 + 7) or – Therefore, the missing numbers are (–12 – 9) or –
21, (–21 + 7) or –14, (–14 + 7) or –7, and (–7 + 7) 21, (–21 – 9) or –30, (–30 – 9) or –39, (–39 – 9) or
or 0. –48, and (–48 – 9) or –57.
   
ANSWER:   ANSWER:  
–21, –14, –7, 0 –21, –30, –39, –48, –57
   

36.  84. ? , ? . ? . ? . 39 38.  182. ? , ? . ? . ? . ? , I 04


   
SOLUTION:   SOLUTION:  
Here a 1 = 84 and a 6 = 39. Here a 1 = 182 and a 7 = 104.
   
<t,, =a1 +(11-l)cl a,,= a,+ (11- l)d
a6 =a, +(6-l)d a, =a, +(7-l)d
39 =84 + 5d 104=182+6d
d=-9 d=-13
   
Therefore, the missing numbers are (84 – 9) or 75, Therefore, the missing numbers are (182 – 13) or
(75 – 9) or 66, (66 – 9) or 57, and (57 – 9) or 48. 169, (169 – 13) or 156, (156 – 13) or 143, (143 –
  13) or 130, and (130 – 13) or 117.
 
ANSWER:  
75, 66, 57, 48 ANSWER:  
  169, 156, 143, 130, 117
 
37. 
Find the sum of each arithmetic series.
 
 
SOLUTION:   39. the first 100 even natural numbers
Here a 1 = –12 and a 7 = –66.  
  SOLUTION:  
Here a 1 = 2 and a 100 = 200.
 
eSolutions Manual - Powered by Cognero Find the sum. Page 11
 
 
   
ANSWER:   ANSWER:  
169, 156, 143,Sequences
10-2 Arithmetic 130, 117 and Series 40,000
   

Find the sum of each arithmetic series. 41. the first 100 odd natural numbers
   
39. the first 100 even natural numbers
SOLUTION:  
 
Here a 1 = 1 and a 100 = 199.
SOLUTION:  
 
Here a 1 = 2 and a 100 = 200. n = 100
   
Find the sum. Find the sum.
   
S" = u (a, +a,.)
2
S=(a,+a,.)2
,, II

= 100(2 +;oo) = 100(1 +;99)


= 10,100 = 10.000
   
ANSWER:   ANSWER:  
10,100 10,000
   

42. the first 300 even natural numbers


40. the first 200 odd natural numbers
   
SOLUTION:  
SOLUTION:  
Here a 1 = 2 and a 300 = 300.
Here a 1 = 1 and a 200 = 399.
   
Find the sum. n = 300
   
S =
,, II
(a, +a,.)
2
Find the sum.
 
S =
" II
(a, +a,.)
2
= 200(1 + i99)

=40.000 = 300( 2+;00)


 
=90,300
ANSWER:    
40,000
ANSWER:  
 
90,300
41. the first 100 odd natural numbers  
 
43. –18 + (–15) + (–12) + … + 66
SOLUTION:    
Here a 1 = 1 and a 100 = 199.
SOLUTION:  
  –15 – (–18) = 3
n = 100 –12 – (–15) = 3
   
eSolutions Manual - Powered by Cognero Page 12
Find the sum. The common difference is 3.
   
   
ANSWER:   ANSWER:  
90,300
10-2 Arithmetic Sequences and Series 696
   

43. –18 + (–15) + (–12) + … + 66 44. –24 + (–18) + (–12) + … + 72


   
SOLUTION:   SOLUTION:  
–15 – (–18) = 3 –18 – (–24) = 6
–12 – (–15) = 3 –12 – (–18) = 6
   
The common difference is 3. The common difference is 6.
   
a, =-18 a, =-24
a,. =66 Cl,, ; 72
   
Find the value of n. Find the value of n.
   
a,, =tt1 +(11-l)cl c,,, =<1 1 +(u-l)cl
66=-18+(11-1)3 72=-24+(11-1)6
311 = 87 611 = 102
II =29 II: 17
   
Find the sum. Find the sum.
   

   
ANSWER:   ANSWER:  
696 408
   

44. –24 + (–18) + (–12) + … + 72 45. a 1 = –16, d = 6, n = 24


   
SOLUTION:   SOLUTION:  
–18 – (–24) = 6
–12 – (–18) = 6 S,. = %[2c,, + (11- l)c!]
 
The common difference is 6. S2, = 224[2(-16)+(24-1)6]
 
= 1272
 
  ANSWER:  
Find the value of n. 1272
   

46. n = 19, a n = 154, d = 8


eSolutions Manual - Powered by Cognero   Page 13

SOLUTION:  
   
ANSWER:   ANSWER:  
1272
10-2 Arithmetic Sequences and Series 1558
   

46. n = 19, a n = 154, d = 8 47. CONTESTS The prizes in a weekly radio contest


  began at $150 and increased by $50 for each week
that the contest lasted. If the contest lasted for
SOLUTION:   eleven weeks, how much was awarded in total?
Find the value of a 1.  
a,, =a, +(n-l)d SOLUTION:  
154=a1 +(19-1)8 Given, a 1 = 150, d = 50 and n = 11.
a,= 10  
  Find the value of a 11.
Find the sum.  
  a,, =a, +(11-l)d
= 150+ 10(50)
=650
 
Find the sum.
 
 
S : II
(
a,+a) II

" 2
ANSWER:  
1558 = I { 150;650)
 
=4400
47. CONTESTS The prizes in a weekly radio contest  
began at $150 and increased by $50 for each week A cash prizes totaled $4400 for the eleven week
that the contest lasted. If the contest lasted for contest.
eleven weeks, how much was awarded in total?  
 
ANSWER:  
SOLUTION:  
$4400
Given, a 1 = 150, d = 50 and n = 11.  
 
Find the first three terms of each arithmetic
Find the value of a 11.
series.
   
48. n = 32, a n = –86, S n = 224
 
SOLUTION:  
 
Find the sum. Find the value of a 1.
   

 
A cash prizes totaled $4400 for the eleven week
contest.
eSolutions Manual - Powered by Cognero Page 14
   
Find the value of d.
 
ANSWER:  
ANSWER:  
100, 94, 88
$4400
10-2 Arithmetic Sequences and Series  
 

Find the first three terms of each arithmetic 49. a 1 = 48, a n = 180, S n = 1368
series.
 
 
48. n = 32, a n = –86, S n = 224 SOLUTION:  
  Find the value of n.

SOLUTION:   S,, --JI


L
(a, +u,,)
--
2
Find the value of a 1.
  1368 = 11( 48~180)
S,, = II (a, +a,
2 ,) 1368= 11411
II= 12

224 =32( a, ;86)  


Find the value of d.
224 = 16a1 -1376 ,,,, = a1 + (11- l)d
1600 = 16a1 180=48+(12-l)d
a1 = 100 132= I Id
  t/=12
Find the value of d.  
  So, the sequence is 48, 60, 72, …
a11 =t11 +(11-l)cl  
-86=100+(32-l)d ANSWER:  
-186=3 Id 48, 60, 72
d=-6  
 
So, the sequence is 100, 94, 88, … 50. a 1 = 3, a n = 66, S n = 759
   

ANSWER:   SOLUTION:  
100, 94, 88 Find the value of n.
   

49. a 1 = 48, a n = 180, S n = 1368


 
SOLUTION:  
Find the value of n.  
Find the value of d.
 

   
Find the value of d. Therefore, the first three terms are 3, 6 and 9.
 
eSolutions Manual - Powered by Cognero Page 15
ANSWER:  
3, 6, 9
 
   
ANSWER:   ANSWER:  
48, 60, 72 Sequences and Series
10-2 Arithmetic 3, 6, 9
   

50. a 1 = 3, a n = 66, S n = 759 51. n = 28, a n = 228, S n = 2982


   
SOLUTION:   SOLUTION:  
Find the value of n. Find the value of a 1.
   
S,, =11(0• +a,
2 ,) s; --JI((I' +Cl
2 ,.)

T
759=11 ( ' + 66) 2982 = 2s( a, \228)
II= 22
  a, +I 14=106.5
Find the value of d. 2
  a, =-15
''" =ti, +(11-1),1  
Find the value of d.
66=3+(22-l)d  
21d =63 lt,, =(ti +(11-1),t
d=3 228 = -15+(28- l)d
 
Therefore, the first three terms are 3, 6 and 9. 27d =243
  d=9
 
ANSWER:   Therefore, the first three terms are –15, –6 and 3.
3, 6, 9  
 
ANSWER:  
51. n = 28, a n = 228, S n = 2982 –15, –6, 3
   

SOLUTION:   52. a 1 = –72, a n = 453, S n = 6858


Find the value of a 1.  
  SOLUTION:  
Find the value of n.
 

   
Find the value of d. Find the value of d.
   

eSolutions Manual - Powered by Cognero Page 16


   
Therefore, the first three terms are –15, –6 and 3. Therefore, the first three terms are –72, –57 and –
  42.
   
ANSWER:   ANSWER:  
–15, –6, 3 Sequences and Series
10-2 Arithmetic –72, –57, –42
   

52. a 1 = –72, a n = 453, S n = 6858 53. n = 30, a n = 362, S n = 4770


   
SOLUTION:   SOLUTION:  
Find the value of n. Find the value of a 1.
   
S =
II II
(a, +a,
2
.) S = 11((/' 2 +a,.)
'II

6858=11 (
-72 + 453)
2 4 770 = 30( a, +/ 62)
II= J6 a, +362 =318
 
Find the value of d. a, =-44
   
a11 =c11 +(u-l)cl Find the value of d.
 
453=-72+(36-l)d
Ct,,= Ct1 + (11- l)cf
35d = 525
362=-44+(30-l)d
d=l5
  29d =406
Therefore, the first three terms are –72, –57 and – d=l4
42.  
  Therefore, the first three terms are –44, –30 and –
16.
ANSWER:    
–72, –57, –42
  ANSWER:  
–44, –30, –16
53. n = 30, a n = 362, S n = 4770  
 
54. a 1 = 19, n = 44, S n = 9350
SOLUTION:    
Find the value of a 1.
SOLUTION:  
 
Find the value of a n.
 

 
Find the value of d.
 
 
Find the value of d.
 

eSolutions Manual - Powered by Cognero Page 17


 
Therefore, the first three terms are –44, –30 and –
16.
   
ANSWER:   ANSWER:  
–44, –30, –16Sequences and Series
10-2 Arithmetic 19, 28, 37
   

54. a 1 = 19, n = 44, S n = 9350 55. a 1 = –33, n = 36, S n = 6372


   
SOLUTION:   SOLUTION:  
Find the value of a n. Find the value of a n.
  L,,S' _,, +a.)
- ("• 2  
S' = II
• II
(a,+ a.)
2

9350 = -14('9; "·)


6372 = 36(-33/ a,.)
.!2_+ a,. =212.S ",. -33 = 35-l
2 2
a.= 406 «: =387
 
  Find the value of d.
Find the value of d.  
  « ::,,1 +(11-l)cl (/11 3(/1 +(11-l)<f
406= 19+(44-l)d 387=-33+(36-l)d
43d=387 35d =420
d=9 c/=12

 
  Therefore, the first three terms are –33, –21 and –
Therefore, the first three terms are 19, 28 and 37. 9.
   
ANSWER:   ANSWER:  
19, 28, 37 –33, –21, –9
   

55. a 1 = –33, n = 36, S n = 6372 56. PRIZES A radio station is offering a total of $8500
  in prizes over ten hours. Each hour, the prize will
increase by $100. Find the amounts of the first and
SOLUTION:   last prize.
Find the value of a n.  
  SOLUTION:  
Given n = 10, d = 100 and S 10 = 8500.
 
Find the value of a 1.
 

 
Find the value of d.
 

 
eSolutions Manual - Powered by Cognero Find the value of a 10. Page 18

 
 
   
ANSWER:   ANSWER:  
–33, –21, –9Sequences and Series
10-2 Arithmetic $400 and $1300
   

56. PRIZES A radio station is offering a total of $8500 Find the sum of each arithmetic series.
in prizes over ten hours. Each hour, the prize will  
increase by $100. Find the amounts of the first and 16

last prize. 57.  I<4k-2>


hi
 
 
SOLUTION:  
SOLUTION:  
Given n = 10, d = 100 and S 10 = 8500.
There are 16 – 1 + 1 or 16 terms, so n = 16.
   
Find the value of a 1. a, =4(1)-2 or2
  "1• =4(16)-2 or 62
s =2"[ 2a, +(11-l)d]
-»  
Find the sum.
10[ 2c,1 +(10-1)100]
S,o =2  

8500 = s[2a, + 900) SII = II ( a+a)


I "
2
a, = 400
  s, = 16( 2 ~62)
Find the value of a 10. =512
   
16
"10 = a1 + (11- l}tl
Therefore, I<4k-2>=s12 .
=400+(10-1)100 l•I

= 1300  
 
ANSWER:  
ANSWER:   512
$400 and $1300  
 

Find the sum of each arithmetic series. 58. 


 
 
57. 
SOLUTION:  
  There are 13 – 4 + 1 or 10 terms, so n = 10.
 
SOLUTION:  
There are 16 – 1 + 1 or 16 terms, so n = 16.
 
 
Find the sum.
 
 
Find the sum.
 

 
eSolutions Manual - Powered by Cognero Page 19

Therefore, .
 
 
ANSWER:  
ANSWER:  
512
10-2 Arithmetic Sequences and Series 350
 
 

.
l.t 16
58.  ~:,: < 4k + I) 59.  I<2k +6>
k•.S
   
SOLUTION:   SOLUTION:  
There are 13 – 4 + 1 or 10 terms, so n = 10. There are 16 – 5 + 1 or 12 terms, so n = 12.
   
a, =4(4)+1 or 17 a, =2(5)+6orl6
,,10=4{13)+1 or53 a12 = 2(16)+6 or 38
   
Find the sum. Find the sum.
   

   

.,
H 16
Therefore, L(4k + I) =350 . Therefore, I<2k + 6> = 324 .
k•.S
   
ANSWER:   ANSWER:  
350 324
   

59.  60. 

   
SOLUTION:   SOLUTION:  
There are 16 – 5 + 1 or 12 terms, so n = 12. There are 12 – 0 + 1 or 13 terms, so n = 13.
   

   
Find the sum. Find the sum.
   

   

Therefore, . Therefore, .
eSolutions Manual - Powered by Cognero Page 20
   
ANSWER:   ANSWER:  
   
ANSWER:   ANSWER:  
324
10-2 Arithmetic Sequences and Series –208
   

60. 
..."
.t<-3k+2)
61. FINANCIAL LITERACY Daniela borrowed
some money from her parents. She agreed to pay
$50 at the end of the first month and $25 more each
  additional month for 12 months. How much does
SOLUTION:   she pay in total after the 12 months?
There are 12 – 0 + 1 or 13 terms, so n = 13.  
  <1<,= -3(0)+2or2 SOLUTION:  
a., =-3(12)+2 or -34 Given a 1 = 50, d = 25 and n = 12.
 
  Find the sum.
Find the sum.  
  S,, = "(__,a,_;_a,,..
) ~ "[
·• =2 2a, +(11-l)d J
)?
s,1 =
s,. = 13(2~34) 2-[2(so)+(12-1)2s]
=2250
=-208
 
She pays $2250.
  " + 2J =-20s
I<-Jk  
Therefore, ' 0 .
ANSWER:  
  $2250
 
ANSWER:  
–208 62. GRAVITY When an object is in free fall and air
  resistance is ignored, it falls 16 feet in the first
second, an additional 48 feet during the next second,
61. FINANCIAL LITERACY Daniela borrowed and 80 feet during the third second. How many total
some money from her parents. She agreed to pay feet will the object fall in 10 seconds?
$50 at the end of the first month and $25 more each  
additional month for 12 months. How much does
SOLUTION:  
she pay in total after the 12 months?
  Given a 1 = 16, n = 10.
 
SOLUTION:  
The common difference d is 32.
Given a 1 = 50, d = 25 and n = 12.  
  Find the sum.
Find the sum.  
 

 
  The object will fall 1600 ft in 10 seconds.
She pays $2250.  
 
ANSWER:  
ANSWER:   1600 ft
eSolutions Manual - Powered by Cognero   Page 21
$2250
 
Use the given information to write an equation
   
ANSWER:   ANSWER:  
$2250
10-2 Arithmetic Sequences and Series 1600 ft
   

62. GRAVITY When an object is in free fall and air Use the given information to write an equation
resistance is ignored, it falls 16 feet in the first that represents the nth term in each arithmetic
second, an additional 48 feet during the next second, sequence.
and 80 feet during the third second. How many total  
feet will the object fall in 10 seconds? 63. The 100th term of the sequence is 245. The
  common difference is 13.
 
SOLUTION:  
Given a 1 = 16, n = 10. SOLUTION:  
  Given a 100 = 245, d = 13 and n = 100.
The common difference d is 32.  
  Find the value of a 1.
Find the sum.
 
 
a,, =a1 +(n-J}d
5;, = f[ 2a1 + (11- l)d] 245=a, +(100-1)13
12°[2(16)+(10-1)32] a,=-1042
=
 
=1600 Substitute the values of a 1 and d to find the nth
 
term.
The object will fall 1600 ft in 10 seconds.
 
 
c,11 =a1 +(n-l}d
ANSWER:   a,, =-1042+(11-1)13
1600 ft
a; =1311-1055
 
 
Use the given information to write an equation
that represents the nth term in each arithmetic ANSWER:  
sequence. a n = 13n – 1055
   
63. The 100th term of the sequence is 245. The
common difference is 13.
64. The eleventh term of the sequence is 78. The
 
common difference is –9.
SOLUTION:    
Given a 100 = 245, d = 13 and n = 100. SOLUTION:  
  Given a 11 = 78, d = –9 and n = 11.
Find the value of a 1.  
  Find the value of a 1.
 

 
Substitute the values of a 1 and d to find the nth  
term. Substitute the values of a 1 and d to find the nth
 
term.
 
eSolutions Manual - Powered by Cognero Page 22
 
ANSWER:  
ANSWER:  
a n = 13n – 1055
10-2 Arithmetic Sequences and Series a n = –9n + 177
 
 

64. The eleventh term of the sequence is 78. The 65. The sixth term of the sequence is –34. The 23rd
common difference is –9. term is 119.
   
SOLUTION:   SOLUTION:  
Given a 11 = 78, d = –9 and n = 11. Given a 6 = –34 and a 23 = 119.
   
Find the value of a 1. Therefore, there are (23 – 6 + 1) or 18 terms
between –34 and 119.
 
 
(111 =a1 +(n-l}d
Find the common difference of the series with a 1 =
78=a1 +(I 1-1)(-9)
–34 and a 18 = 119.
a,=168
 
  (111 = a1 + (11- J)tJ
Substitute the values of a 1 and d to find the nth
119=-34+(18-l)d
term.
  d=9
a,, =a1 +(n-1)(1  
Find the value of a 1.
a,, =168+(11-1)(-9)
 
a; = -911 + 177 a,, =a1 +(n-l)d
 
I 19=a, +(23-1)9
ANSWER:   a, =-79
a n = –9n + 177  
  Substitute the values of a 1 and d to find the nth
term.
65. The sixth term of the sequence is –34. The 23rd
 
term is 119.
  c,11 =a1 +(n-l}d
a,, =-79+(11-1)9
SOLUTION:  
a; = 911-88
Given a 6 = –34 and a 23 = 119.
 
 
Therefore, there are (23 – 6 + 1) or 18 terms ANSWER:  
between –34 and 119.
a n = 9n – 88
 
Find the common difference of the series with a 1 =  
–34 and a 18 = 119. 66. The 25th term of the sequence is 121. The 80th
  term is 506.
 
SOLUTION:  
Given a 25 = 121 and a 80 = 506.
   
Find the value of a 1. Therefore, there are (80 – 25 + 1) or 56 terms
between 121 and 506.
 
 
Find the common difference of the series with a 1 =
eSolutions Manual - Powered by Cognero Page 23
121 and a 56 = 506.
 
  arrangements.
 
ANSWER:  
a n = 9n – 88Sequences and Series
10-2 Arithmetic
 
 
66. The 25th term of the sequence is 121. The 80th a. Make drawings to find the next three numbers as
term is 506. tables are added one at a time to the arrangement.
   
b. Write an equation representing the nth number in
SOLUTION:  
this pattern.
Given a 25 = 121 and a 80 = 506.  
  c. Is it possible to have seating for exactly 100
Therefore, there are (80 – 25 + 1) or 56 terms people with such an arrangement? Explain.
between 121 and 506.  
 
SOLUTION:  
Find the common difference of the series with a 1 =
a. For each increase in the number of table, the
121 and a 56 = 506. number of people who can sit is increased by 4.
  That is, the common difference is 4.
+(n-l)tl
(Jn =Cl1 Therefore, the next three numbers are (10 + 4) or
14, (14 + 4) or 18 and (18 + 4) or 22.
506=121+(56-l)d
d=7
 
Find the value of a 1.
 
an= a, +(n-l)d
506 = "• + (80- 1) 7
a, = -47
 
Substitute the values of a 1 and d to find the nth
term.  
  b. Substitute a 1 = 6 and d = 4 in a.= a,+ (11- l)d .
a,, =a1 +(n-l)cl
"· =6+(11-1)4
a,, =-47+(n-1)7 =6+411-4
a,,=7n-54 = 411+ 2
   
c. No; there is no whole number n for which
ANSWER:  
411+ 2 = 100 .
a n = 7n – 54  
 
ANSWER:  
67. CCSS MODELING  The rectangular tables in a a. 14, 18, 22
reception hall are often placed end-to-end to form
one long table. The diagrams below show the
number of people who can sit at each of the table
arrangements.
 

 
a. Make drawings to find the next three numbers as
eSolutions Manual
tables are- added
Poweredone
by Cognero
at a time
to the arrangement. Page 24
 
b. Write an equation representing the nth number in  
c. No; there is no whole number n for which  
.
  ANSWER:  
10-2 Arithmetic Sequences and Series 13
ANSWER:    
a. 14, 18, 22
69. SALARY Terry currently earns $28,000 per year.
If Terry expects a $4000 increase in salary every
year, after how many years will he have a salary of
$100,000 per year?
 
SOLUTION:  
Given a 1 = 28000, d = 4000 and a n = 100000.
 
Substitute the values of a 1, a n and d and solve for
n.
 
 
b. p n = 4n + 2 a,, =c,1 +(11-l)d

  I 00000 = 28000 + (11 - I) 4000


c. No; there is no whole number n for which 4n + 2 400011 = 76000
= 100. II= 19
   
So he will have a salary of $100,000 per year after
68. PERFORMANCE A certain company pays its the 19th year.
employees according to their performance. Belinda  
is paid a flat rate of $200 per week plus $24 for
every unit she completes. If she earned $512 in one ANSWER:  
week, how many units did she complete? the 19th year
   
SOLUTION:  
70. SPORTS While training for cross country, Silvia
Given a 1 = 200, d = 24 and a n = 512. plans to run 3 miles per day for the first week, and
  then increase the distance by a half mile each of the
Substitute the values of a 1, a n and d and solve for following weeks.
 
n.
a. Write an equation to represent the nth term of
 
the sequence.
an =c,. +(11-l)ct  
512 = 200+ (11-1)24 b. If the pattern continues, during which week will
2411 = 336 she be running 10 miles per day?
II= 14  
  c. Is it reasonable for this pattern to continue
14th term in the sequence is 512. indefinitely? Explain.
   
Therefore, she completed (14 – 1) or 13 units. SOLUTION:  
  a. Given a 1 = 3 and d = 0.5.
ANSWER:    
13
 

69. SALARY Terry currently earns $28,000 per year.


If Terry expects a $4000 increase in salary every
year, after how many years will he have a salary of
$100,000
eSolutions Manual -per year?by Cognero
Powered Page 25
 
SOLUTION:  
  b. 15th wk
 
ANSWER:   c. Sample answer: No; eventually the number of
the 19th yearSequences and Series
10-2 Arithmetic miles per day will become unrealistic.
   

70. SPORTS While training for cross country, Silvia 71. MUTIPLE REPRESENTATIONS Consider
plans to run 3 miles per day for the first week, and •
then increase the distance by a half mile each of the I<2k+2).
k•I
following weeks.
   
a. Write an equation to represent the nth term of a. TABULAR Make a table of the partial sums of
the sequence. the series for 1 ≤ k ≤ 10.
   
b. If the pattern continues, during which week will b. GRAPHICAL Graph (k, partial sum).
she be running 10 miles per day?  
  c. GRAPHICAL Graph f (x) = x2 + 3x on the
c. Is it reasonable for this pattern to continue same grid.
indefinitely? Explain.  
  d. VERBAL What do you notice about the two
graphs?
SOLUTION:    
a. Given a 1 = 3 and d = 0.5. e. ANALYTICAL What conclusions can you
  make about the relationship between quadratic
Find the nth term. functions and the sum of arithmetic series?
   
a,, = a, + (11- l}d f. ALGEBRAIC Find the arithmetic series that
2
=3+(11-1)0.5 relates to g(x) = x + 8x.
 
= 0.511 + 2.5
  SOLUTION:  
b. Substitute 10 for a n in a. = 0.511 + 2.5  and solve  a.
for n.
1 4
 
2 10
IO= 0.511 + 2.5 3 18
11 = 15 4 28
  5 40
During 15th week, she will be running 10 miles per 6 54
day. 7 70
8 88
 
9 108
c. Sample answer: No; eventually the number of 10 130
miles per day will become unrealistic.
   
b.
ANSWER:   I
1l5 ~-+++-H-+++-i
a. a n = 2.5 + 0.5n 120 H-+++-H-++-t-1
e 105 ~--1-~4--'-ll-l--1--f-l
~ 90 1-1-4---1-+-'-l-+-r+-i
 
j 75
b. 15th wk ~ 60 ~--1-~4--'-ll-l-+-H
  45 l--l--+--1-+-+-,f--+-+-+-,
30 l--l--+--1--1.--+-,f--+-+-+-,
c. Sample answer: No; eventually the number of 15 ~~ ..... 4--'-ll-l--1--H
miles per day will become unrealistic.
  0 1234567891
Term
71. MUTIPLE REPRESENTATIONS Consider  
c.
eSolutions Manual - Powered by Cognero Page 26

 
a. TABULAR Make a table of the partial sums of
10-2 Arithmetic Sequences and Series
   
c. c.
y y
us us
120
e IOS
'I 120
e IOS
,
/ :, I/
~ 90 , 90
"' 7S
l...
7S
60 l...60
....
4S ....4S
30
1S

0
.. :..,

1234S6789x
30
IS

0
..... ""'
1234S6789x
Term Term
   
d. Sample answer: The graphs cover the same d. Sample answer: The graphs cover the same
range. The domain of the series is the natural range. The domain of the series is the natural
numbers, while the domain of the quadratic function numbers, while the domain of the quadratic function
is all real numbers, 0 ≤ x ≤ 10. is all real numbers, 0 ≤ x ≤ 10.
   
e . Sample answer: For every partial sum of an e . Sample answer: For every partial sum of an
arithmetic series, there is a corresponding quadratic arithmetic series, there is a corresponding quadratic
function that shares the same range. function that shares the same range.
   
x •
f. I:2k+1 f. I:2k+7
hi
4·1
 
ANSWER:  
a. Find the value of x.
 
I 4
2 10 72. 
3 18
4 28  
5 40
SOLUTION:  
6 54
7 70 There are x – 3 + 1 or x – 2 terms, so n = x – 2.
8 88  
9 108
10 130
 
b.  
) Find the sum.
13S
120
 

"'
..
E IOS
90
.,
.-;;; 7S
II 60
.... 4S
30
IS

0 1234S6789X
Term
   
c. Equate the sum with the given value and solve for
x.
eSolutions Manual - Powered by Cognero   Page 27
 
 
ANSWER:  
f. 18
10-2 Arithmetic Sequences and Series
 
 

Find the value of x. '


  73.  L<sk + 2> = 1032
hS

., '
72.  L<6k-5)=92s

 
 
SOLUTION:  
There are x – 5 + 1 or x – 4 terms, so n = x – 4.
SOLUTION:    
There are x – 3 + 1 or x – 2 terms, so n = x – 2. a, =8(5)+ 2 or 42
 
"•·• =8(x)+2 or8x+2
"• =6(3)-5 or 13  
" ' •, =6(,·) - or 6x - 5
• - ::> Find the sum.
   
Find the sum.
  Sn = ,{ "1 ~ a,, )
s., = (x-4)( 42 + ~x+ 2)
=(x-4)(4x+22)
=4x! +6x-88
 
Equate the sum with the given value and solve for
  x.
Equate the sum with the given value and solve for 4xi + 6x-88 = 1032
x.
 
4xi + 6x -1120 =0
 
3xl -2x-8=928  
3x2 -2x-936 = 0
 
x= -6± J61 --1(4)(-1120)
2(4)
.r -(-2)±J{-2)2 4(3)(-936) = -6± 134
2(3) 8
= 2± 106 =16or -17.5
6  
The value of x should be positive. Therefore, x =
=18or -17.3
16.
 
 
The value of x should be positive. Therefore, x =
18. ANSWER:  
  16
ANSWER:    
18 74. CCSS CRITIQUE  Eric and Juana are
  determining the formula for the nth term for the
sequence –11, –2, 7, 16, … . Is either of them
correct? Explain your reasoning.
73.   
 
SOLUTION:  
eSolutions Manual - Powered by Cognero Page 28
There are x – 5 + 1 or x – 4 terms, so n = x – 4.
 
 
ANSWER:  
ANSWER:   Sample answer: Eric; Juana missed the step of
16
10-2 Arithmetic Sequences and Series multiplying d by n – 1.
   

74. CCSS CRITIQUE  Eric and Juana are 75. REASONING If a is the third term in an


determining the formula for the nth term for the arithmetic sequence, b is the fifth term, and c is the
sequence –11, –2, 7, 16, … . Is either of them eleventh term, express c in terms of a and b.
correct? Explain your reasoning.  
 
SOLUTION:  
Evie
Given a 3 = a, a 5 = b and a 11 = c.
J..= ,,-10..'1,"'1=-I\  
"""= -11 + (" - l)'l Find the common difference.
 
= "" - 20 b =a+ 2d
d=b-a
}UMCA. 2
d = 16- 7 DI' 1, A.1 = -11  
Find the value of a 1.
«.. = s« - 11
 

  a =01 + 2( b;a)
SOLUTION:   a, = 2,, _,,
Sample answer: Eric; Juana missed the step of
multiplying d by n – 1.  
  Find the value of c in terms of a and b.
 
ANSWER:   a,, -a1 +(n-l)d
Sample answer: Eric; Juana missed the step of
multiplying d by n – 1. c=2a-b+(l 1-l)(h;tt)
 
~ .\b-3(1
75. REASONING If a is the third term in an  
arithmetic sequence, b is the fifth term, and c is the
ANSWER:  
eleventh term, express c in terms of a and b.
  4b – 3a
 
SOLUTION:  
Given a 3 = a, a 5 = b and a 11 = c. 76. CHALLENGE There are three arithmetic means
between a and b in an arithmetic sequence. The
  average of the arithmetic means is 16. What is the
Find the common difference. average of a and b?
   
SOLUTION:  
The three arithmetic means between a and b are
  .
Find the value of a 1.  
The average of the arithmetic means is
 
.
 
Therefore,
eSolutions Manual - Powered by Cognero Page 29
.
The term b can be written as a + 4d.
 
 
   
ANSWER:   ANSWER:  
4b – 3a
10-2 Arithmetic Sequences and Series 16
   

76. CHALLENGE There are three arithmetic means 77. CHALLENGE Find S n for (x + y) + (x + 2y) + (x
between a and b in an arithmetic sequence. The + 3y) + … .
average of the arithmetic means is 16. What is the  
average of a and b?
  SOLUTION:  
(x + 2y) – (x + y) = y
SOLUTION:  
(x + 3y) – (x + 2y) = y
The three arithmetic means between a and b are  
a+ d . a + 2,/. a + 3d.
The common difference is y.
   
The average of the arithmetic means is "• = (x + y)
3a+6d = 16
.  
3
Find the sum.
   
Therefore, Cl+2cl= 16
.
s. = ~(2a1 + (11 I )d)
The term b can be written as a + 4d.
  -~(2(x+y) (11-l)y)
2« + 4d _ + 'cl
The average of a and b is ----ll
2
-.
=!!.(2r+2n 11y-y)
2 •
  11' I' IIJ
We know that a..-2d 16. = /IX + II)' + .,-'-- - ?
- -
(, )
 
Therefore, the average of a and b is 16. 11· + II
 
s. =nx-t )' ~

ANSWER:    
16
ANSWER:  

( )
  11· + II
s.=11x+y ~'
77. CHALLENGE Find S n for (x + y) + (x + 2y) + (x
+ 3y) + … .  
 
SOLUTION:   78. OPEN ENDED Write an arithmetic series with 8
terms and a sum of 324.
(x + 2y) – (x + y) = y
 
(x + 3y) – (x + 2y) = y
  SOLUTION:  
The common difference is y. Sample answer: 9 + 18 + 27 + … + 72
   
ANSWER:  
 
Find the sum. Sample answer: 9 + 18 + 27 + … + 72
   

79. WRITING IN MATH Compare and contrast


arithmetic sequences and series.
 
SOLUTION:  
Sample answer: An arithmetic sequence is a list of
eSolutions Manual - Powered by Cognero Page 30
terms such that any pair of successive terms has a
common difference. An arithmetic series is the sum
of the terms of an arithmetic sequence.
  Sample answer: An arithmetic sequence is a list of
terms such that any pair of successive terms has a
ANSWER:   common difference. An arithmetic series is the sum
Sample answer:
10-2 Arithmetic 9 + 18 +and
Sequences … + 72
27 +Series of the terms of an arithmetic sequence.
   

79. WRITING IN MATH Compare and contrast 80. PROOF Prove the formula for the nth term of an
arithmetic sequences and series. arithmetic sequence.
   
SOLUTION:   SOLUTION:  
Sample answer: An arithmetic sequence is a list of Sample answer:
terms such that any pair of successive terms has a Let a n = the nth term of the sequence and d = the
common difference. An arithmetic series is the sum
common difference
of the terms of an arithmetic sequence.
a 2 = a 1 + d Definition of the second term of an
 
arithmetic sequence
ANSWER:   a 3 = a 2 + d Definition of the third term of an
Sample answer: An arithmetic sequence is a list of arithmetic sequence
terms such that any pair of successive terms has a a 3 = (a 1 + d ) + d Substitution
common difference. An arithmetic series is the sum
of the terms of an arithmetic sequence. a 3 = a 1 + 2d Associative Property of Addition
  a 3 = a 1 + (3 – 1)d      3 – 1 = 2
80. PROOF Prove the formula for the nth term of an a n = a 1 + (n – 1)d      n = 3
arithmetic sequence.  
 
ANSWER:  
SOLUTION:   Sample answer:
Sample answer: Let a n = the nth term of the sequence and d = the
Let a n = the nth term of the sequence and d = the
common difference
common difference a 2 = a 1 + d Definition of the second term of an
a 2 = a 1 + d Definition of the second term of an
arithmetic sequence
arithmetic sequence a 3 = a 2 + d Definition of the third term of an
a 3 = a 2 + d Definition of the third term of an
arithmetic sequence
arithmetic sequence a 3 = (a 1 + d ) + d Substitution
a 3 = (a 1 + d ) + d Substitution
a 3 = a 1 + 2d Associative Property of Addition
a 3 = a 1 + 2d Associative Property of Addition
a 3 = a 1 + (3 – 1)d      3 – 1 = 2
a 3 = a 1 + (3 – 1)d      3 – 1 = 2
a n = a 1 + (n – 1)d      n = 3
a n = a 1 + (n – 1)d      n = 3
 
 
81. PROOF Derive a sum formula that does not
ANSWER:   include a 1.
Sample answer:
 
Let a n = the nth term of the sequence and d = the
common difference SOLUTION:  
a 2 = a 1 + d Definition of the second term of an
General sum formula
arithmetic sequence
a 3 = a 2 + d Definition of the third term of an  
arithmetic sequence a n = a 1 + (n – 1)d   Formula for nth term
a 3 = (a 1 + d ) + d Substitution  
a 3 = a 1 + 2d Associative Property of Addition a n – (n – 1)d = a 1    Subtract (n – 1)d from both
a 3 = a 1 + (3 – 1)d      3 – 1 = 2 sides.
eSolutions
 
a Manual
n= a +- (n
1
Powered by Cognero
– 1)d      n =3 Page 31

   Substitution
a 3 = a 1 + 2d Associative Property of Addition
 
a 3 = a 1 + (3 – 1)d      3 – 1 = 2
 Simplify.
10-2 Arithmetic – 1)d      n =and
a n = a 1 + (n Sequences 3 Series
   

81. PROOF Derive a sum formula that does not 82. PROOF Derive the Alternate Sum Formula using
include a 1. the General Sum Formula.
 
 
SOLUTION:  
SOLUTION:  
General sum formula
General sum formula
 
 
a n = a 1 + (n – 1)d   Formula for nth term
a n = a 1 + (n – 1)d   Formula for nth term
 
 
a n – (n – 1)d = a 1    Subtract (n – 1)d from both S,, =[a1 +a1 +(11-l)dl{f)  Substitution
sides.
   

S,. -[a,, -(11- l)d + a.,) { f)  Substitution S,, -[2a +(11-l)dJ{f)


1  Simplify.

   

S., -[2c,,, -(n-l)dl{f)  Simplify. ANSWER:  

  S,, =(a1 +a.>{f) General sum formula

ANSWER:    
a n = a 1 + (n – 1)d   Formula for nth term
General sum formula

 
S,, =[a1 +(11-l)d]{;)  Substitution
~ a1

S,. =[2a +(11-l)dl{f)  Simplify.


a n = a 1 + (n – 1)d   Formula for nth term
1
 
a n – (n – 1)d = a 1    Subtract (n – 1)d from both
83. SAT/ACT  The measures of the angles of a
sides triangle form an arithmetic sequence. If the
. measure of the smallest angle is 36°, what is the 
S., =I«, -(11- l)d + a.,)-( f)  Substitution  
measure of the largest angle?

 
S., = [2c,,, -(n- l)d)-( f)  Simplify.

   
A 54°
82. PROOF Derive the Alternate Sum Formula using
 
the General Sum Formula.
B 75°
 
 
SOLUTION:   C 84°
 
General sum formula D 90°
   
a n = a 1 + (n – 1)d   Formula for nth term E 97°
 
 
eSolutions Manual - Powered by Cognero Page 32
SOLUTION:  
 Substitution The sum of the interior angle of a triangle is 180°.
Since the measures of the angles of a triangle form
Option C is the correct answer.
 Substitution
ANSWER:  
10-2 Arithmetic Sequences and Series
 Simplify. C
 

83. SAT/ACT  The measures of the angles of a I ,


triangle form an arithmetic sequence. If the 84. The area of a triangle is -q· -8  and the height is q
2
measure of the smallest angle is 36°, what is the  + 4. Which expression best describes the triangle’s
measure of the largest angle? length?
   
F  (q + 1)
 
G (q + 2)
   
A 54° H (q – 3)
   
B 75° J   (q – 4)
   
C 84° SOLUTION:  
 
D 90° The area of the triangle is .
   
E 97°
  ~(</-16)
The length of the triangle is -=--,---q-4
SOLUTION:   2(q+4)
The sum of the interior angle of a triangle is 180°.
Since the measures of the angles of a triangle form  
an arithmetic sequence and the smallest angle is Option J is the correct answer.
36°, the other two angles are 36° + d and 36° + 2d.  
36 + 36 + d + 36 + 2d = 180
ANSWER:  
d = 24
J
Therefore, the largest angle is 36° + 48° = 84°.
 
 
Option C is the correct answer.
85. The expression  is equivalent to
ANSWER:    
C
  A

 
84. The area of a triangle is  and the height is q
B
+ 4. Which expression best describes the triangle’s
length?  
 
F  (q + 1) C
 
G (q + 2)  
  D
H (q – 3)
   
J   (q – 4)
SOLUTION:  
 
SOLUTION:  
eSolutions Manual - Powered by Cognero Page 33

The area of the triangle is .  


Option A is the correct answer.
  Option A is the correct answer.
 
ANSWER:  
J
10-2 Arithmetic Sequences and Series ANSWER:  
  A

85. The expression  is equivalent to 86. SHORT RESPONSE Trevor can type a 200-word


  essay in 6 hours. Minya can type the same essay in
4..!.
2 hours. If they work together, how many hours
A
will it take them to type the essay?
   
B SOLUTION:  
200
  Trevor can type 6  words in an hour.

C  
400
Minya can type 9  words in an hour.
 
 
D 200 400 700
-+-
..-
They type 6 9 9 words together in an
 
hour.
SOLUTION:    
\ I 200 _ 200(9)
-r-»-:» I I
(7~01- 700
 
Option A is the correct answer. --187
 
ANSWER:  
A  
86. SHORT RESPONSE Trevor can type a 200-word They will type the essay in  hours if they work 
essay in 6 hours. Minya can type the same essay in
together.
hours. If they work together, how many hours  
will it take them to type the essay? ANSWER:  
 
SOLUTION:  
 
Trevor can type  words in an hour.
  Determine whether each sequence is
arithmetic. Write yes or no.
Minya can type  words in an hour.  
87. –6, 4, 14, 24, …
   
They type words together in an SOLUTION:  
hour. Since there is a common difference between the
  consecutive terms, this is an arithmetic sequence.
 
ANSWER:  
yes
eSolutions Manual - Powered by Cognero   Page 34
 
ANSWER:  
ANSWER:  
10-2 Arithmetic Sequences and Series no
   

Determine whether each sequence is Solve each system of inequalities by graphing.


arithmetic. Write yes or no.  
  x+2y> I
87. –6, 4, 14, 24, … 90. 
x1 + y1 S 25
 
 
SOLUTION:  
SOLUTION:  
Since there is a common difference between the y
consecutive terms, this is an arithmetic sequence. ~ ~
  - ,
"~
ANSWER:   I
J ' .. .. ..
yes 0 .. . I
 
' I .-,

7 4 I " .. ~
1..1

88.  2~-.-.-, ...


5 5 5
 
 
SOLUTION:   ANSWER:  
y
Since there is a common difference between the l,o ....
consecutive terms, this is an arithmetic sequence. / r,;:
  ·,'
.. , .
ANSWER:  
0 .. I
yes
  ' ...
89. 10, 8, 5, 1, ... "' .... ., /

   
SOLUTION:  
Since there is no common difference between the
consecutive terms, this is not an arithmetic 91. 
sequence.
   
SOLUTION:  
ANSWER:  
no
 

Solve each system of inequalities by graphing.


 
90. 

 
SOLUTION:    
ANSWER:  

eSolutions Manual - Powered by Cognero Page 35


10-2 Arithmetic Sequences and Series
 
 
x+ y:S;2 ./ + / .::4
91.  92. 
4x2 - J'2 .:: 4 4/ ·9x1 :S;36
   

\ .
SOLUTION:  


y

SOLUTION:  
y

" .... I
/,;
..
N
\ J
1
'' '\. I
0 IC
..... ~
,
i....
0 ..... IC "
, \
~ ' 11

   

ANSWER:   ANSWER:  
• ' y ,. y

'' J
' ~
(,; ....
\ J
'
' I'\. I
0 IC

......
\'-
,
i.,i

0 '- IC "
, '.. '-
'lll
 
 
93. PHYSICS The distance a spring stretches is
related to the mass attached to the spring. This is
92. 
represented by d = km, where d is the distance, m is
the mass, and k is the spring constant. When two
 
springs with spring constants k 1 and k 2 are attached
SOLUTION:   in a series, the resulting spring constant k is found
I I I
by the equation -=-+-.
k k, *i
 
a. If one spring with constant of 12 centimeters per
gram is attached in a series with another spring with
constant of 8 centimeters per gram, find the
resultant spring constant.
 
  b. If a 5-gram object is hung from the series of
springs, how far will the springs stretch? Is this
ANSWER:   answer reasonable in this context?
 

eSolutions Manual - Powered by Cognero Page 36


resultant spring constant. would stretch the first spring 60 cm and would
  stretch the second spring 40 cm. The object would
b. If a 5-gram object is hung from the series of have to stretch the combined springs less than it
springs, howSequences
10-2 Arithmetic far will the springs stretch? Is this
and Series would stretch either of the springs individually.
answer reasonable in this context?  
 
Graph each function. State the domain and
range.
Spmg1 { ~
k, = 12 cmto ;:;:  
;:; SOflno 1

-- I
Spring 2 { ~ 94.  f(x) = ~(2')
ki = 8cmto ~ .)

::::  
d :::: SOf,no 2
...._ SOLUTION:  
_I :::: I Graph the function.
so  
8
,, ~

  6 ~
41-
~ l. I
2t:;, ~ /(.-) = 3 (2')
SOLUTION:  
a. Given k 1 = 12 and k 2 = 8. - -6-4-20
.
2 • 6 8.r
  !..4
Substitute the values and evaluate the value of k. 6
1 I I ,8
-=-+-  
k k, k!
1 I The function is defined for all values of x.
=-+-  
12 8
Therefore, the domain is D = {all real numbers}.
5  
-=-
k 24 The value of the f (x) tends to 0 as x tends to –∞.
k =4.8 crn/g  
  The value of the f (x) tends to ∞ as x tends to ∞.
b. Substitute 4.8 and 5 for k and m respectively in  
the equation d = km. Therefore, the range of the function is R ={f (x) | f
  (x) > 0}.
d=5(4.8)  
d = 24 Clll ANSWER:  
  roo
The answer is reasonable. The object would stretch 8
the first spring 60 cm and would stretch the second 6
4<-7
.
spring 40 cm. The object would have to stretch the
combined springs less than it would stretch either of 217- 11..i = f<2'l I
the springs individually. - -6-4-20 2 • 6 8.r
  !.4
6
ANSWER:   -8

a. 4.8 cm/g  
  D = {all real numbers}, R = {f (x) | f (x) > 0}
b. 24 cm; The answer is reasonable. The object  
would stretch the first spring 60 cm and would
stretch the second spring 40 cm. The object would
have to stretch the combined springs less than it 95. 
would stretch either of the springs individually.  
 
SOLUTION:  
Graph each function. State the domain and Graph the function.
eSolutions Manual - Powered by Cognero Page 37
range.  
 
 
 
D = {all real numbers}, R = {f (x) | f (x) > 0}
10-2 Arithmetic Sequences and Series D = {all real numbers}, R = {f (x) | f (x) > 3}
 
 

95.  f(x) ~ 4·' + 3


96. 
 
SOLUTION:    
Graph the function. SOLUTION:  
  Graph the function.
f(XJ  
' f(.w)

I ((.-) = 4x + 31
!.,'

I((.-) .. 21tr- , , /(

0
0 /(

 
The function is defined for all values of x.  
  The function is defined for all values of x.
Therefore, the domain is D = {all real numbers}.  
  Therefore, the domain is D = {all real numbers}.
The value of the f (x) tends to 3 as x tends to –∞.  
  The value of the f (x) tends to ∞ as x tends to –∞.
The value of the f (x) tends to ∞ as x tends to ∞.  
  The value of the f (x) tends to –1 as x tends to ∞.
Therefore, the range of the function is R ={f (x) | f  
(x) > 3}. Therefore, the range of the function is R ={f (x) | f
  (x) > –1}.
 
ANSWER:  
ANSWER:  
f(.w)
' J f(.w)
~
J f(.-) - 4x + 31
!.,'

I'(.-)= 2ltY- 1
x
0
0 /(

 
D = {all real numbers}, R = {f (x) | f (x) > 3}  
  D = {all real numbers}, R = {f (x) | f (x) > –1}

Solve each equation. Round to the nearest ten-


96.  thousandth.
 
  x
97. 5 = 52
SOLUTION:    
Graph the function.
SOLUTION:  
 

eSolutions Manual - Powered by Cognero Page 38


 
ANSWER:  
 
10-2 Arithmetic Sequences and Series 0.5537
D = {all real numbers}, R = {f (x) | f (x) > –1}  

Solve each equation. Round to the nearest ten- n+2


99. 3 = 14.5
thousandth.  
 
x
97. 5 = 52 SOLUTION:  
3• 1 = 14.S
 
11 + 2 - log, 14.S
SOLUTION:  
5' =52 ,,_ log 14.S
log3
--.,
x log,52 ""0.-13-11
log52
=-  
log5
~2.-isso ANSWER:  
  0.4341
 
ANSWER:  
2.4550 d –4 3–d
100. 16 =3
   
SOLUTION:  
3p 16J I: 3J-J
98. 4 = 10
  16'' 3'
SOLUTION:   16' =3.,
4•,· .. 10 3'116'1 = 3) X 16 I

3p = log, 10 48'' = 1769-172


loglO log48" = log 1769-172
JJ- 3log4
c/log48 = log I 769472
0.5537
d = log 1769-172
11:

  log-18
ANSWER:   c/;:; 3.7162
0.5537  
 
ANSWER:  
n+2 3.7162
99. 3 = 14.5
   
SOLUTION:  

 
ANSWER:  
0.4341
 
eSolutions Manual - Powered by Cognero Page 39
d –4 3–d
100. 16 =3
 
 
ANSWER:  
10-3 Geometric Sequences and Series 2046
 

1. CCSS REGULARITY  Dean is making a family Write an equation for the nth term of each
tree for his grandfather. He was able to trace many geometric sequence.
generations. If Dean could trace his family back 10  
generations, starting with his parents how many 2. 2, 4, 8, …
ancestors would there be?  
  SOLUTION:  
SOLUTION:   r ="
+ 2 or 2
a1 - 2
Let a 1 = 2,  r = 2, and n = 10.
   
  a,, -a1r n-1
a1 -a/1 = "),
Sn=--.__
l-r
 
(I
" -- -,n-1

2 -2(210)
- 1-2 ANSWER:  
2 -2(1024) a,, 2·2°' I
-1  
2 -2048
- -1
3. 18, 6, 2, …
-2046
-1  
=2046 SOLUTION:  
  I
r = 6+ 18 or -
  3
Dean will identify 2046 ancestors.. (/1 -18
   
aII - aI ,·n-1
ANSWER:  
2046
 
a" ( I)""'
18· -..,
.>

Write an equation for the nth term of each  


geometric sequence. ANSWER:  
 
2. 2, 4, 8, …
 
«; =18· ( 3I)""'
 
SOLUTION:  
4. –4, 16, –64, …
 
  SOLUTION:  

   
ANSWER:  

   
eSolutions Manual - Powered by Cognero Page 1
ANSWER:  
3. 18, 6, 2, …
 
ANSWER:   ANSWER:  

10-3 Geometric Sequences and Series


   
4. –4, 16, –64, … I 3
  6.  ""' = -.r
8 4
SOLUTION:    
r - 16 + ( 4) or ( -4)
 
SOLUTION:  
"• -4
  Find a 1.
6-1
a; - c,,, ·" I a6 =air
Q6
a1=-
 
,.s
1
ANSWER:   8
a1= (l)5
 
-
1024
5. a 2 = 4, r = 3 1944
  = 128
243
SOLUTION:  
 
Find a 1. Write the equation.
  a,, -- ·" I a,,
ti = 128 (~)" I
a, IJ 243 4
"• --
,.  
4
a, -- ANSWER:  
3
  (I = 128 ( 3 )"-'
Write the equation. " 243 4
   
a,, - a,, ,n I

7. a 2 = –96, r = –8
«.>':4 ( 3 )"
.)
I
 
 
SOLUTION:  
ANSWER:   Find a 1.
ti
,, = .:!_(3)"
"
.)
I

6. 
 
   
Write the equation.
SOLUTION:  
Find a 1.

eSolutions Manual - Powered by Cognero   Page 2

ANSWER:  
ANSWER:  
ANSWER:  
1, 4, 16 or –1, 4, –16]
10-3 Geometric Sequences and Series  
 

7. a 2 = –96, r = –8 9.  0.20. ? . ? • ? .125


   
SOLUTION:   SOLUTION:  
Find a 1. Given a 1 = 0.20 and a 5 = 125.
a2 = a1r-'-I  
a,, = a1r
11-I
a,
u, =--,. 125 = 0.20r5-1
-96 /'t -625
a,= -8
r= ±5
=12  
  The geometric means are 1, 5, 25 or –1,5,–25.
Write the equation.  
a,,= a1r n-1
a,, -12· ( -8r ANSWER:  
1, 5, 25 or –1, 5, –25
   
ANSWER:  
10. GAMES Miranda arranges some rows of dominoes
'1 12( -8)"-I
11 - so that after she knocks over the first one, each
  domino knocks over two more dominoes when it
falls. If there are ten rows, how many dominoes does
Find the geometric means of each sequence. Miranda use?
   
8.  0.25. ? . ? . ? . 64  
  SOLUTION:  
SOLUTION:    
Here a 1 = 0.25 and a 5 = 64. ANSWER:  
  1023
a; = a1r It-I  
64 =0.25rH Find the sum of each geometric series.
r1 = 256  
r=t4
11. 
 
The geometric means are 1, 4, 16 or –1, 4, –16.  
 
SOLUTION:  
ANSWER:   Given r = 4.
1, 4, 16 or –1, 4, –16]  
  There are 6 – 1 + 1 or 6 terms, so n = 6.
 
1–1
9.  a 1 = 3(4) =3
   
Find the sum.
SOLUTION:    
Given a 1 = 0.20 and a 5 = 125.
eSolutions Manual - Powered by Cognero Page 3
 
 
ANSWER:  
ANSWER:  
4095
10-31023
Geometric Sequences and Series  
 

Find the sum of each geometric series.


  12.  t 4(.!.)t-1
' I 2
11.   

  SOLUTION:  
Given r = 4.
SOLUTION:    
Given r = 4. There are 8 – 1 + 1 or 8 terms, so n = 8.
   
There are 6 – 1 + 1 or 6 terms, so n = 6.
 
1–1
a 1 = 3(4) =3  
  Find the sum.
Find the sum.  
  a1 -a1r n
(JI - Cl1I ·" S,, = I -r
=
( I)~
S,, 1-r

3-3(4}6 4-4
2
S<, = 1-4 Ss = I
1-
=4095 2
  = 7.96875
i:3(4)t-l = 4095  
A I
  t,~1 4(~)
-
k-1

= 7.96875
ANSWER:    
4095 ANSWER:  
 
7.96875
 

12.  Find a 1 for each geometric series described.

   

SOLUTION:   13. 
Given r = 4.  
 
There are 8 – 1 + 1 or 8 terms, so n = 8. SOLUTION:  
 

 
Find the sum.  
 
Substitute the corresponding values and solve for a 1.
 

eSolutions Manual - Powered by Cognero Page 4


  ANSWER:  

ANSWER:  
10-37.96875
Geometric Sequences and Series  
 

Find a 1 for each geometric series described.


14.  S,, -91 12,r-3.n-1
I
 
 
13.  s n =85 16,r=4,n=6
5
SOLUTION:  
 
S -
.>,, -
a, - aI r"
SOLUTION:   1-r
,S',,- - a, - ti I r" = a1 ( I - r")
1-r 1-r
= "1 ( 1-r")  
1-r
Substitute the corresponding values and solve for a 1.
   
Substitute the corresponding values and solve for a 1. 91 _!_ - a, ( I - 37 )
  2 1-3

852_= (1-4") 01 1~3=a,(1 2187)


16 1-4 2
1365 = a,(1 4096) 01
= 183. -2
16 -3 2 -2186
a,=1365_ -3
16 -4096 12
 
16 ANSWER:  
 
ANSWER:   12
I  
16
  15. 
 
14.  SOLUTION:  
 
SOLUTION:    
Substitute the corresponding values and solve for a 1.
 

 
Substitute the corresponding values and solve for a 1.
 

 
eSolutions Manual - Powered by Cognero Page 5
ANSWER:  
512
 
ANSWER:  
ANSWER:  
10-3 Geometric Sequences and Series 512
   

I I I I
15.  Sn= I 020.CI,, = 4.r =- 16.  Sn -Iii- a -- r--
- 3' II 3' 3
2
   
SOLUTION:   SOLUTION:  
c- __ a1 - a,,r S = a1 -a,,r
~),, -
I <r " I r
   
Substitute the corresponding values and solve for a 1. Substitute the corresponding values and solve for a 1.
   

1020 =
a1-4(1)
2 121.!.=
(I

I
- '(I)
J J
t- 1 3 I
1-
2 3
I
o1 - 2 = I 020 · -
2
(II =510+ 2
=512 3
  I 364 2
a--=---
1 9 3 3
ANSWER:   728 I
512 a,=-+-
9 9
  729
=-
9
16.  =81
   

SOLUTION:   ANSWER:  
81
 
 
17. WEATHER Heavy rain in Brieanne’s town caused
Substitute the corresponding values and solve for a 1. the river to rise. The river rose three inches the first
  day, and each day after rose twice as much as the
previous day. How much did the river rise in five
days?
 
SOLUTION:  
Let a 1 = 3, r = 2, and n =5.
 

eSolutions Manual - Powered by Cognero Page 6

 
   
ANSWER:   ANSWER:  
10-381
Geometric Sequences and Series 93 in.
   

17. WEATHER Heavy rain in Brieanne’s town caused Find a n for each geometric sequence.
the river to rise. The river rose three inches the first  
day, and each day after rose twice as much as the u,; 2400,r ;-.11-
I
7
previous day. How much did the river rise in five 18.  4
days?  
 
SOLUTION:  
SOLUTION:  
The nth term of a geometric sequence is .
Let a 1 = 3, r = 2, and n =5.
 
  II ' I
S n= a1 -air a,, = 2400( ~)
1-r

3-3(25) 2400
::::::--
1-2 4096
3-3(32) -0.5859375
-1
 
3-96
=--=r- ANSWER:  
-93 75
= ----=T 128or 0.5859375
=93  
 
I
  a1 - 800.r aaa -.11- 6
19.  2
The river rose 93 inches in 5 days.
   

ANSWER:   SOLUTION:  
93 in. The nth term of a geometric sequence is u - aI r". '
II

   
1
«. -800 ( 2 I)<>
Find a n for each geometric sequence.
  800
18.  32
-25
 
 
SOLUTION:  
ANSWER:  
The nth term of a geometric sequence is .
25
   

20. 
 
SOLUTION:  
 
The nth term of a geometric sequence is .
ANSWER:    

or 0.5859375
eSolutions Manual - Powered by Cognero Page 7

 
   

ANSWER:   ANSWER:  
10-325
Geometric Sequences and Series 512
   

22. BIOLOGY A certain bacteria grows at a rate of 3


20.  cells every 2 minutes. If there were 260 cells initially,
  how many are there after 21 minutes?
 
SOLUTION:  
SOLUTION:  
The nth term of a geometric sequence is .
3
  Given, a 1 = 260, n = 21 and r = .
2
2 ( )'-1  
(I ,, -- 9 3
1458
a,, = arn
1
I

=-9- J)ZI I
«: -260 ( 2
=162
=864567
 
ANSWER:  
162 ANSWER:  
  864,567
 
21.  01 = -4.r = -2.11 - 8
  Write an equation for the nth term of each
geometric sequence.
SOLUTION:    
The nth term of a geometric sequence is . 23. –3, 6, –12, …
   
a,,= (-4)(-2)8 I SOLUTION:  
=512 r=6+(-3) or -2
  "1 = -3
ANSWER:    
,,_,
512 a" =a,r
  (/11 = (-J)(-2)"-I

22. BIOLOGY A certain bacteria grows at a rate of 3  


cells every 2 minutes. If there were 260 cells initially,
ANSWER:  
how many are there after 21 minutes?
n –1
  a n = (–3)(–2)
SOLUTION:    

Given, a 1 = 260, n = 21 and r = . 24. 288, –96, 32, …


   
SOLUTION:  

   

ANSWER:  
864,567
eSolutions Manual - Powered by Cognero Page 8
 
 
Write an equation for the nth term of each
ANSWER:  
n –1
ANSWER:  
a n = (–3)(–2) n –1
10-3  Geometric Sequences and Series a n = (–1)(–1)
 
24. 288, –96, 32, … 1 2 4
26. 
  3'9'27'' ..
SOLUTION:    
I SOLUTION:  
r-(-96)+288or -3
"I I 2
-
r=-+-;:: or -3
a1 - 288 9 .)
 
,,_, a=-
, 3
0 = a,r
 
a 11 - 288(-~),r-l
II .)

 
ANSWER:  
 
ANSWER:  
 

25. –1, 1, –1, …
 
 
SOLUTION:   I
27.  8 . 2 . -2 ....
,. = 1 + (-1) or - I
 
"1 =-I
  SOLUTION:  

,. = 2 +8 or 4I
,r-1
a,,= a,r
,,_,
all =(-t)(-1)
"• - 8
   
n-1
ANSWER:   a" =a,r ),,_,
n –1 I
a n = (–1)(–1)
 
«. -8
- ( -4
 
26.  ANSWER:  
 
SOLUTION:  
 

28. 
 
 
SOLUTION:  

eSolutions
  Manual - Powered by Cognero Page 9
 
ANSWER:  
ANSWER:  
ANSWER:  

10-3 Geometric Sequences and Series


   

64 30. a 4 = –8, r = 0.5


28.  12.-16.~····
.)  
 
SOLUTION:  
SOLUTION:   Find a 1.
4
,. _ (-16) .;-12 or --::-  
.)
a~ = a1r ~ I
a,= 12
  (/~
a, =l r:

a=-
-8
Cl =12(-!)"-I • o.s'
n 3
=-64
   
Write the equation.
ANSWER:    
,,_,
a = 12·
fl ( _:!_3 ) «: = ( -64) ( 0.5)
11-I

 
 
29. a 3 = 28, r = 2
ANSWER:  
  n –1
a n = –64(0.5)
SOLUTION:    
Find a 1.
  31. a 6 = 0.5, r = 6
 
a, SOLUTION:  
a,=--,,.-
Find a 1.
28  
a, = 22
=7
 
Write the equation.
 

,,_,
a; =7 ( 2)
   
Write the equation.
ANSWER:  
 

30. a 4 = –8, r = 0.5


 
 
ANSWER:  
SOLUTION:  
eSolutions Manual - Powered by Cognero Page 10
Find a 1.
   
ANSWER:  
ANSWER:  
n –1
a n = –64(0.5)
10-3 Geometric Sequences and Series
   

31. a 6 = 0.5, r = 6 I
32.  "' = 8.r = 2
 
 
SOLUTION:  
Find a 1. SOLUTION:  
Find a 1.
 
a~ - a1r
6· I  
)-1
a., =a1r
lit,
ll1 =\ a,
r a, = --:;-
,..
0.5
ll1 =7
I
= 15552
  -32
Write the equation.  
  Write the equation.
a,, :;;:; a1r 11-I  
,,_,
o,, = a,r
--'-(6)"'
15552
"11

 
-

a -"
"
32(!)"-I
2
ANSWER:    
_
a,, - 1-). 5--,
I
,_ (G)"-1 ANSWER:  

  a =
"
32( 21 )"-i
 
32. 
  33. 
SOLUTION:    
Find a 1.
SOLUTION:  
 
Find a 1.
 

 
Write the equation.
   
Write the equation
.

eSolutions Manual - Powered by Cognero Page 11


 
ANSWER:   ANSWER:  

10-3 Geometric Sequences and Series


   

I 34. a 4 = 80, r = 4
33.  a4 - 24,r _ -
3  
 
SOLUTION:  
SOLUTION:   Find a 1.
Find a 1.
 
  a4 = a1r4-1
04 = 01,.4-1

a,=-
04
a,=1
r
""
r' 80
24 a,=-

a,= (~J 4l
_
,,, 5
4
=648  
  Write the equation.
Write the equation  
.
a = «r:'
I
II

n-1
a
,,
=~(4)"·
4
I

«; -648( f )  
  ANSWER:  
ANSWER:   Cl : ~(4)" I
I),,_, " 4
a,,= 648(  
3
  Find the geometric means of each sequence.
 
34. a 4 = 80, r = 4 35. 
   
SOLUTION:   SOLUTION:  
Find a 1. Since there are three terms between the first and last
  term, there are 3 + 2 or 5 total terms, so n = 5.
Given a 1 = 810 and a 5 = 10.
 

 
Write the equation.  
  The geometric means are 270, 90, 30 or –270, 90, –
30.
 
eSolutions Manual - Powered by Cognero Page 12
ANSWER:  
  270, 90, 30 or –270, 90, –30
 
ANSWER:  
ANSWER:  
270, 90, 30 or –270, 90, –30
10-3 Geometric Sequences and Series  
 

Find the geometric means of each sequence. 36.  640. 2._· 2._. ? .2.S
   
35.  810, 2._. 2_. ? . IO
SOLUTION:  
 
Since there are three terms between the first and last
SOLUTION:   term, there are 3 + 2 or 5 total terms, so n = 5.
Since there are three terms between the first and last Given a 1 = 640 and a 5 = 2.5.
term, there are 3 + 2 or 5 total terms, so n = 5.  
Given a 1 = 810 and a 5 = 10.
a,, - a,,. ..-•
 
2.5 = 640rS-I

10-810r5-I ,.4 =-1-


256
,.4 _ _!_
81 r- ±.!.4
r-±.!.  
3 The geometric means are 160, 40, 10 or –160, 40, –
  10.
The geometric means are 270, 90, 30 or –270, 90, –  
30.
  ANSWER:  
160, 40, 10 or –160, 40, –10
ANSWER:    
270, 90, 30 or –270, 90, –30
 
37. 

36.   
  SOLUTION:  
Since there are three terms between the first and last
SOLUTION:   term, there are 3 + 2 or 5 total terms, so n = 5.
Since there are three terms between the first and last  
term, there are 3 + 2 or 5 total terms, so n = 5.
Given a 1 = 640 and a 5 = 2.5. Given a 1 =  and a 5 = .
   

 
The geometric means are 160, 40, 10 or –160, 40, –  
10.
  The geometric means are  or 

ANSWER:   .
160, 40, 10 or –160, 40, –10
   
eSolutions Manual - Powered by Cognero Page 13
ANSWER:  
37. 
or
 
ANSWER:  
ANSWER:  
or
10-3160, –160, 40, –10
40, 10 orSequences
Geometric and Series
   

56 729 324
37.  !__ ? ? ? ? ,, ?
2·-· ._.·_· 81 0 38.  64 ._ .• _.. _.. 9
   
SOLUTION:   SOLUTION:  
Since there are three terms between the first and last Since there are three terms between the first and last
term, there are 3 + 2 or 5 total terms, so n = 5. term, there are 3 + 2 or 5 total terms, so n = 5.
   
Given a 1 =
7
 and a 5 =
56
. Given a 1 = -729  and a5 = -324 .
2 81 64 9
   
«; ;;;;a1r"-1
,r-1
a; ;;;;a1r

56 7 ~-1 324 729 .5-1


-=-r -;;;;-,
81 2 9 64
~ 16 J 256
r=- r ;;;;-

81 81
., 4
r = ±.:. .. r ;;;;±- ..
.) .)

   
The geometric means are
7 14 28
- - -  or  The geometric means are 243. !!.. 27  or 
3 9 27 16 4
7 14 28
-----
3 9 27
. _ 243 -~
16 4
-27 .

   
ANSWER:   ANSWER:  
7 14 28
or
7 14 28 243_!.!._27 or _ 243 .!.!..-21
3. 9 27 27 16 4 16 4
   

39. Find two geometric means between 3 and 375.


38.   
  SOLUTION:  
SOLUTION:   Since there are two terms between the first and last
Since there are three terms between the first and last term, there are 2 + 2 or 4 total terms, so n = 4.
term, there are 3 + 2 or 5 total terms, so n = 5.  
  Given a 1 = 3 and a 4 = 375.
 
Given a 1 =  and a 5 = .
 

 
The geometric means are .
 

eSolutions Manual - Powered by Cognero


ANSWER:   Page 14
  15, 75
 
The geometric means are  or 
The geometric means are .
ANSWER:  
ANSWER:  
or
10-3 Geometric Sequences and Series –8, 4
   

39. Find two geometric means between 3 and 375. 41. CCSS PERSEVERANCE  A certain water
  filtration system can remove 70% of the
contaminants each time a sample of water is passed
SOLUTION:   through it. If the same water is passed through the
Since there are two terms between the first and last system four times, what percent of the original
term, there are 2 + 2 or 4 total terms, so n = 4. contaminants will be removed from the water
  sample?
Given a 1 = 3 and a 4 = 375.  
  SOLUTION:  
,,_,
a,, = "1,. Let 100% contaminated water passes for the
375 = 3,.~-I purification.
That is, a 0 = 1.
,..\ -125
After the filtration, the system removes 70% of the
r=S contaminant.
  That is 30% of contamination still in the water.
The geometric means are 15. 75 . Therefore, r = 100% – 70% = 30% or 0.3.
  The number of times (n) is 4.
 
ANSWER:  
15, 75
  a, =.3(0.3t-•
40. Find two geometric means between 16 and –2. "'.3(0.3f
  =0.081
 
SOLUTION:   Therefore, after four times of filtration, 1 – 0.081 =
Since there are two terms between the first and last 0.9919 or 99.19% of the original contaminants will be
term, there are 2 + 2 or 4 total terms, so n = 4. removed.
   
Given a 1 = 16 and a 4 = –2.
ANSWER:  
 
u-1
99.19%
a,, - a1r  
-2 = I 6r1-1
Find the sum of each geometric series.
l I
,. =--  
8
,. _I 42. 
2
  SOLUTION:  
The geometric means are -8.4 . Find the sum.

ANSWER:  
–8, 4
 

41. CCSS PERSEVERANCE  A certain water


filtration system can remove 70% of the
contaminants each time a sample of water is passed
through it. If the same water is passed through the
system four times, what percent of the original ANSWER:  
contaminants
eSolutions will be
Manual - Powered byremoved
Cognero from the water 53.9918 Page 15
sample?  
 
   
ANSWER:   ANSWER:  
10-399.19%
Geometric Sequences and Series 31.9375
   

Find the sum of each geometric series. 3


  44.  a1 - 240.r - -,11- 7
4
I  
42.  a1 -36,r--.n-8
3
SOLUTION:  
SOLUTION:   Find the sum.
Find the sum.  
a1 -a1r"
s, I -r
7
240-240(})
=
36-36(~J 1-- 3
,_ 1 4

3  
=831.855
= 53.9918
ANSWER:   ANSWER:  
53.9918 831.855
   

4
I 45.  '11 =360,r=-;;-,11=8
43.  a1 -16,r--.n-9 .}

2  
 
SOLUTION:  
SOLUTION:   Find the sum.
Find the sum.  
S = 01 -<11" r" S = a1 -o1r
IJ

" 1-r " 1-r

16-16(~Y 360-36o(iJ
= =
I- I I- 4
2 3
=31.9375 =9707.82
   
ANSWER:   ANSWER:  
31.9375 9707.82
   

44.  46. VACUUMS A vacuum claims to pick up 80% of the


dirt every time it is run over the carpet. Assuming
  this is true, what percent of the original amount of
dirt is picked up after the seventh time the vacuum is
SOLUTION:   run over the carpet?
Find the sum.  
 
SOLUTION:  
Let 100% of dirt be in the carpet.
 
That is, a 0 = 1.
eSolutions Manual - Powered by Cognero Page 16
 
A vacuum claims to pick up 80% of the dirt every
   
ANSWER:   ANSWER:  
10-39707.82
Geometric Sequences and Series 99.99%
   

46. VACUUMS A vacuum claims to pick up 80% of the Find the sum of each geometric series.
dirt every time it is run over the carpet. Assuming  
this is true, what percent of the original amount of 7

dirt is picked up after the seventh time the vacuum is 47.  L,4(-3i-l
run over the carpet? ,.1
   

SOLUTION:   SOLUTION:  
Let 100% of dirt be in the carpet. Given r = –3.
  There are 7 – 1 + 1 or 7 terms, so n = 7.
1–1
That is, a 0 = 1. a 1 = 4(–3) =4
  Find the sum.
A vacuum claims to pick up 80% of the dirt every  
time it is run over the carpet. S = a1 - c,1r"
That is 20% of dirt still in the carpet. II
1-r
  7
Therefore, r = 100% – 80% = 20% or 0.2. s, _ 4 - 4 ( - 3)
The number of times (n) is 7. 1-(-3)
  =2188
 
Cl- =1(0.2)' '
L,4(-3)'-I -2188
= 0.0000128 ,.1
   
Therefore, after seven times of cleaning, 1 – ANSWER:  
0.0000128 = 0.9999872 or 99.99872% of the original
2188
dirt is picked up.
   

ANSWER:  
99.99% 48. 
 
 
Find the sum of each geometric series. SOLUTION:  
 
Given r = –2. 
47.   
There are 8 – 1 + 1 or 8 terms, so n = 8.
   
1–1
SOLUTION:   a 1 = (–3)(–2) = –3
Given r = –3.  
There are 7 – 1 + 1 or 7 terms, so n = 7. Find the sum.
a 1 = 4(–3)
1–1
=4  
Find the sum.
 

 
eSolutions Manual - Powered by Cognero Page 17

 
 
 
ANSWER:  
ANSWER:  
2188
10-3  Geometric Sequences and Series 255
 
8 9
48.  ~)-3X-2)A-I 49.  I:<-IX4)4-I
A I 4 I
   
 
SOLUTION:  
Given r = –2.  SOLUTION:  
  Given r = 4.
There are 8 – 1 + 1 or 8 terms, so n = 8.  
  There are 9 – 1 + 1 or 9 terms, so n = 9.
1–1  
a 1 = (–3)(–2) = –3
1–1
  a 1 = (–1)(4) = –1
Find the sum.  
  Find the sum.
,,  
S = a, - "•" ,,
n 1- I'
S
11
=" -a r
1 1

-3-(-3)(-2t 1-r
ss; i-(-2) -1-(-1)(4)'1
=255
s., - 1-(4)
  =-87381
8  
L(-3X 2)'-1 = 255 9
A I I:<-,><4>'-' ;-s13s1
  A I
 
ANSWER:  
255 ANSWER:  
  –87, 381
 
49. 
50. 
 
   
SOLUTION:   SOLUTION:  
Given r = 4. Given r = –1.
   
There are 9 – 1 + 1 or 9 terms, so n = 9. There are 10 – 1 + 1 or 10 terms, so n = 10.
   
1–1 1–1
a 1 = (–1)(4) = –1 a 1 = 5(–1) =5
   
Find the sum. Find the sum.
   

eSolutions Manual - Powered by Cognero Page 18


   
   

ANSWER:   ANSWER:  
10-3–87, 381
Geometric Sequences and Series 0
   

Find a 1 for each geometric series described.


50. 
 
  51. S n = –2912, r = 3, n = 6

SOLUTION:    
Given r = –1. SOLUTION:  
 
S - (II - (J J'11
There are 10 – 1 + 1 or 10 terms, so n = 10. .>,, - I

  1-r

a 1 = 5(–1)
1–1
=5 = 01 ( 1-r")
  1-r
Find the sum.  
  Substitute the corresponding values and solve for a 1.
S _a,-ar"  
II - I
1-r -2912 - 01
(I -36)
e- _5-5(-t)'o 1-3
.~,o - 1-(-1) "• = -2912. ---2_
-728
=0
  =-8
to  
~)(-t)'-1 =0
! I ANSWER:  
  –8
 
ANSWER:  
0
52. S n = –10,922, r = 4, n = 7
 
 
Find a 1 for each geometric series described.
SOLUTION:  
 
51. S n = –2912, r = 3, n = 6
 
SOLUTION:  
 
Substitute the corresponding values and solve for a 1.
 

 
Substitute the corresponding values and solve for a 1.
 

 
ANSWER:  
–2
 
  Manual - Powered by Cognero
eSolutions Page 19

ANSWER:   53. 
 
ANSWER:  
ANSWER:  
–8
10-3  Geometric Sequences and Series 64
 

52. S n = –10,922, r = 4, n = 7
54.  S,, = 4118.u,, = J 28.r = ~
  3
 
SOLUTION:  
SOLUTION:  
S,, =
a I - a I r"
e- a1 -a r
1-r '"'l - n
" I - ,.
_a 1 ( I- r")  
I -r Substitute the corresponding values and solve for a 1.
   
Substitute the corresponding values and solve for a 1.
  a, -12s(2)
4118= 3
-I 0922 =
a,(1-47)_..:.. 1-2
1-4 3
3 256 I
u, = - I 0922 . - a, --3- -4118. ~
-16383 .)

=-2 4118 256


u,---+-
  3 3
=1458
ANSWER:    
–2
  ANSWER:  
1458
53.  S,, - 1330.a,, - 486,r _ 2,  
2
  55. a n = 1024, r = 8, n = 5
SOLUTION:    
<; _ a, -a,,r SOLUTION:  
,,,- I ,. The nth term of a geometric sequence is (I
n
= .,.., I
11.11-I
.
   
Substitute the corresponding values and solve for a 1.
I 024 = a (sf-'
1

a, -486(3} a1 =0.25
1330= 2  
I- 3
2 ANSWER:  
0.25
a, -729=1330{-lJ  
a, = -665 + 729 56. a n = 1875, r = 5, n = 7
=64
   
SOLUTION:  
ANSWER:  
64 The nth term of a geometric sequence is .
   

54. 
eSolutions Manual - Powered by Cognero Page 20
 
SOLUTION:  
   
ANSWER:   ANSWER:  
10-30.25
Geometric Sequences and Series 193.75 ft
   

56. a n = 1875, r = 5, n = 7 58. CHEMISTRY Radon has a half-life of about 4


  days. This means that about every 4 days, half of the
mass of radon decays into another element. How
SOLUTION:   many grams of radon remain from an initial 60 grams
after 4 weeks?
The nth term of a geometric sequence is .
 
  • I
1875 = "1 ( s) SOLUTION:  
1875 Given a 0 = 60 g, ,. "'50% or 0.5, n = 7
"• aa I 5625
 
3
= 25
= 60{o.sf
  =0.46875
ANSWER:    
3 After 4 weeks the amount of radon remain is about
25 0.46875 g.
 
 
ANSWER:  
57. SCIENCE One minute after it is released, a gas- about 0.46875 g
filled balloon has risen 100 feet. In each succeeding  
minute, the balloon rises only 50% as far as it rose
in the previous minute. How far will it rise in 5 59. CCSS REASONING  A virus goes through a
minutes? computer, infecting the files. If one file was infected
  initially and the total number of files infected doubles
every minute, how many files will be infected in 20
SOLUTION:   minutes?
Let a 1 = 100, r = 50% or 0.5, and n = 5.  
  II SOLUTION:  
a1-air
Sn= 1-r Given, a 1 = 1, r = 200% or 2, n = 20.
100 -100(0.55)  
ll,, = a1r • I
1-0.S
100 -100(0.03125) : 1(2}" I

O.S = 524.288
100 -3.125
O.S ANSWER:  
96.875 524,288
- 0.5
 
=193.75
  60. GEOMETRY In the figure, the sides of each
The balloon will rise 193.75 feet. equilateral triangle are twice the size of the sides of
  its inscribed triangle. If the pattern continues, find the
sum of the perimeters of the first eight triangles.
ANSWER:    
193.75 ft
 

58. CHEMISTRY Radon has a half-life of about 4


eSolutions
days.Manual
This -means
Powered by Cognero
that about every4 days, half of the Page 21
mass of radon decays into another element. How
many grams of radon remain from an initial 60 grams  
 
ANSWER:   ANSWER:  
10-3524,288
Geometric Sequences and Series about 119.5 cm
   

60. GEOMETRY In the figure, the sides of each 61. PENDULUMS The first swing of a pendulum
equilateral triangle are twice the size of the sides of travels 30 centimeters. If each subsequent swing
its inscribed triangle. If the pattern continues, find the travels 95% as far as the previous swing, find the
sum of the perimeters of the first eight triangles. total distance traveled by the pendulum after the 30th
  swing.
 
SOLUTION:  
Given a 1 = 30 cm, r = 95 %, n = 30.
 
Find the sum.
   
SOLUTION:   c- a1 - a r"
. _,n = I
Here a 1 = 60 cm, r = 0.5, n = 8. 1-r
  = 30-30(0.95f°
Ss = 60-60(o.5t 1-0.95
1-0.5 ::::471
_ 59.765625  
The distance traveled is about 471 cm.
0.5
 
""119.5
  ANSWER:  
The sum of the perimeters of the first eight triangles about 471 cm
is about 119.5 cm.  
 
  62. PHONE CHAINS A school established a phone
chain in which every staff member calls two other
ANSWER:   staff members to notify them when the school closes
about 119.5 cm due to weather. The first round of calls begins with
  the superintendent calling both principals. If there are
94 total staff members and employees at the school,
61. PENDULUMS The first swing of a pendulum how many rounds of calls are there?
travels 30 centimeters. If each subsequent swing  
travels 95% as far as the previous swing, find the
total distance traveled by the pendulum after the 30th SOLUTION:  
swing. Given, a 1 = 2, r = 2 and a n = 94
   
SOLUTION:   Find n.
Given a 1 = 30 cm, r = 95 %, n = 30.  
 
Find the sum.
 

 
There are 7 rounds of calls.
   
The Manual
eSolutions distance traveled
- Powered is about
by Cognero 471 cm. ANSWER:   Page 22
 
7
 
 
SOLUTION:  
ANSWER:   Given
10-3about 471 cmSequences and Series
Geometric a. Substitute 5, 1.1 and 10 for a 1, r and n
 
respectively in  then simplify.
62. PHONE CHAINS A school established a phone aII = a I r" 1
chain in which every staff member calls two other 1
staff members to notify them when the school closes "10 - 5 ( I. I)'°
due to weather. The first round of calls begins with = 11.79
the superintendent calling both principals. If there are Substitute 5, 1.1 and 20 for a 1, r and n respectively in
94 total staff members and employees at the school,
how many rounds of calls are there? a,, = a, ,.11-1  then simplify.
  aII = aI r" 1

SOLUTION:   "lo= 5(1.1)20-1


Given, a 1 = 2, r = 2 and a n = 94 =30.58
  Substitute 5, 1.1 and 40 for a 1, r and n respectively in
Find n. a,, = a, ,.11-1  then simplify.
  1
n I aII = aI r"
an =air -
"•o =5(1.1)40-1
94 = 2c2t-1
= 205.75
94 = 2i1 b. Substitute 5, 1.1 and 40 for a 1, r and n
n = log294 <· _ a1 - a1 r"
respectively in •>n -  then simplify.
1-r
  <' _ a, - a1r
,,
..,,, -
There are 7 rounds of calls. 1-r
 
= s-s(1.,r
ANSWER:   I -I.I
7 =2212.96
  c. Each payment made is rounded to the nearest
penny, so the sum of the payments will actually be
63. TELEVISIONS High Tech Electronics advertises a more than the sum found in part b.
weekly installment plan for the purchase of a popular
brand of high definition television. The buyer pays $5 ANSWER:  
at the end of the first week, $5.50 at the end of the a. $11.79, $30.58, $205.72
second week, $6.05 at the end of the third week, and  
so on got one year. Assume 1 year = 52 weeks) b. $7052.15
   
a. What will the payments be at the end of the 10th, c. Each payment made is rounded to the nearest
20th, and 40th weeks? penny, so the sum of the payments will actually be
  more than the sum found in part b.
b. Find the total cost of the TV.  
 
c. Why is the cost found in part b not entirely 64. PROOF Derive the General Sum Formula using
accurate? theAlternate Sum Formula.
   
SOLUTION:   SOLUTION:  
Given a, = 5. and ,. = I . I
a. Substitute 5, 1.1 and 10 for a 1, r and n
respectively in (I
II
= (I
I
,-11-I  then simplify.

eSolutions Manual - Powered by Cognero Page 23

 
 
c. Each payment made is rounded to the nearest
penny, so the sum of the payments will actually be
10-3more
Geometric sum found in
than theSequences and b.
partSeries
   

64. PROOF Derive the General Sum Formula using 65. PROOF Derive a sum formula that does not include
theAlternate Sum Formula. a 1.
 
 
SOLUTION:  
SOLUTION:  

s .. "• aL General sum formula ~
• If ..
"• -a,.r
-i:;- Ahcrnatc sum formula
· • 1-r

.. "• -(l•r" '·r r·r1' • r"


a; =a, -r ·-· Formula for nth term
1-r Di, idc both sides b~ r" 1

"• "1 . r • I Formula for nth term


o,
. ,_,.
s :!!L...!!L Substitution s,
,.--• -a.r
1-r
...
Substitution.
r:
  a.
r •
o r-r
r•, ...
\tuhipl) b) - _r-.
1-r I
ANSWER:  
"
"•( I r•)
s. (~

1-r
General sum formula
I r
Simplif).

a1 -a1r
1-r
·-· -r r·r" • - r 11.(1-r ) Oh idc b) (1 r).
r" 1(1-r)
"• -a1 -r ·-· Formula for 11th term
11.(1-r•)
Simplif).
S _ a1 -a.r Substitution r• I -r
• 1-r
   
ANSWER:  
65. PROOF Derive a sum formula that does not include S : u1 -u.r Alternate sum formub
a 1. • 1-r
u,, = "• . ,..-• Fonnu la for nth tern,
  1
Divide bolh sides b) r"
SOLUTION:   (I
ar
s r: Substiunion.
1-r
t1,.r·r" 1 r" I

~,,, Mullipl) b) • ~-
1-r
II ( I r )
r
1-r
t1.{1-r•)
Divide by (I - r).
r-1(1-r)
".(1-r•)
r ·-· -r ..
Simplify.

66. OPEN ENDED Write a geometric series for which


 and n = 6.
 
 
ANSWER:   SOLUTION:  
Sample answer:
 

eSolutions Manual - Powered by Cognero Page 24


 
ANSWER:  
terms for both series will be identical (a 1 in the first
series will equal a 0 in the second series, and so on),
10-3 Geometric Sequences and Series and the series will be equal to each other.
   

66. OPEN ENDED Write a geometric series for which 68. PROOF Prove the formula for the nth term of a
geometric sequence.
,. - -3  and n = 6.  
4
  SOLUTION:  
SOLUTION:   Sample answer: Let a n = the nth term of the
Sample answer: sequence and r = the common ratio.
   
"'4" a1 - a, · r Delin it ion of the second term
256+ 192+ 144+ 108+81 +~
4 of a geometric sequence
  a.,= a1 · r Definition of the third term of
a geometric sequence
ANSWER:  
Sample answer: a, = "• · r · r Substitution
  a,• = aI . ,.2 Associative Property of
"'4" Multiplication
256 + 192 + 144 + I 08 + 81 + - .>
4 a, = a1 • ,.~·• 3- I=2
  a,, = a1 • ,.,,., 11 = 3

ANSWER:  
67. REASONING Explain how  needs to be 
Sample answer: Let a n = the nth term of the
altered to refer to the same series if k = 1 changes to sequence and r = the common ratio.
k = 0. Explain your reasoning.  
  a1 =a1 -r Definition of the second term
SOLUTION:   of a geometric sequence
Sample answer: n – 1 needs to change to n, and the "·' = a1 · r Definition oft he third term of
10 needs to change to a 9. When this happens, the a geometric sequence
terms for both series will be identical (a 1 in the first a3 = a1 • r- r Substitution
series will equal a 0 in the second series, and so on), a_, = a1 • r2 Associative Property of
and the series will be equal to each other. Multiplication
  a_, = a1 • r l.l 3 - I = 2
a,, = a1 • ,-"·' II =)
ANSWER:  
Sample answer: n – 1 needs to change to n, and the  
10 needs to change to a 9. When this happens, the
69. CHALLENGE The fifth term of a geometric
terms for both series will be identical (a 1 in the first
series will equal a 0 in the second series, and so on), sequence is th of the eighth term. If the ninth
and the series will be equal to each other. term is 702, what is the eighth term?
   

68. PROOF Prove the formula for the nth term of a SOLUTION:  


geometric sequence.
Given a 5 =  and a 9 = 702.
 
 
SOLUTION:  
Find the value of r.
Sample answer: Let a n = the nth term of the  
sequence and r = the common ratio.
 
eSolutions Manual - Powered by Cognero Page 25
 
ANSWER:  
10-3 Geometric Sequences and Series 234
   

69. CHALLENGE The fifth term of a geometric 70. CHALLENGE Use the fact that h is the geometric
4
sequence is th of the eighth term. If the ninth mean between x and y in the figure to find h in
27 terms of x and y.
term is 702, what is the eighth term?  
  c
SOLUTION:  

Given a 5 =
 
 and a 9 = 702.

 
A
A x D Y 8

Find the value of r.


  SOLUTION:  
a8 = a5r ' The triangle ADC and the triangle CDB are similar.
 
(18 l
a8 =-r Therefore, AC2 = x2 + J/  and  .
27
 
r' =27
Consider the triangle ABC.
,. = 3  
  (·'·+ nc'·= ( x+y )2
A
Find the value of a 8.
x + y = x· + J,· + J,· + y·
~ "'t .., , "t
( )
 
a9 =a8r x:' + 2.,:1• + y·' = x 2 + 21,·' + y·'
702 =a11{3) 2.,y = 21,2
tlg = 234 .\)' = J,2
  !,~ =x 'y '
ANSWER:    
234 ANSWER:  
  2 2
xy
70. CHALLENGE Use the fact that h is the geometric  
4
mean between x and y in the figure to find h in 71. OPEN ENDED Write a geometric series with 6
terms of x and y. terms and a sum of 252.
   
SOLUTION:  
Sample answer: 4 + 8 + 16 + 32 + 64 + 128
 

  ANSWER:  
Sample answer: 4 + 8 + 16 + 32 + 64 + 128
SOLUTION:    
The triangle ADC and the triangle CDB are similar.
  72. WRITING IN MATH How can you classify a
Therefore,  and  . sequence? Explain your reasoning.
   
Consider the triangle ABC. SOLUTION:  
 
Sample answer: A series is arithmetic if every pair
eSolutions Manual - Powered by Cognero of consecutive terms shares a common difference.Page 26
A series is geometric if every pair of consecutive
terms shares a common ratio. If the series displays
   
ANSWER:   ANSWER:  
10-3Sample answer:
Geometric 4 + 8 + 16and
Sequences + 32Series
+ 64 + 128 B
   

72. WRITING IN MATH How can you classify a 74. The first term of a geometric series is 5, and the
sequence? Explain your reasoning. common ratio is −2. How many terms are in the
  series if its sum is -6825?
 
SOLUTION:   F  5
Sample answer: A series is arithmetic if every pair  
of consecutive terms shares a common difference. G  9
A series is geometric if every pair of consecutive  
terms shares a common ratio. If the series displays H 10
both qualities, then it is both arithmetic and  
geometric. If the series displays neither quality, then J   12
it is neither geometric nor arithmetic.  
 
SOLUTION:  
ANSWER:   s• =!!L..!!L General sum formula
1-r
Sample answer: A series is arithmetic if every pair 5 5( ::?)"
--6825
of consecutive terms shares a common difference. I ( 2)
A series is geometric if every pair of consecutive 6825 ,. 5 - S( 2>"
terms shares a common ratio. If the series displays 3

both qualities, then it is both arithmetic and 68:?5(3) .. 5 5( ar


20.475 5 = 5( ::?)'
geometric. If the series displays neither quality, then
-20.480 = -5(-2)"
it is neither geometric nor arithmetic.
20.480 = ( 2)"
  5
4096=(-2)"
73. Which of the following is closest to "f .j.:. .
l~?
21: =4096
   
A 1.8 Therefore, J is the correct answer.
   
B 1.9
ANSWER:  
 

C 2.0
 
 
D 2.1 75. SHORT RESPONSE Danette has a savings
  account. She withdraws half of the contents every
year. After 4 years, she has $2000 left. How much
SOLUTION:  
did she have in the savings account originally?
~7.32 ~ 1.9
 
 
Option B is the correct answer. SOLUTION:  
  Given n = 4,a 4 = 2000 r = 0.5
 
ANSWER:  
B
 

74. The first term of a geometric series is 5, and the


common ratio is −2. How many terms are in the  
series if its sum is
 
F  5 Manual - Powered by Cognero
eSolutions Page 27
 
G  9
   
ANSWER:   ANSWER:  
10-3J Geometric Sequences and Series $32,000
   

75. SHORT RESPONSE Danette has a savings 76. SAT/ACT  The curve below could be part of the
account. She withdraws half of the contents every graph of which function?
year. After 4 years, she has $2000 left. How much
did she have in the savings account originally?
 
SOLUTION:  
Given n = 4,a 4 = 2000 r = 0.5
 

.L
0 '
 
u,, = a1r "I  
1 A y={x
'.2000=a1{0.5)1
  2
B y =X
"1 = 16000 -5x+4
   
ao = a1
r C y = -x+20
16,000  
ao= l
D y = logx
2
 
= 32,000 E xy=4
   
She have invested (a 0) $32,000.  
 
SOLUTION:  
ANSWER:   For x = 0, the value of y become 0 in the equation
$32,000 y={x. 
  Therefore, this is not a correct answer.
 
76. SAT/ACT  The curve below could be part of the The Graph of the equations
graph of which function? y = x2 - Sx + 4 andy = - x = 20
 represents 
 
are a parabola and a linear equation. 
So they are not the correct answer.
  y = logx
The graph of increases as x increases;
the given graph is decreasing.
Therefore, option E is the correct answer.
 
 
A ANSWER:  
  E
B  
 
77. MONEY Elena bought a high-definition LCD
C
television at the electronics store. She paid $200
  immediately and $75 each month for a year and a
D half. How much did Elena pay in total for the TV?
   
E
SOLUTION:  
  She paid $75 for 18 months.
  So, she paid in total of 200 + (75 × 18) or $1550 for 
eSolutions Manual - Powered by Cognero Page 28
SOLUTION:   TV.
 
For x = 0, the value of y become 0 in the equation
 
ANSWER:  
ANSWER:  
Neither; there is no common difference or ratio.
10-3EGeometric Sequences and Series  
 

77. MONEY Elena bought a high-definition LCD 7 13 6 11


79. 
television at the electronics store. She paid $200 25 --50 - 25 - so"
immediately and $75 each month for a year and a  
half. How much did Elena pay in total for the TV?
  SOLUTION:  
Find any common difference or ratio in the sequence.
SOLUTION:    
She paid $75 for 18 months.
So, she paid in total of 200 + (75 × 18) or $1550 for 
7
25
--5013 6
25
--5011
TV.
  13 12 11
Ratio
14 13 12
ANSWER:   l I I
$1550
Difference -50 50 50
   
I
Determine whether each sequence is There common difference is - .
arithmetic, geometric, or neither. Explain your 50
reasoning.  
  Therefore, this is an arithmetic sequence.
I 3 7 17  
78.  w·s· 20 · 20 ···· ANSWER:  
 
Arithmetic; the common difference is .
SOLUTION:   50
Find any common difference or ratio in the sequence.  
I 3 7 17
- - - 22 68 208 632
10 5 20 20 80.  -
7 17 3 9 27 81
Ratio 6  
12 7
I SOLUTION:  
Difference
2 4 2 Find any common difference or ratio in the sequence.
  22 68 208 632
There is no common difference or ratio. 3 9 27 81
  34 52 79
Ratio ......
ANSWER:  
.) .) 51 78
2 4 8
Neither; there is no common difference or ratio. Difference
  9 27 81
 
There is no common difference or ratio.
79.   

  ANSWER:  
Neither; there is no common difference or ratio.
SOLUTION:  
 
Find any common difference or ratio in the sequence.
  Find the center and radius of each circle. Then
graph the circle.
 
2 2
81. (x – 3) + (y – 1) = 25
eSolutions Manual - Powered by Cognero   Page 29

SOLUTION:  
 
ANSWER:  
10-3Neither;
GeometricthereSequences
is no common
anddifference
Series or ratio.  
 

Find the center and radius of each circle. Then 2 2


82. (x + 3) + (y + 7) = 81
graph the circle.  
 
2 2 SOLUTION:  
81. (x – 3) + (y – 1) = 25
  Rewrite the equation in the standard form of the
circle.
SOLUTION:    
Rewrite the equation in the standard form of the (x+3)2 +(y+7}2 =81
circle.
  (x + 3}2 + (y+ 1)2 = 92
(x-3)2 +(y-1)2 -25  
The center and the radius of the circle are (–3, –7)
( X-.> ')2 + ( y- 1)2 =:>-_, and 9 units respectively.
  Graph the equation.
The center and the radius of the circle are (3, 1) and  
5 units respectively. ··--- y
Graph the equation.
 
l!x+ 3f +(y+
... ~
71 =~
1

.I>"'
lif-8-1 -4-
..... -

Cl
r-,
2" 6 ax.
y
I \
... ~ ~
v 6
J '
Ai\ -8

- ~1---Jf - +(y-1}1:25 t - \
10
12
-14 I..,, ~
I

"'" r-,
0 j ,.

' I'\ ~ /
I
 
....
ANSWER:  
  (–3, –7), 9 units
l!x + 3f + (y+··---
y
7J' = ~ ..... -
ANSWER:  
(3, 1), 5 units
... ~ ~I
.I>"'
-4-20
-· r-,
2" 6 ax.
 
y
I L. \
... r,..., 6
v -8
J '~ i\ \
10
I
- '-< (.-- 3)1
,- - -- -·-
+ (y- 1)2:: 25
- -
'-~
"'" r-,
12
-14 I..,, ~

0
, j ,.
 
' I'\ ,I
.... i"'" 2 2
83. (x – 3) + (y + 7) = 50
   
SOLUTION:  
2
82. (x + 3) + (y + 7) = 81
2 Rewrite the equation in the standard form of the
  circle.
 
SOLUTION:  
Rewrite the equation in the standard form of the
circle.
 
eSolutions Manual - Powered by Cognero Page 30
 
The center and the radius of the circle are (3, –7)
and  units respectively.
10-3 Geometric Sequences and Series
   
2 2 84. Suppose y varies jointly as x and z. Find y when x =
83. (x – 3) + (y + 7) = 50
  9 and z = –5, if y = –90 when z = 15 and x = –6.
 
SOLUTION:  
SOLUTION:  
Rewrite the equation in the standard form of the
circle. Given .l' = kx: .
   
Find the value of k.
(x-3l +(y+7)2 =SO
 
(x -3}2 + (y + 1)1 = { sfi.)2 Substitute –6, 90, 15 for x, y and z respectively in
.l' = kx:  then solve for k.
 
 
The center and the radius of the circle are (3, –7)
-90-k(-6)15
and sJi  units respectively.
Graph the equation. k = -90
  (-6)1 S
I 2 ~3l'+ (y+ 7):.501 =I
I 1/ T
-6-4-?Q 2 4 ...... 101
 
Substitute 9, –5 and 1 for x, z and k respectively then
-- /l-4 \.

---- --6 evaluate.


~-8  
,10 I/ y= I (9){-5)
--
- -
-~- ./
y--45
-.161
 
 
ANSWER:  
ANSWER:   –45
(3,-7),Sfi. units  
 
85. SHOPPING A certain store found that the number
2~-31' + er+
'/ T
n: -sol of customers who will attend a sale can be modeled
I I

- -4-"' 2 4 ...... 101 by  where N is the number of


--- ---
/_4
""-6
--8
I\.
customers expected, P is the percent of the sale
discount, and t is the number of hours the sale will
last. Find the number of customers the store should
,10

---
/ expect for a sale that is 50% off and will last four
-ti~ ""' hours.
- -.16  
  SOLUTION:  
84. Suppose y varies jointly as x and z. Find y when x = Given P = 50% or 0.5 and t = 4.
9 and z = –5, if y = –90 when z = 15 and x = –6.  
 
SOLUTION:  
Given .
   
Find the value of k.
  ANSWER:  
Substitute –6, 90, 15 for x, y and z respectively in 731 customers
 then solve for k.
eSolutions Manual - Powered by Cognero   Page 31
 
Evaluate each expression if a = –2, b =  and 
  ANSWER:  

ANSWER:  
10-3–45
Geometric Sequences and Series
 
 

85. SHOPPING A certain store found that the number a-c


of customers who will attend a sale can be modeled 87. 
a+c
by N = 125';/IOOP/.  where N is the number of  
customers expected, P is the percent of the sale
discount, and t is the number of hours the sale will SOLUTION:  
last. Find the number of customers the store should Substitute the values of the variables.
expect for a sale that is 50% off and will last four  
hours. a-c = -2-(-12)
  a+c -2+(-12)
SOLUTION:   10
=-
Given P = 50% or 0.5 and t = 4. -14
  5
=--
N = 12 5 'Jloopi 7
 
-125~100(0.5)( 4)
= 731 ANSWER:  
  s
7
ANSWER:  
 
731 customers
 
88. 
Evaluate each expression if a = –2, b =  and   
c = –12. SOLUTION:  
  Substitute the values of the variables.
3ab  
86. 
c ,/-c _ (-2}'-{-12}
 
SOLUTION:  
b! - (~J
Substitute the values of the variables.
-8+12
  =--
1
9
=36
c -12  
I
::a-
6 ANSWER:  
  36
 
ANSWER:  
l
89. 
6
   
SOLUTION:  
Substitute the values of the variables.
87. 
 
 
eSolutions Manual - Powered by Cognero Page 32
SOLUTION:  
Substitute the values of the variables.
 
 
ANSWER:  
10-336
Geometric Sequences and Series
 

c+3
89. 
ab
 
SOLUTION:  
Substitute the values of the variables.
 
c+3 -12+3
--;;i;= (-2)(~)
-9
=2
3
27
=-
2
 
ANSWER:  
27
2
 

eSolutions Manual - Powered by Cognero Page 33


 
ANSWER:  
10-4 Infinite Geometric Series divergent
 

Determine whether each infinite geometric 3. 0.5 + 0.7 + 0.98 + …


series is convergent or divergent.  
 
SOLUTION:  
1. 16 – 8 + 4 – …
  Find the value of r.
 
SOLUTION:   ,. _ 0. 7
Find the value of r. - 0.5 or 1.4

,. ~16
-8
or_.!_
2
 
Since p.4 J > I , the series is divergent.
   
Since , the series is convergent.
ANSWER:  
  divergent
 
ANSWER:  
convergent 4. 1 + 1 + 1 + …
   
SOLUTION:  
2. 32 – 48 + 72 – … Find the value of r.
   
I
SOLUTION:   ,. = - or I
Find the value of r. I
   
Since 1 = 1 , the series is divergent.
,. - -48  
- 32 or -1.5
  ANSWER:  
Since 1-t.5 I> I , the series is divergent. divergent
   

ANSWER:   Find the sum of each infinite series, if it exists.


divergent  
  5. 440 + 220 + 110 + …
 
3. 0.5 + 0.7 + 0.98 + …
  SOLUTION:  
Find the value of r.
SOLUTION:    
Find the value of r.
 
 

  Since , the series is convergent.


Since , the series is divergent.  
  Find the sum.
 
ANSWER:  
divergent
 
eSolutions Manual - Powered by Cognero Page 1

4. 1 + 1 + 1 + …
 
 
ANSWER:  
ANSWER:  
880
10-4divergent
Infinite Geometric Series  
 

Find the sum of each infinite series, if it exists. 6. 520 + 130 + 32.5 + …
   
5. 440 + 220 + 110 + …
SOLUTION:  
 
Find the value of r.
SOLUTION:    
Find the value of r. 130 I
  '>?O
1•
- 520 or 4
,. - =-=--
4.JO or 2I  

  Since , the series is convergent.

Since , the series is convergent.  


Find the sum.
   
Find the sum. S= ''L
  I ,.
S=..:!L 520
1-r =
---
,_,
4.JO

2
4
I

= 693.!.
3
~880
 
 
440 + 220 + 110 + … = 880 520 + 130 + 32.5 + … =
   
ANSWER:   ANSWER:  
880
693.!.
  3
 
6. 520 + 130 + 32.5 + …
 
7. 
SOLUTION:  
 
Find the value of r.
  SOLUTION:  
Find the value of r.
 
 
Since , the series is convergent.

 
Find the sum.  
 
Since , the series is diverges and the sum
does not exist.
 
ANSWER:  
eSolutions Manual - Powered by Cognero Page 2
No sum exists.
 
 
ANSWER:  
ANSWER:  
10-4 Infinite Geometric Series No sum exists.
   

I 3 9 9. CCSS SENSE-MAKING  A certain drug has a


7.  -+-+-+
4 8 16 ... half-life of 8 hours after it is administered to a
  patient. What percent of the drug is still in the
patient’s system after 24 hours?
SOLUTION:    
Find the value of r.
SOLUTION:  
 
3 Given a 1 = 100% or 1 and r =  and n = 4.
8 or -3 2
r=-  
I 2
4
 
Since , the series is diverges and the sum
a,= {f}
does not exist. --8I
 
= 0.125 01" 12.5%
ANSWER:    
No sum exists. After 24 hours, 12.5% of the drug is still in the
  patient’s system.
 
8.  ANSWER:  
  12.5%
 
SOLUTION:  
Find the value of r. Find the sum of each infinite series, if it exists.
   
16
10. 
r =..1... or~
32 2  
9
SOLUTION:  
 
Since , the series is diverges and the
Since , the series is diverges and the sum sum does not exists.
does not exist.  
  ANSWER:  
ANSWER:   No sum exists.
No sum exists.  
 

9. CCSS SENSE-MAKING  A certain drug has a 11. 


half-life of 8 hours after it is administered to a
patient. What percent of the drug is still in the  
patient’s system after 24 hours? SOLUTION:  
 
Since , the series is convergent.
SOLUTION:    
Given a = 100% or 1 and r =
1 - Powered by Cognero
eSolutions Manual
 and n = 4. Page 3

 
 
ANSWER:  
ANSWER:  
No sum exists.
10-4  Infinite Geometric Series 15
 
,,
11.  I:(-2)·(0.5)1-I
~ I (3
13.  I:-· -
)t-1
••• '"' 2 4
   
SOLUTION:   SOLUTION:  
Since , the series is convergent.
Since , the series is convergent.
 
s-...£L  
1-r
S= '"-
-2 1-r
=--
1-0.5 I
=-4
  =~
I - .,
"'
I:<-2>·(0.s)'-' =-4
4
,., =2
   

ANSWER:  
"' I (3)•-i
~2- 4 =2
–4
   
ANSWER:  
12.  "' (4)•-
LJ· - 1 2
 
'"' 5
 
Write each repeating decimal as a fraction.
SOLUTION:    
Since , the series is convergent. 14. 
 
 
SOLUTION:  
S=...£L
1-r
J
=
I- 4
5  
=15
  Find the value of r.

., (4)•-
I:J· - 1
-15
 

hi 5
 
ANSWER:  
15
 
 
 
13. 

 
eSolutions Manual - Powered by Cognero Page 4

SOLUTION:  
 
ANSWER:  
 
10-42Infinite Geometric Series  
 

Write each repeating decimal as a fraction. 0.6-12


15. 
   
14.  0.35
SOLUTION:  
  0.642 = 0.6-12 + 0.000642 + ...
SOLUTION:   6-12 642
w--+-- + ...
0.35 - 0.35 + 0.0035 + 0.000035 t ... 1000 1000000
35 35 35  
=-+--+ + ...
100 10000 1000000 Find the value of r.
   
642
1.000.000
Find the value of r. r=
6-12
 
35 1000
r = 10000 I
35 1000
100  
.- I
100
s=.!!L
I r
  642
  = I 1000I
s=.!!L 1000
1-r
35 6-12 21-1
= or
_ 100 999 333
I- 1  
100
35 ANSWER:  
21-1
99
333
 
 
ANSWER:  
35 Determine whether each infinite geometric
99 series is convergent or divergent.
   
  16. 21 + 63 + 189 + …
 

15.  SOLUTION:  
  Find the value of r.
 
SOLUTION:  

 
Since , the series is divergent.
   
Find the value of r.
  ANSWER:  
eSolutions Manual - Powered by Cognero divergent Page 5
 
 
ANSWER:  
ANSWER:  
10-4 Infinite Geometric Series convergent
   

Determine whether each infinite geometric 3 9 27


series is convergent or divergent. 18.  -+-+-+ ...
4 8 16
   
16. 21 + 63 + 189 + …
  SOLUTION:  
Find the value of r.
SOLUTION:    
Find the value of r. 9
 
63 r=- or -3
8
r =- or 3 3 2
21 4
   
Since 131>1 , the series is divergent. Since , the series is divergent.
 
ANSWER:  
divergent ANSWER:  
  divergent
 
17. 480 + 360 + 270 + …
  S 10 40
19.  ---+-+
6 9 27 ...
SOLUTION:  
 
Find the value of r.
  SOLUTION:  
360 I Find the value of r.
r=- or -  
480 3
  10

Since , the series is convergent. r=..2....or.i


S 3
  6
 
ANSWER:  
Since , the series is divergent.
convergent
   
ANSWER:  
18. 
divergent
   
SOLUTION:   20. 0.1 + 0.01 + 0.001 + …
Find the value of r.  
 
SOLUTION:  
Find the value of r.
 

 
 
Since , the series is divergent. Since , the series is convergent.
eSolutions Manual - Powered by Cognero Page 6
 
ANSWER:  
   
ANSWER:   ANSWER:  
10-4divergent
Infinite Geometric Series divergent
   

20. 0.1 + 0.01 + 0.001 + … Find the sum of each infinite series, if it exists.
  0.01 O  
r""-or .J 22. 18 + 21.6 + 25.92 + …
SOLUTION:   0. I
 
Find the value of r.
  0. I/< I SOLUTION:  
Find the value of r. :? I .6 .,
  r"'- or I._
18
 
Since , the series is convergent. /1.2/>I
   
Since , the series is diverges and the sum
ANSWER:   does not exist.
convergent  
 
ANSWER:  
21. 0.008 + 0.08 + 0.8 + … No sum exists.
  0.08 O  
r"'-orl
SOLUTION:   0.008
Find the value of r. 23. –3 – 4.2 – 5.88 – …
  10/ > I  
SOLUTION:  
-J.:?
Find the value of r.
  r"'-or1.4
-.,~
 
Since , the series is divergent.
  IA I> I
 
ANSWER:  
Since , the series is diverges and the sum
divergent
  does not exists.
 
Find the sum of each infinite series, if it exists.
ANSWER:  
 
22. 18 + 21.6 + 25.92 + … No sum exists.
   

SOLUTION:  
24. 
Find the value of r.
   
SOLUTION:  
Find the value of r.
   
Since , the series is diverges and the sum
does not exist.
 
ANSWER:  
 
No sum exists.
  Since , the series is convergent.
eSolutions Manual - Powered by Cognero Page 7

23. –3 – 4.2 – 5.88 – …  


  Find the sum.
 
ANSWER:  
ANSWER:  
10-4No sum exists.
Infinite Geometric Series
   

I I I 12 6 3
24.  -+-+-+ ... 25.  -+-+-+ ...
2 6 18 5 5 5
   
SOLUTION:   SOLUTION:  
Find the value of r. Find the value of r.
   
6
6 I 5 I
r=-or- r=- or -
1 3 12 2
2 5
   
Since , the series is convergent. Since , the series is convergent.

   
Find the sum. Find the sum.
   
a, s = ..!!1_
S=-
1-,. I r
I 12
=-2- =-5-
1-1 1-1
3 2
3 24
=-
4 5
   
1 I I 3 12 6 3 24
-+-+-+ ... =- -+-+-+ ... =-
2 6 18 4 5 5 5 5
   
ANSWER:   ANSWER:  
3 24
4 5
   

25.  26. 
   
SOLUTION:   SOLUTION:  
Find the value of r. Find the value of r.
   

  Since , the series is convergent.

Since , the series is convergent.  


eSolutions Manual - Powered by Cognero Find the sum. Page 8
   
Find the sum.
 
ANSWER:  
ANSWER:  
10-4 Infinite Geometric Series No sum exists.
   

28 28. SWINGS If Kerry does not push any harder after


26.  21 + 14+-+ ... his initial swing, the distance traveled per swing will
3
  decrease by 10% with each swing. If his initial swing
traveled 6 feet, find the total distance traveled when
SOLUTION:   he comes to rest.
Find the value of r.  
 
14 2
r=- or -
21 3
 
Since , the series is convergent.

 
Find the sum.  
 
SOLUTION:  
S= t'i__ Given a 1 = 6 and r = 100% – 10% or 0.9
1-r
21  
Find the sum.
-1-2
 
3
-63 s • .!!L
  1-r
28 6
21 + I 4 + J + ... = 63 =--
1-0.9
  =60
 
ANSWER:   The total distance traveled is 60 ft.
63  
 
ANSWER:  
27. 32 + 40 + 50 + … 60 ft
   
SOLUTION:   Find the sum of each infinite series, if it exists.
Find the value of r.  
 
40 29. 
~.,
r=- .,_ or 1.25
 
 
Since [t .251 > I , the series is diverges and the sum SOLUTION:  
does not exists.
Since , the series is diverges and the
 
sum does not exists.
ANSWER:  
 
No sum exists.
  ANSWER:  
No sum exists.
28. SWINGS If Kerry does not push any harder after  
his initial swing, the distance traveled per swing will
decrease
eSolutions by- Powered
Manual 10% with each swing. If his initial swing
by Cognero Page 9
traveled 6 feet, find the total distance traveled when
he comes to rest. 30. 
   
ANSWER:   ANSWER:  
10-460 ft
Infinite Geometric Series No sum exists.
   
Find the sum of each infinite series, if it exists.
  31.  f ~·(1.)A-1
-> 7
29.  i~-(~)!-1
kal .> 4
hi

  SOLUTION:  

SOLUTION:   Since , the series is convergent.

Since , the series is diverges and the  

sum does not exists. s = ..!!.!_


I r
  s
ANSWER:   =-3-
No sum exists. 1- 3
  7
35
-
12
30.   

 
f-5 (3)'-
Z:: -·
.lal37
1 ~
.>5
=12
SOLUTION:    
Since , the series is diverges and the sum
ANSWER:  
does not exists.
  35
-
12
ANSWER:    
No sum exists.
 
32.  f~- (~).1-1
,., .) 3
31.   
SOLUTION:  
 
SOLUTION:   Since , the series is diverges and the

Since , the series is convergent. sum does not exists.


 
 
ANSWER:  
No sum exists.
 

33. 

 
SOLUTION:  
 
Since , the series is convergent.
eSolutions Manual - Powered by Cognero   Page 10

 
   
ANSWER:   ANSWER:  
10-4No sum exists.
Infinite Geometric Series 16
   

,. 8 (5)•-I ,. I ( I )t-1
33.  I-·
,., 3 6
- 34.  I-·
hi
-
8 12
   
SOLUTION:   SOLUTION:  

Since , the series is convergent. Since , the series is convergent.

   
S=~ S=- "1
1-r 1 -r
8 I
__ 8_
=~ - I
I - :, 1--
6 12
=16 3
=-
  22
i-1
,,. 8 5  
I-·3 _ 6
hi
( )
=16

 
 
ANSWER:  
16 ANSWER:  
  3
22
34.   

  Write each repeating decimal as a fraction.


 
SOLUTION:   35. 
 
Since , the series is convergent.
SOLUTION:  
 
The number  can be written as 
.
 

 
Find the value of r.
 
 

 
ANSWER:  
 
eSolutions Manual - Powered by Cognero Page 11

 
ANSWER:  
ANSWER:  

10-4 Infinite Geometric Series  


 

Write each repeating decimal as a fraction. 36.  0.145


   
35.  0.321
  SOLUTION:  
The number 0.145  can be written as 
SOLUTION:  
0.145 = 0.1 + 0.045 + 0.00045 + 0.0000045 + ... .
The number 0.321  can be written as   
0.321 = 0.3 + 0.021 + 0.00021 + 0.0000021 + ... . 45 45 45
  Lc1S=--+ + +...
1000 100000 10000000
21 21 21  
Lc1S=--+ + +...
1000 100000 10000000 Find the value of r.
   
Find the value of r. 45
  ,. = 100000
21 45
r- 100000 1000
21 I
1000 =
100
I  
=-
100 45
 
S=O.I+ 100~
21
1-
S=0.3+ 100~ 100
1- -15
100 =0.l+-
990
~ 21  
= 0 . .>+-
990 Therefore:
   
Therefore: - 45
  0.145=0.I +-
990
~:::-1 = O . .>+-
0 • .)i. ~ 21 8
990 =-
55
SJ  
=-
165
  ANSWER:  
8
ANSWER:   55
53  
165
  37. 
 
36.  SOLUTION:  
 
The number  can be written as 
SOLUTION:   .
The number  can be written as   
.
 
eSolutions Manual - Powered by Cognero Page 12
 
Find the value of r.
 
ANSWER:   ANSWER:  

10-4 Infinite Geometric Series


   

37.  2.18 38.  4.96


   
SOLUTION:   SOLUTION:  
The number 2.18  can be written as  The number 4.96  can be written as 
2.18 = 2 + 0.18+ 0.0018+ 0.000018+ ... . 4.96 = 4 + 0.96 + 0.0096 + 0.000096 + ... .
   
18 18 18 96 96 96
LctS=-+--+ +... LctS=-+--+ +...
100 10000 1000000 100 10000 1000000
   
Find the value of r. Find the value of r.
   
18 96
r= 10000 r= 10000
18 96
100 100

 
---I
100
 
---
I
100

18 96
S= 100 S= 100
I I
1- 1-
100 100
18 96
=- =-
99 99
   
Therefore: Therefore:
   
- 18 - 96
2.18=2 + 4.96=4+-
99 99
24 164
=- ::;-

II 33
   
ANSWER:   ANSWER:  
24 164
-11 33
   

38.  39. 
   
SOLUTION:   SOLUTION:  
The number  can be written as  The number  can be written as 
. .
   

eSolutions Manual - Powered by Cognero Page 13


   
Find the value of r. Find the value of r.
ANSWER:   ANSWER:  

10-4 Infinite Geometric Series


   

39.  0.1214 40.  0.4336


   
SOLUTION:   SOLUTION:  
The number 0.1214  can be written as  The number 0.4336 can be written as
0.1214 = 0.12 + 0.0014+0.000014 + ... . 0.4336 = 0.43 + 0.0036 + 0.000036 + ... .
   
14 14 36 36
LctS=--+ +...
Let S = l 0000 + l 000000 + ... 10000 1000000
   
Find the value of r. Find the value of r.
   
14 36
r =1000000 r= 1000000
I .J 36
10000 IOOOO
I I
=- =-
100 100
   
14 36
S= 10000
S = 10000 I
1- I I-
100
100
36
14 =--
=-- 9900
9900
 
 
Therefore:
Therefore:
 
 
- 14 0.4336 = 0.43 + ~
0.1214:0.12+-- 9900
9900  
477
601 =-
:-- 1100
4950
 
 
ANSWER:  
ANSWER:  
477
601
1100
4950
 
 
41. FANS A fan is running at 10 revolutions per second.
40.  After it is turned off, its speed decreases at a rate of
  75% per second. Determine the number of
SOLUTION:   revolutions completed by the fan after it is turned off.
 
The number can be written as
SOLUTION:  
.
  Given a 1 = 10 and r = 100% – 75% or 0.25.
 
eSolutions Manual - Powered by Cognero
Find the sum. Page 14
   
Find the value of r.
ANSWER:   ANSWER:  

10-4 Infinite Geometric Series


   

41. FANS A fan is running at 10 revolutions per second. 42. CCSS PRECISION  Kamiko deposited $5000 into
After it is turned off, its speed decreases at a rate of an account at the beginning of the year. The account
75% per second. Determine the number of earns 8% interest each year.
revolutions completed by the fan after it is turned off.  
  a. How much money will be in the account after 20
1
SOLUTION:   years? (Hint: Let 5000(1 + 0.08) represent the end
of the first year.)
Given a 1 = 10 and r = 100% – 75% or 0.25.
 
  b. Is this series convergent or divergent? Explain.
Find the sum.  
 
SOLUTION:  
S=..!i. Given principal = 5000, interest 8% or 0.08 years =
I - ,.
10 20.
=  
1-0.25
10 Amount= 5000{1 + 0.08}2°
=- a.
0.75 =23304.79
1000  
=--
75 b. It is a diverging series. The ratio is 1.08, which is
40 greater than 1.
=-  
3
  ANSWER:  
40
The fan completed  revolutions after it is turned  a. $23,304.79
3  
off.
b. It is a diverging series. The ratio is 1.08, which is
 
greater than 1.
ANSWER:    
40
43. RECHARGEABLE BATTERIES A certain
3 rechargeable battery is advertised to recharge back
  to 99.9% of its previous capacity with every charge.
If its initial capacity is 8 hours of life, how many total
42. CCSS PRECISION  Kamiko deposited $5000 into hours should the battery last?
an account at the beginning of the year. The account  
earns 8% interest each year.
  SOLUTION:  
a. How much money will be in the account after 20 Given a 1 = 8 and r = 99.9% or 0.999
1
years? (Hint: Let 5000(1 + 0.08) represent the end  
of the first year.) Find the sum.
   
b. Is this series convergent or divergent? Explain.
 
SOLUTION:  
Given principal = 5000, interest 8% or 0.08 years =
20.
   

a. ANSWER:  
eSolutions Manual - Powered by Cognero 8000 hrs Page 15
   
b. It is a diverging series. The ratio is 1.08, which is
Find the sum of each infinite series, if it exists.
a. $23,304.79  
 
b. It is a diverging series. The ratio is 1.08, which is ANSWER:  
10-4greater
Infinite than 1.
Geometric Series 8000 hrs
   

43. RECHARGEABLE BATTERIES A certain Find the sum of each infinite series, if it exists.
rechargeable battery is advertised to recharge back  
to 99.9% of its previous capacity with every charge. 7 21 63
If its initial capacity is 8 hours of life, how many total 44.  -+-+-+ ...
5 20 80
hours should the battery last?  
 
SOLUTION:  
SOLUTION:  
Find the value of r.
Given a 1 = 8 and r = 99.9% or 0.999  
  21
Find the sum. r = 20 or i
  7 4
S=....!!L 5
1-r  
8
=---
1-0.999
Since , the series is convergent.
=8000  
  Find the sum.
 
ANSWER:  
8000 hrs S=_!i_
1-r
 
7
Find the sum of each infinite series, if it exists. =_L
  I- 3
4
44.  28
=-
  5
 
SOLUTION:  
7 21 63 28
Find the value of r. -+-+-+ ... =-
  5 20 80 5
 
ANSWER:  
28
5
   
Since , the series is convergent.

  45. 
Find the sum.
 
 
SOLUTION:  
Find the value of r.
 

eSolutions Manual - Powered by Cognero   Page 16

 
Since , the series is convergent.
ANSWER:  
ANSWER:  

10-4  Infinite Geometric Series


 

15 5 5 16 4
45.  -+-+-+ ... 46.  --+--1+ ...
4
 
., ~
- ., 9 3
 
SOLUTION:   SOLUTION:  
Find the value of r. Find the value of r.
   
5 4
r=-
2 or
2
-
3 3
r=-- or --
15 3 16 4
4 9
   
Since , the series is convergent. Since , the series is convergent.

   
Find the sum. Find the sum.
   
S=~ S=~
1-r I -r
15 16

=~
I- -
3
45 64
;--
4 63
   
15 5 5 45 16 4 64
-+-+-+ ... =- --+--1+ ... =--
4 2 3 4 9 3 63
   
ANSWER:   ANSWER:  
-15 64
4 63
   

46.  47. 
   
SOLUTION:   SOLUTION:  
Find the value of r. Find the value of r.
   

   
Since , the series is convergent. Since , the series is diverges and the sum
eSolutions Manual - Powered by Cognero Page 17
  does not exists.
Find the sum.  
 
ANSWER:  
ANSWER:  
10-4 Infinite Geometric Series No sum exists.
   

15 5 10 18 12 8
47.  -+-+-+ ... 49.  --+---+
7 7 7 ...
8 2 3
   
SOLUTION:   SOLUTION:  
Find the value of r. Find the value of r.
   
5 12
2 4 7 2
r=- or - r=-or --
15 3 18 3
8 7
   
Since , the series is diverges and the sum Since , the series is convergent.

does not exists.  


  Find the sum.
 
ANSWER:  
No sum exists. s =..!!!__
I -r
 
-18
21 7 7
= 7
48.  -+-+-+ ...
16 4 3 1-(-~)
  54
;--

SOLUTION:   35
Find the value of r.  
  18 12 8 54
7
--+---+
7 7 7 ... =--35
r =...!..or.:!.  
21 3
16 ANSWER:  
  54
35
Since , the series is diverges and the sum  
does not exists. 50. MULTIPLE REPRESENTATIONS In this
  problem, you will use a square of paper that is at
least 8 inches on a side.
ANSWER:  
 
No sum exists.
a. CONCRETE Let the square be one unit. Cut
 
away one half of the square. Call this piece Term 1.
Next, cut away one half of the remaining sheet of
49.  paper. Call this piece Term 2. Continue cutting the
remaining paper in half and labeling the pieces with a
  term number as long as possible. List the fractions
SOLUTION:   represented by the pieces.
 
Find the value of r.
  b. NUMERICAL If you could cut the squares
indefinitely, you would have an infinite series. Find
eSolutions Manual - Powered by Cognero the sum of the series. Page 18
 
c. VERBAL How does the sum of the series relate
remaining paper in half and labeling the pieces with a b. 1
term number as long as possible. List the fractions  
represented by the pieces. c. The original square has area 1 unit and the area of
10-4  Infinite Geometric Series all the pieces cannot exceed 1.
b. NUMERICAL If you could cut the squares  
indefinitely, you would have an infinite series. Find
the sum of the series. 51. PHYSICS In a physics experiment, a steel ball on a
  flat track is accelerated, and then allowed to roll
c. VERBAL How does the sum of the series relate freely. After the first minute, the ball has rolled 120
to the original square of paper? feet. Each minute the ball travels only 40% as far as
  it did during the preceding minute. How far does the
ball travel?
SOLUTION:    
I I I I
a. ---- SOLUTION:  
2 4 8 16
  Given a 1 = 120 and r = 40% or 0.4.
b. Find r.  
Find the sum.
4 I  
r=-=-
1 2 s = _!:!}_
2 1-r
  120
=--
Find the sum. 1-0.4
=200
s =.!!.L
1-r  
I The ball travels 200 ft.
 
=_L
1-' ANSWER:  
2
200 ft
=I
 
 
c. The original square has area 1 unit and the area of 52. PENDULUMS A pendulum travels 12 centimeters
all the pieces cannot exceed 1. on its first swing and 95% of that distance on each
  swing thereafter. Find the total distance traveled by
the pendulum when it comes to rest.
ANSWER:    
I I I I
a.
2'4'g''i6' ... SOLUTION:  
  Given a 1 = 12 and r = 95% or 0.95
b. 1  
  Find the sum.
c. The original square has area 1 unit and the area of  
all the pieces cannot exceed 1.
 
S= "1
1-r
12
51. PHYSICS In a physics experiment, a steel ball on a =---
1-0.95
flat track is accelerated, and then allowed to roll
freely. After the first minute, the ball has rolled 120 =240
feet. Each minute the ball travels only 40% as far as  
it did during the preceding minute. How far does the The pendulum travels 240 cm.
ball travel?  
 
ANSWER:  
SOLUTION:   240 cm
Given a 1 = 120 and r = 40% or 0.4.  
  Manual - Powered by Cognero
eSolutions
53. TOYS If a rubber ball can bounce back to 95%Page 19
of its
Find the sum. original height, what is the total vertical distance that
  it will travel if it is dropped from an elevation of 30
   

ANSWER:   ANSWER:  
10-4240
Infinite
cm Geometric Series 1170 ft
   

53. TOYS If a rubber ball can bounce back to 95% of its 54. CARS During a maintenance inspection, a tire is
original height, what is the total vertical distance that removed from a car and spun on a diagnostic
it will travel if it is dropped from an elevation of 30 machine. When the machine is turned off, the
feet? spinning tire completes 20 revolutions the first second
  and 98% of the revolutions each additional second.
How many revolutions does the tire complete before
SOLUTION:   it stops spinning?
Distance traveled by the rubber ball in downward  
direction.
  SOLUTION:  
Given, a 1 = 30 and r = 95% or 0.95. Given a 1 = 20 and r = 98% or 0.98
   
Find the sum. Find the sum.
   
S=~ s-~
1-r 1-r
30 20
=
1-0.95 1-0.98
== 600 -1000
   
Distance traveled by the rubber ball in upward The tire completes 1000 revolutions.
direction.  
 
ANSWER:  
Given, a 1 = 28.5 and r = 95% or 0.95.
1000 revolutions
   
Find the sum.
  55. ECONOMICS A state government decides to
S= "• stimulate its economy by giving $500 to every adult.
1-r The government assumes that everyone who
28.5 receives the money will spend 80% on consumer
1-0.95 goods and that the producers of these goods will in
-570 turn spend 80% on consumer goods. How much
money is generated for the economy for every $500
  that the government provides?
The total distance traveled by the rubber ball is 600 +  
570 or 1170 ft.
  SOLUTION:  
Here, a 1 = 500 and r = 80% or 0.8.
ANSWER:  
1170 ft  
  Find the sum.
 
54. CARS During a maintenance inspection, a tire is
removed from a car and spun on a diagnostic
machine. When the machine is turned off, the
spinning tire completes 20 revolutions the first second
and 98% of the revolutions each additional second.
How many revolutions does the tire complete before
 
it stops spinning?
  ANSWER:  
eSolutions Manual - Powered by Cognero
SOLUTION:   $2500 Page 20
 
Given a 1 = 20 and r = 98% or 0.98
   
ANSWER:   ANSWER:  
10-41000 revolutions
Infinite Geometric Series $2500
   

55. ECONOMICS A state government decides to 56. SCIENCE MUSEUM An exhibit at a science


stimulate its economy by giving $500 to every adult. museum offers visitors the opportunity to experiment
The government assumes that everyone who with the motion of an object on a spring. One visitor
receives the money will spend 80% on consumer pulls the object down and lets it go. The object
goods and that the producers of these goods will in travels 1.2 feet upward before heading back the
turn spend 80% on consumer goods. How much other way. Each time the object changes direction, it
money is generated for the economy for every $500 decreases its distance by 20% when compared to the
that the government provides? previous direction. Find the total distance traveled by
  the object.
 
SOLUTION:  
Here, a 1 = 500 and r = 80% or 0.8. SOLUTION:  
  Here a 1 = 1.2, r = 1 – 0.2 or 0.8.
Find the sum.   s = ..!!l.-
1-r
 
S-=
01

1-r - 1.2
I 0.8
500 1.2
=- =-
1-0.8 0.2
ac2500 =6
 
 
ANSWER:  
The total distance traveled by the object is 6 feet.
$2500  
 
ANSWER:  
56. SCIENCE MUSEUM An exhibit at a science 6 ft
museum offers visitors the opportunity to experiment  
with the motion of an object on a spring. One visitor
pulls the object down and lets it go. The object Match each graph with its corresponding
travels 1.2 feet upward before heading back the description.
other way. Each time the object changes direction, it  
decreases its distance by 20% when compared to the
previous direction. Find the total distance traveled by
the object.
 
SOLUTION:  
Here a 1 = 1.2, r = 1 – 0.2 or 0.8.
  57. 
 
a. converging geometric series
 
b. diverging geometric series
 
c. converging arithmetic series
 
  d. diverging arithmetic series
The total distance traveled by the object is 6 feet.  
 
eSolutions Manual - Powered by Cognero SOLUTION:   Page 21
ANSWER:   The graph is diverging geometric series. Therefore,
6 ft option b is the correct answer.
 
ANSWER:  
ANSWER:  
b
10-46Infinite
ft Geometric Series  
 

Match each graph with its corresponding y


description.
 
y

0 Jl
58. 
 
a. converging geometric series
57.  ...
Jl
 
  b. diverging geometric series
a. converging geometric series  
  c. converging arithmetic series
b. diverging geometric series  
  d. diverging arithmetic series
c. converging arithmetic series  
 
d. diverging arithmetic series SOLUTION:  
  The graph is diverging arithmetic series. Therefore,
option d is the correct answer.
SOLUTION:    
The graph is diverging geometric series. Therefore,
option b is the correct answer. ANSWER:  
  d
 
ANSWER:  
b
 

59. 
 
a. converging geometric series
58.   
  b. diverging geometric series
a. converging geometric series  
  c. converging arithmetic series
b. diverging geometric series  
  d. diverging arithmetic series
c. converging arithmetic series  
  SOLUTION:  
d. diverging arithmetic series The graph is converging geometric series. Therefore,
  option a is the correct answer.
 
SOLUTION:  
The graph is diverging arithmetic series. Therefore, ANSWER:  
option
eSolutions d is-the
Manual correct
Powered answer.
by Cognero a Page 22
 
60. ERROR ANALYSIS  Emmitt and Austin are asked
ANSWER:  
  option a is the correct answer.
 
ANSWER:  
10-4dInfinite Geometric Series ANSWER:  
  a

1 60. ERROR ANALYSIS  Emmitt and Austin are asked


to find the sum of 1 – 1 + 1 – … . Is either of them
correct? Explain your reasoning.
 

E""'""''H-
59.  0
x Tue Sv.\.'\ is O bec.,..i..se l-~e
  S V.\.'\ o4! e"'c~ P"'iv o4! l-evlo'\S i'I
a. converging geometric series l-~e seqv.e>1ce ls O.
 
b. diverging geometric series
  A/Mltu-t
c. converging arithmetic series
  Ttr,w, " 1UI - MUl,(,U Ir I 2 1,
d. diverging arithmetic series MUi. tfu ~ 4WWjM.
 
SOLUTION:  
The graph is converging geometric series. Therefore,  
option a is the correct answer.  
 
SOLUTION:  
ANSWER:   Sample answer: Austin; the common ratio of the
a series r = –1, so the absolute value of r = 1 and the
series diverges.
60. ERROR ANALYSIS  Emmitt and Austin are asked  
to find the sum of 1 – 1 + 1 – … . Is either of them
correct? Explain your reasoning. ANSWER:  
  Sample answer: Austin; the common ratio of the
series r = –1, so the absolute value of r = 1 and the
series diverges.
 

61. PROOF Derive the formula for the sum of an


infinite geometric series.
 
SOLUTION:  
Sample answer:
 
The sum of a geometric series is .
 
For an infinite series with | r | < 1,
  .
   
SOLUTION:   Thus,
Sample answer: Austin; the common ratio of the
series r = –1, so the absolute value of r = 1 and the  
series
eSolutions diverges.
Manual - Powered by Cognero Page 23
ANSWER:  
 
Sample answer:
ANSWER:  
Sample answer: Austin; the common ratio of the  
series r = –1, so the absolute value of r = 1 and the Thus,
10-4series diverges.
Infinite Geometric Series
   

61. PROOF Derive the formula for the sum of an 62. CHALLENGE For what values of b does 3 + 9b +
infinite geometric series. 2 3
27b + 81b + … have a sum?
   
SOLUTION:   SOLUTION:  
Sample answer: 9b
  The common ratio is - or3b .
i1 3
The sum of a geometric series is Sn=----a1 -air .  
1-r The series have a sum, if the absolute value of the
  common ratio is less than 1.
For an infinite series with | r | < 1,  
. That is, .
   
Thus, S- ", -a,(0) or~. Solve the inequality.
1-r 1-r  
  l3b <I
ANSWER:   -I <3h< I
Sample answer: I I
--<b<-
S _ a1• a1r" 3 3
The sum of a geometric series is n I ,. .  
  I I
Therefore, the value of b should be --<h<- .
For an infinite series with | r | < 1, r" -+0 (IS II-+~ . 3 3
   

Thus, S - a, -a,(O) or ....:L. ANSWER:  


1-r 1-r I I
  -:«s«:
3 3
 
62. CHALLENGE For what values of b does 3 + 9b +
2 3
27b + 81b + … have a sum? 63. REASONING When does an infinite geometric
  series have a sum, and when does it not have a sum?
Explain your reasoning.
SOLUTION:  
 
The common ratio is . SOLUTION:  
  Sample answer: An infinite geometric series has a
The series have a sum, if the absolute value of the sum when the common ratio has an absolute value
common ratio is less than 1. less than 1. When this occurs, the terms will
  approach 0 as n approaches infinity. With the future
terms almost 0, the sum of the series will approach a
That is, .
limit. When the common ratio is 1 or greater, the
  terms will keep increasing and approach infinity as n
Solve the inequality. approaches infinity and the sum of the series will
  have no limit.
 
ANSWER:  
Sample answer: An infinite geometric series has a
sum when the common ratio has an absolute value
  less than 1. When this occurs, the terms will
eSolutions Manual - Powered by Cognero Page 24
approach 0 as n approaches infinity. With the future
Therefore, the value of b should be . terms almost 0, the sum of the series will approach a
  limit. When the common ratio is 1 or greater, the
ANSWER:   ANSWER:  
Sample answer: Sometimes; the statement is true for
10-4 Infinite Geometric Series all infinite geometric series.
   

63. REASONING When does an infinite geometric 65. OPEN ENDED Write an infinite series with a sum
series have a sum, and when does it not have a sum? that converges to 9.
Explain your reasoning.  
 
SOLUTION:  
SOLUTION:   Sample answer:
Sample answer: An infinite geometric series has a
sum when the common ratio has an absolute value 3+2+.'.!.+ ...
3
less than 1. When this occurs, the terms will  
approach 0 as n approaches infinity. With the future
terms almost 0, the sum of the series will approach a ANSWER:  
limit. When the common ratio is 1 or greater, the Sample answer:
terms will keep increasing and approach infinity as n 4
approaches infinity and the sum of the series will 3+2+-+ ...
have no limit. 3
   

ANSWER:   66. OPEN ENDED Write 3 – 6 + 12 – … using sigma


Sample answer: An infinite geometric series has a notation in two different ways.
sum when the common ratio has an absolute value  
less than 1. When this occurs, the terms will SOLUTION:  
approach 0 as n approaches infinity. With the future
terms almost 0, the sum of the series will approach a "'
L3(-2)1-1 =3-6+ 12- ...
limit. When the common ratio is 1 or greater, the ,.1
terms will keep increasing and approach infinity as n ....
approaches infinity and the sum of the series will I3c-2)" -3-6+ 12- ...
have no limit. ,.u
   

64. CCSS ARGUMENTS  Determine whether the ANSWER:  


following statement is sometimes, always, or never
true. Explain your reasoning.
If the absolute value of a term of any geometric
series is greater than the absolute value of the  
previous term, then the series is divergent.
67. WRITING IN MATH Explain why an arithmetic
 
series is always divergent.
SOLUTION:    
Sample answer: Sometimes; the statement is true for SOLUTION:  
all infinite geometric series.
An arithmetic series has a common difference, so
 
each term will eventually become more positive or
ANSWER:   more negative, but never approach 0. With the terms
not approaching 0, the sum will never reach a limit
Sample answer: Sometimes; the statement is true for
and the series cannot converge.
all infinite geometric series.
 
 
ANSWER:  
65. OPEN ENDED Write an infinite series with a sum
An arithmetic series has a common difference, so
that converges to 9.
each term will eventually become more positive or
 
more negative, but never approach 0. With the terms
SOLUTION:   not approaching 0, the sum will never reach a limit
Sample answer: and the series cannot converge.
eSolutions Manual - Powered by Cognero   Page 25

68. SAT/ACT  What is the sum of an infinite geometric


  series with a first term of 27 and a common ratio of
ANSWER:   each term will eventually become more positive or
more negative, but never approach 0. With the terms
not approaching 0, the sum will never reach a limit
10-4 Infinite Geometric Series and the series cannot converge.
   

67. WRITING IN MATH Explain why an arithmetic 68. SAT/ACT  What is the sum of an infinite geometric
series is always divergent. series with a first term of 27 and a common ratio of
 
SOLUTION:  
 
An arithmetic series has a common difference, so
A 18
each term will eventually become more positive or
more negative, but never approach 0. With the terms  
not approaching 0, the sum will never reach a limit B 34
and the series cannot converge.  
  C 41
 
ANSWER:   D 65
An arithmetic series has a common difference, so  
each term will eventually become more positive or E 81
more negative, but never approach 0. With the terms
 
not approaching 0, the sum will never reach a limit
and the series cannot converge. SOLUTION:  
  2
Given a 1 = 27, r = .
68. SAT/ACT  What is the sum of an infinite geometric 3
series with a first term of 27 and a common ratio of  
Find the sum.
 
  S=~
A 18 1-r
  27
B 34
=--,
,_.::.
  3
C 41 =81
   
D 65 Option E is the correct answer.
   
ANSWER:  
E
 

69. Adelina, Michelle, Masao, and Brandon each


simplified the same expression at the board. Each
student’s work is shown below. The teacher said that
while two of them had a correct answer, only one of
them had arrived at the correct conclusion using
correct steps.
 
Adelina’s work

 
Masao’s work
eSolutions Manual - Powered by Cognero Page 26
Adelina’s work  
ANSWER:  
10-4 Infinite Geometric Series H
 
 
Masao’s work 70. GRIDDED RESPONSE Evaluate log8 60 to the
nearest hundredth.
 
SOLUTION:  
  log, 60 = log,0 60
Michelle’s work log'° 8
l .2 ""1.97
.,...,..-s =~
.,.-s  
- ., .... ,l .x" 0
-
ANSWER:  
  1.97
Brandon’s work  
1 ,2
x·v,.-s =~
xs 71. GEOMETRY The radius of a large sphere was
=.l.x,eO
multiplied by a factor of  to produce a smaller 
 
Which is a completely accurate simplification? sphere.
   
F  Adelina’s work
 
G Michelle’s work
 
H Masao’s work
   
J   Brandon’s work How does the volume of the smaller sphere compare
  to the volume of the larger sphere?
 
SOLUTION:  
Masao’s work is a completely accurate A The volume of the smaller sphere is  as large.
simplification.
 
Option H is the correct answer.
  B The volume of the smaller sphere is  as large.
ANSWER:    
H
  C The volume of the smaller sphere is  as large.
 
70. GRIDDED RESPONSE Evaluate log8 60 to the
nearest hundredth. D The volume of the smaller sphere is  as large.
   
SOLUTION:   SOLUTION:  

The volume of the smaller sphere is  as large.


 
  Option C is the correct answer.
 
ANSWER:  
1.97Manual - Powered by Cognero
eSolutions
ANSWER:   Page 27
  C
 
71. GEOMETRY The radius of a large sphere was
   
ANSWER:   ANSWER:  
10-41.97
Infinite Geometric Series C
   

71. GEOMETRY The radius of a large sphere was 72. GAMES An audition is held for a TV game show.
At the end of each round, one half of the prospective
multiplied by a factor of  to produce a smaller  contestants are eliminated from the competition. On
3
sphere. a particular day, 524 contestants begin the audition.
   
a. Write an equation for finding the number of
contestants who are left after n rounds.
 
b. Using this method, will the number of contestants
Radius= r Radius = '¥ who are to be eliminated always be a whole number?
Explain.
   
How does the volume of the smaller sphere compare
to the volume of the larger sphere? SOLUTION:  
  I
a. Given a 10 = 524 and r = .
I 2
A The volume of the smaller sphere is  as large.
9  
  a" -- ,.,,o, ,11
,I'

B The volume of the smaller sphere is

 
 as large.
=524 2 ( I)"
 
C The volume of the smaller sphere is  as large. b. No; at the beginning of the third round there will
27
be 131 contestants and one-half of that is 65.5.
 
 
D The volume of the smaller sphere is  as large.
3 ANSWER:  
 
a. a = 524(.!.)"
SOLUTION:   " 2
 
The volume of the smaller sphere is  as large.
27 b. No; at the beginning of the third round there will
  be 131 contestants and one-half of that is 65.5.
Option C is the correct answer.  
 
73. CLUBS A quilting club consists of 9 members.
ANSWER:   Every week, each member must bring one completed
C quilt square.
   
a. Find the first eight terms of the sequence that
72. GAMES An audition is held for a TV game show. describes the total number of squares that have been
At the end of each round, one half of the prospective made after each meeting.
contestants are eliminated from the competition. On  
a particular day, 524 contestants begin the audition. b. One particular quilt measures 72 inches by 84
  inches and is being designed with 4-inch squares.
a. Write an equation for finding the number of After how many meetings will the quilt be complete?
contestants who are left after n rounds.  
 
SOLUTION:  
b. Using this method, will the number of contestants
who are to be eliminated always be a whole number? a. 9, 18, 27, 36, 45, 54, 63, 72
Explain.  
eSolutions
  Manual - Powered by Cognero b. The number of squares required is Page 28

SOLUTION:   .
ANSWER:  
  a. 9, 18, 27, 36, 45, 54, 63, 72
b. No; at the beginning of the third round there will  
10-4be 131 contestants
Infinite GeometricandSeries
one-half of that is 65.5. b. 42 meetings
   

73. CLUBS A quilting club consists of 9 members. Find each function value.


Every week, each member must bring one completed  
quilt square. 74. f (x) = 5x – 9,  f (6) 
   
a. Find the first eight terms of the sequence that
SOLUTION:  
describes the total number of squares that have been
made after each meeting. Substitute 6 for x and evaluate.
   
b. One particular quilt measures 72 inches by 84 /(6)=5(6)-9
inches and is being designed with 4-inch squares. =30-9
After how many meetings will the quilt be complete? =21
   
SOLUTION:  
ANSWER:  
a. 9, 18, 27, 36, 45, 54, 63, 72 21
   
b. The number of squares required is
72x84 ,78 2
--Or.> . 75. g(x) = x – x, g(4)
4x4
 
 
That is a n = 378. SOLUTION:  
  Substitute 4 for x and evaluate.
Given a 1 = 9 and d = 9.  
  g(4)=(4}2-(4)
Find n. =16-4
  =12
a,, =a, +(n-l)d  
378=9+(11-1)9 ANSWER:  
378 = 9n 12
n=42  
  2
76. h(x) = x – 2x – 1, h(3)
ANSWER:    
a. 9, 18, 27, 36, 45, 54, 63, 72
SOLUTION:  
 
Substitute 3 for x and evaluate.
b. 42 meetings
 
 
h(3) = (3)1 -2(3)-1
Find each function value. =9-6-1
 
=2
74. f (x) = 5x – 9,  f (6) 
 
 
SOLUTION:   ANSWER:  
Substitute 6 for x and evaluate. 2
   

eSolutions Manual - Powered by Cognero Page 29

 
 
ANSWER:  
10-5 Recursion and Iteration –3, 5, 13, 21, 29
 

Find the first five terms of each sequence 3. a 1 = 5, a n + 1 = 3a n + 2


described.
 
 
1. a 1 = 16, a n + 1 = a n + 4 SOLUTION:  
  a,,., = 3a,, + 2
SOLUTION:  
a1•1 = 3a + 2
1

a,,., = a,, + 4 a2 -3(5)+2or 17


a,., =t1 +41
a3 =3(17)+2 or53
a2 = 16 + 4 or 20 o4 =3(53)+2 or 161
o3 = 20 + 4 or 24 01 =3(161)+2 or485
c,, = 24 + 4 or 28  
a~ = 28 + 4 or 32 The first five terms of the sequence are 5, 17, 53,
161 and 485.
 
 
The first five terms of the sequence are 16, 20, 24,
28 and 32. ANSWER:  
 
5, 17, 53, 161, 485
ANSWER:    
16, 20, 24, 28, 32
4. a 1 = –4, a n + 1 = 2a n  – 6
 
2. a 1 = –3, a n + 1 = a n + 8
  SOLUTION:  

SOLUTION:  
a,,., ::
2a,, - 6
a1., = 2a1 - 6
a,,,, =a,, +8
a,., =a+8
1
a2 = 2 ( -4 )- 6 or - 14

a2 =-3 +8 or 5
a3 = 2 ( -1-1) - 6 or - 34

o3 =5+8 or 13 a, =2(-3-1)-6 or -74


a, =13+8or21 a1=2{-74)-6or -154
as = 21 + 8 or 29  
  The first five terms of the sequence are –4, –14, –34,
The first five terms of the sequence are –3, 5, 13, 21 –74 and –154.
and 29.  
  ANSWER:  
ANSWER:   –4, –14, –34, –74, –154
–3, 5, 13, 21, 29  
 
Write a recursive formula for each sequence.
 
3. a 1 = 5, a n + 1 = 3a n + 2
5. 3, 8, 18, 38, 78, …
   
SOLUTION:   SOLUTION:  

eSolutions Manual - Powered by Cognero Page 1


 
a. Write a recursive formula for the balance owed at
ANSWER:   the end of each month.
 
10-5–4, –14, –34,and
Recursion –74,Iteration
–154
  b. Find the balance owed after the first four months.
 
Write a recursive formula for each sequence. c. How much interest has accumulated after the first
  six months?
5. 3, 8, 18, 38, 78, …  
 
SOLUTION:  
SOLUTION:   a. Here a 1 = 1500.
a1 =3 The recursive formula for the balance owed at the
8=2(3)+2 end of each month is a.=I.Ola,,1-100 .
18-2(8)+2  
b. Substitute n = 1 in a.=I.Ola,,_1-100 .
38=2(18)+2
a1 .. I .01a0 - I 00
78=2(38)+2
=1.01(1500)-100
a,,,1 = 2a,, + 2
=1515-100
 
= 1415
ANSWER:   a1 =1.01(1415)-100
a n + 1 = 2a n + 2; a 1 = 3 = 1329.15
  a_,= 1.01(1329.15) 100
-1242.44
6. 5, 14, 41, 122, 365, …
{/~ = l.01(1242.44)-100
 
-1154.87
SOLUTION:   The balance owed after the first four months will be
a1 = 5 $1154.87.
14=3(5)-1  
c. Find the balance owed after the first six months.
41=3(14)-1
a,= l.01(1154.87)-100
122-3(41)-1
""1066.42
365 = 3(122)-1
{16 = l.01(1066.42)-100
a,,., - 3c,,, - 1
=977.84
   
Amount paid for the first six months is $600.
ANSWER:  
1500-600 ... 900
a n + 1 = 3a n – 1; a 1 = 5  
  So, after six months $900 should be paid if there is no
interest at all.
7. FINANCING Ben financed a $1500 rowing 977.08-900= 77.08
machine to help him train for the college rowing  
team. He could only make a $100 payment each After the first six months, the interest has
month, and his bill increased by 1% due to interest at accumulated to $77.08.
the end of each month.  
 
a. Write a recursive formula for the balance owed at ANSWER:  
the end of each month. a. a n = 1.01a n – 1 – 100; a 1 = 1500
 
 
b. Find the balance owed after the first four months.
b. $1415, $1329.15, $1242.44, $1154.87
 
 
c. How much interest has accumulated after the first
eSolutions Manual - Powered by Cognero c. $77.08 Page 2
six months?
   

Find the first three iterates of each function for


ANSWER:    
a. a n = 1.01a n – 1 – 100; a 1 = 1500
ANSWER:  
  –18, 74, –294
10-5 Recursion and Iteration
b. $1415, $1329.15, $1242.44, $1154.87  
 
c. $77.08 10. f (x) = 6x + 3, x 0 = –4
   
Find the first three iterates of each function for SOLUTION:  
the given initial value.
 
x1 = /(x0)
8. f (x) = 5x + 2, x 0 = 8 = 6(-4) + 3 or -21
  x2 =/(x1)
SOLUTION:  
=6(-21)+3 or -123
x1 = /(x0} x,=/(x2)
=5(8)+2or42 =6(-123)+3 or - 735
 
X2=/(x1)
The first three iterates are –21, –123 and –735.
=5(42)+2 or 212  
x, = /(x2) ANSWER:  
=5(212)+2 or 1062 –21, –123, –735
   
The first three iterates are 42, 212 and 1062.
  11. f (x) = 8x – 4, x0 = –6
 
ANSWER:  
42, 212, 1062 SOLUTION:  
  x1 =/(x0)
=8(-6)-4 or -52
9. f (x) = –4x + 2, x0 = 5
x2 =f(x1)
 
=8(-52)-4 or --120
SOLUTION:  
X.1=f(x2)
x1 = /(x0) = 8(-420)- 4 or - 3364
=-4(5)+2 or -18  
x2 =/(x1) The first three iterates are –52, –420 and –3364.
=-4(-18)+2or74  
x1 = /(x2) ANSWER:  
=-4(74)+2 01' -294 –52, –420, –3364
 
 
The first three iterates are –18, 74 and –294. CCSS PERSEVERANCE  Find the first five
  terms of each sequence described.
 
ANSWER:  
12. a 1 = 10, a n + 1 = 4a n + 1
–18, 74, –294
   
SOLUTION:  
10. f (x) = 6x + 3, x 0 = –4
 
SOLUTION:  
eSolutions Manual - Powered by Cognero Page 3
   
ANSWER:   ANSWER:  
10-5–52, –420, –3364
Recursion and Iteration –9, –10, –12, –16, –24
   

CCSS PERSEVERANCE  Find the first five 14. a 1 = 12, a n + 1 = a n + n


terms of each sequence described.
 
 
12. a 1 = 10, a n + 1 = 4a n + 1 SOLUTION:  
  a,,.1 :: ll" + II

SOLUTION:   a1•1 = a1 + I
a,,•• :: 4a,, + I a2 -(12) + I or 13
a1•1 = 4a1 + I a3 = (13) + 2 or IS
a2 = 4 ( I O) + I or 4 I a4 =(15)+3 or 18
a, :: 4 ( 4 I)+ I or 165 a1 =(18)+4 or 22
a4 =4(165)+1 or661  
The first five terms of the sequence are 12, 13, 15,
a$ =4(661)+1 or2645 18 and 22.
   
The first five terms of the sequence are 10, 41, 165,
661 and 2645. ANSWER:  
  12, 13, 15, 18, 22
 
ANSWER:  
10, 41, 165, 661, 2645 15. a 1 = –4, a n + 1 = 2a n + n
   
SOLUTION:  
13. a 1 = –9, a n + 1 = 2a n + 8
 
a,,., ::
2a,, + 11
a,., = 2a + I
1
SOLUTION:   a2 = 2 ( -4) + I or - 7
a,,., - 2a,, + 8 a3 = 2 (- 7) + 2 or - 12
a,., = 2a + 8
1
a4 =2(-12)+3 or -21
a2 = 2 ( -9) + 8 or - IO
a1 =2(-21)+4 or -38
a3 =2(-10)+8 or -12
 
a, = 2 (-12) + 8 or - 16 The first five terms of the sequence are –4, –7, –12,
a$ =2(-16)+8 or -24 –21 and –38.
   
The first five terms of the sequence are –9, –10, –12, ANSWER:  
–16 and –24.
–4, –7, –12, –21, –38
   
ANSWER:  
16. a 1 = 6, a n + 1 = 3a n – n
–9, –10, –12, –16, –24
   
SOLUTION:  
14. a 1 = 12, a n + 1 = a n + n
 
SOLUTION:  

eSolutions Manual - Powered by Cognero Page 4


   
ANSWER:   ANSWER:  
10-5–4, –7, –12, –21,
Recursion and –38
Iteration –2, –8, –36, –174, –862
   

16. a 1 = 6, a n + 1 = 3a n – n 18. a 1 = 7, a 2 = 10, a n + 2 = 2a n + a n + 1


   
SOLUTION:   SOLUTION:  
ll,,+I = 3£1,, - II a,,.2 = 2011 + a,,.,
t,1•1 = 3a1 - I a,.,.2 = 2a, + 01•1

a2 -3(6)-1 or 17 o, =2(7)+(10) or24


a3 =3(17)-2 or49 (J~= 2(1 o) +{24) or 44

a~ =3(49)-3 or 144 ,,, =2(24)+(44) or92


01 =3(144)-4 or428  
The first five terms of the sequence are 7, 10, 24, 44
 
and 92.
The first five terms of the sequence are 6, 17, 49,
 
144 and 428.
  ANSWER:  
ANSWER:   7, 10, 24, 44, 92
 
6, 17, 49, 144, 428
 
19. a 1 = 4, a 2 = 5, a n + 2 = 4a n – 2a n + 1
17. a 1 = –2, a n + 1 = 5a n + 2n  
  SOLUTION:  
SOLUTION:   a,,.2 = 4a,. - 2a,..,
a,,., = St111 + 211 a,.2 = 4t1, - 2t11.1
a,., = Sa, + 2(1) a.l =4(4)-2(5) or6
a2 =5(-2)+2(1) or -8 <J~ =4(5)-2(6) or8
"; =5(-8)+2(2) or -36 "' =4(6)-2(8) or8
a~ = 5{-36) + 2(3) or -174  
The first five terms of the sequence are 4, 5, 6, 8 and
a, =5(-174)+2(4) or -862 8.
   
The first five terms of the sequence are –2, –8, –36,
–174 and –862. ANSWER:  
  4, 5, 6, 8, 8
 
ANSWER:  
–2, –8, –36, –174, –862 20. a 1 = 4, a 2 = 3x, an = a n – 1 + 4a n – 2
   
18. a 1 = 7, a 2 = 10, a n + 2 = 2a n + a n + 1 SOLUTION:  
 
SOLUTION:  

 
eSolutions Manual - Powered by Cognero The first five terms of the sequence are 4, 3x, 3xPage
+ 5
16, 15x + 16 and 27x + 80.
   
   
ANSWER:   ANSWER:  
10-54,Recursion
5, 6, 8, 8 and Iteration 3, 2x, 8x – 9, 26x – 36, 80x – 117
   

20. a 1 = 4, a 2 = 3x, an = a n – 1 + 4a n – 2 22. a 1 = 2, a 2 = x + 3, a n = a n – 1 + 6a n – 2


   
SOLUTION:   SOLUTION:  
a,, = a,. 1 + 4011 2 a,, = a,. 1 + 6a11 2
o, = a.H + 4a1_2 C/1 = + 6(11-2
(I_H

a, =(3x)+4(4) or 3x+ 16 a, =(x+3)+6(2) orx+ 15


"J =(3.I+ 16)+4(3.,·) or 15x+ 16 "J =(x+ 15)+6(.-r+3) or h+33
,,$ =(15x+16)+4(3x+16) or27x+80 a$ =(7x+33)+6(x+ 15) or 13x+ 123
   
The first five terms of the sequence are 4, 3x, 3x + The first five terms of the sequence are 2, x + 3, x
16, 15x + 16 and 27x + 80. +15, 7x + 33 and 13x + 123.
   
ANSWER:   ANSWER:  
4, 3x, 3x + 16, 15x + 16, 27x + 80 2, x + 3, x + 15, 7x + 33, 13x + 123
   

21. a 1 = 3, a 2 = 2x, an = 4a n – 1 – 3a n – 2 23. a 1 = 1, a 2 = x, an = 3a n – 1 + 6a n – 2


   
SOLUTION:   SOLUTION:  
a,, = 4a •. 1 - 3a,, 2 a,, = 3a,, 1 + 6a,, 2
a.,= 4(11-1 -3a,_, o1 = 3a1_1 + 6a,_2
a1 = 4(2x)-3(3) or 8x-9 o_, =3(x)+6(1) or3x+6
"J -4{8.r-9)-3(2.r) or26x-36 "J =3(3x+6)+6(x) or 15.r+IS
a5 = 4{26x-36)-3(8x-9) or 80x -117 ,,$ =3(15x+ 18)+6(3x+6) or63x+90
   
The first five terms of the sequence are 3, 2x, 8x – 9, The first five terms of the sequence are 1, x, 3x +6,
26x – 36 and 80x –117. 15x + 18 and 63x + 90.
   
ANSWER:    
3, 2x, 8x – 9, 26x – 36, 80x – 117 ANSWER:  
  1, x, 3x + 6, 15x + 18, 63x + 90
 
22. a 1 = 2, a 2 = x + 3, a n = a n – 1 + 6a n – 2
 
 
Write a recursive formula for each sequence.
SOLUTION:    
24. 16, 10, 7, 5.5, 4.75, …
 
SOLUTION:  

 
The Manual
eSolutions first five termsby
- Powered ofCognero
the sequence are 2, x + 3, x Page 6
+15, 7x + 33 and 13x + 123.
 
 
ANSWER:  
ANSWER:  
1, x, 3x + 6, 15x + 18, 63x + 90
10-5  Recursion and Iteration a n + 1 = 0.25a n + 4; a 1 = 32
   

Write a recursive formula for each sequence. 26. 4, 15, 224, 50,175, …
   
24. 16, 10, 7, 5.5, 4.75, …
SOLUTION:  
 
a1 = 4
SOLUTION:  
15=(4)2-1
a1 = 16
10=0.5(16)+2 224=(15)J -I

7=0.5(10)+2 50175=(224)2-I
,
5.5=0.5{7)+2 a.,., = (a,,) - I
4.75 = 0.5(5.5) + 2  
a,,.1 = 0.Sa,, + 2 ANSWER:  
  2
a n + 1 = (a n) – 1; a 1 = 4
ANSWER:    
a n + 1 = 0.5a n + 2; a 1 = 16
27. 1, 2, 9, 730, …
   

25. 32, 12, 7, 5.75, … SOLUTION:  


  a1 = I

SOLUTION:   2=(1}'+1

"• = 32 9-{2}'+1
12 = 0.25(32) + 4 730=(9)'+1
7 =0.25(12)+4
a,,.,= (a,,) ' + I
5. 75 = 0.25(7) + 4  
a••, = 0.25a., + 4
ANSWER:  
 
3
a n + 1 = (a n) + 1; a 1 = 1
ANSWER:  
 
a n + 1 = 0.25a n + 4; a 1 = 32
  28. 9, 33, 129, 513, …
 
26. 4, 15, 224, 50,175, …
  SOLUTION:  

SOLUTION:  

 
ANSWER:  
  a n + 1 = 4a n – 3; a 1 = 9
ANSWER:    
eSolutions Manual - Powered by Cognero Page 7
2
a n + 1 = (a n) – 1; a 1 = 4 29. 480, 128, 40, 18, …
   
 
ANSWER:   ANSWER:  
3
10-5aRecursion
n + 1 = (a n)and
+ 1;Iteration
a1 = 1 a n + 1 = 0.25a n + 8; a 1 = 480
   

28. 9, 33, 129, 513, … 30. 393, 132, 45, 16, …


   
SOLUTION:   SOLUTION:  
a1 =9 "• = 393
33 =4(9)-3 ~~_393
1 .).!.--+ I
129=4(33)-3 3

513=4(129)-3 45 -- 132 + I
3
a,,.1 = 4a,, - 3 45
16=-+I
  3
an I
ANSWER:   a,,., =3+
a n + 1 = 4a n – 3; a 1 = 9
 
 
ANSWER:  
29. 480, 128, 40, 18, …
  ; a 1 = 393

SOLUTION:    
a1 =480
31. 68, 104, 176, 320, …
128 = 0.25( 480) + 8  
40=0.25(128)+8 SOLUTION:  
18 = 0.25( 40) + 8 a1 =68
a,,., = 0.25a,, + 8 104-2(68)-32
  176=2(128)-32
ANSWER:   320=2(40)-32
a n + 1 = 0.25a n + 8; a 1 = 480 a,,., = 2a,, - 32
   

30. 393, 132, 45, 16, … ANSWER:  


  a n + 1 = 2a n – 32; a 1 = 84
SOLUTION:    

32. FINANCIAL LITERACY  Mr. Edwards and his


company deposit $20,000 into his retirement account
at the end of each year. The account earns 8%
interest before each deposit.
 
a. Write a recursive formula for the balance in the
account at the end of each year.
 
b. Determine how much is in the account at the end
of each of the first 8 years.
 
 
ANSWER:   SOLUTION:  
; a = 393
eSolutions Manual - Powered by Cognero a. Let a n represent the balance on the account inPage
the 8
1
nth year. The initial balance a 0 is $0.
 
The recursive formula is a = 1.08a + 20000.
  a 6 = $146,718.58,
b. Determine how much is in the account at the end a 7 = $178,456.07,
of each of the first 8 years.
a 8 = $212,732.56
10-5  Recursion and Iteration
 
SOLUTION:  
a. Let a n represent the balance on the account in the Find the first three iterates of each function for
the given initial value.
nth year. The initial balance a 0 is $0.
 
The recursive formula is a n = 1.08a n – 1 + 20000. 33. f (x) = 12x + 8, x 0 = 4
   
b.
(/1 = 1.08( 0) + 20000 SOLUTION:  
= 20.000 Xi =/(xo)
(12 = 1.08(20000) + 20000 =12(4}+8or56
=41.600 .r2 = /(xa)
a_. = 1.08( 41600) + 20000 =12(56)+8or680
=64.928 x, =/(x2)
(I~ = 1.08( 64928) + 20000 = 12 ( 680) + 8 or 8 168
=90.122.24  
    
"s = 1.08(90122.24) + 20000 The first three iterates are 56, 680 and 8168.
 
= 117.332.02
(16 = 1.08(117332.02) + 20000 ANSWER:  
= 146. 718.58 56, 680, 8168
 
"1 = 1.08(146718.58) + 20000
= 178.456 07
34. f (x) = –9x + 1, x 0 = –6
u8 = 1.08( 178456.07) + 20000
= 212. 732.56 SOLUTION:  
                -"1 = f(xo)

ANSWER:   =-9(-6)+ I or 55
a. a n = 1.08a n – 1 + 20000 .\·2 =f(x,)
  =-9(55)+ I or -494
b. x, = .f(x2)
a 1 = $20,000,
=-9(-494) + I or 4447
a 2 = $41,600,
 
a 3 = $64,928, The first three iterates are 55, –494 and 4447.
a 4 = $90,122.24,  
a 5 = $117,332.02, ANSWER:  
a 6 = $146,718.58, 55, –494, 4447
 
a 7 = $178,456.07,
a 8 = $212,732.56 35. f (x) = –6x + 3, x0 = 8
   

Find the first three iterates of each function for SOLUTION:  


the given initial value.
 
33. f (x) = 12x + 8, x 0 = 4
  Manual - Powered by Cognero
eSolutions Page 9

SOLUTION:  
   
ANSWER:   ANSWER:  
10-555, –494, 4447
Recursion and Iteration –29, –229, –1829
   

35. f (x) = –6x + 3, x0 = 8 2


37. f (x) = –3x + 9, x 0 = 2
   
SOLUTION:   SOLUTION:  
x = f(xo)
1
x1 = f(xo)
=-6(8)+3 or -45 =-3(2)2 +9 or -3
X2 =f (x,) Xi= f(x,)
= -6(-45) + 3 or 273
=-3(-3}2+9or -18
x, = f(xz)
X3;j(.Y2)
=-6(273)+3 or -1635
=-3{-18}2 +9 or -963
 
The first three iterates are –45, 273 and –1635.  
  The first three iterates are –3, –18 and –963.
 
ANSWER:  
–45, 273, –1635 ANSWER:  
  –3, –18, –963
 
36. f (x) = 8x + 3, x0 = –4
2
38. f (x) = 4x + 5, x 0 = –2
 
 
SOLUTION:  
SOLUTION:  
x = f(xo)
1

=8(-4)+3 or -29 x1 = /(x0)


X2 = f(x,) =4(-2}2 +5 or 21
=8(-29)+3 or -229 Xi= f(x,)
X1=f(x2) = 4(21)2 +5 or 1769
;8(-229)+3 or -1829 X, -f(.Y2)
  = 4(1769)2 + 5 or 12.517.449
The first three iterates are –29, –229 and –1829.  
  The first three iterates are 21, 1769 and 12,517,449.
ANSWER:    
–29, –229, –1829 ANSWER:  
  21, 1769, 12,517,449
2  
37. f (x) = –3x + 9, x 0 = 2
2
  39. f (x) = 2x – 5x + 1, x0 = 6
SOLUTION:    
SOLUTION:  

eSolutions Manual - Powered by Cognero Page 10


   
ANSWER:   ANSWER:  
10-521, 1769, 12,517,449
Recursion and Iteration –2, 3, 6.75
   
2 I
39. f (x) = 2x – 5x + 1, x0 = 6 2
41. f (x) = x + 2x + 3, Xo=-
  2
 
SOLUTION:  
SOLUTION:  
x1 =/(x0)
x1 = J(x0)
"'2{6)2 -5(6)+ l or43
.\"2 =I (x,)
=Gr +2(f)+3or4.25
.r, = /(x1)
= 2(-13)2 - 5( 43) + l or 3484
= ( 4.25)2 + 2( 4.25) I 3 01· 29.5625
x,=f(xi) x, = J(x2)
- 2(3484)1 -5(3484) + l or 24.259.093 = (29.5625)2 + 2(29.5625) + 3 or 936.06640625
   
The first three iterates are 43, 3484 and 24,259,093. The first three iterates are 4.25, 29.5625 and
  936.06640625.
 
ANSWER:  
43, 3484, 24,259,093 ANSWER:  
  4.25, 29.5625, 936.0664
 
2
40. f (x) = –0.25x + x + 6, x0 = 8
2 I
  42. f (x) = 2x + x + 1, Xo=--
2
SOLUTION:    
x, = f(xo) SOLUTION:  
=-0.25(8)2 +(8)+6 or -2 x1 =J(x0)
Xi=f(x,)
=-0.25(-2)2 +(-2)+6 or3
=2(-~r +(-~)+ I or I
.,·, --.f{x2) Xi =f(x,)
=-0.25(3}2 +(3)+6 or6.75 =2(1}2 +(I}+ I or4
  x, = J(x2)
The first three iterates are –2, 3 and 6.75.
  = 2 (4 )1 + ( 4) + I orJ 7
 
ANSWER:   The first three iterates are 1, 4 and 37.
–2, 3, 6.75  
 
ANSWER:  
2 1, 4, 37
41. f (x) = x + 2x + 3,  
 
43. FRACTALS Consider the figures at the right. The
SOLUTION:   number of blue triangles increases according to a
specific pattern.
 

eSolutions Manual - Powered by Cognero Page 11

 
a. Write a recursive formula for the number of blue
  a. a n = 3a n – 1; a 1 =1
ANSWER:    
10-51,Recursion
4, 37 and Iteration b. 243
   

43. FRACTALS Consider the figures at the right. The 44. FINANCIAL LITERACY  Miguel’s monthly car
number of blue triangles increases according to a payment is $234.85. The recursive formula b n =
specific pattern.
1.005b n – 1 – 234.85 describes the balance left on the
 
loan after n payments. Find the balance of the
$10,000 loan after each of the first eight payments.
 
SOLUTION:  
 
a. Write a recursive formula for the number of blue Given b 0 = 10,000.
triangles in the sequence of figures.  
  b1 = 1.00560 - 234.85
b. How many blue triangles will be in the sixth .. l.005{10000)-234.85
figure?
  =9815.15
b! - 1.005/, - 234.85
SOLUTION:  
-1.005(9815.15) 234.85
a. The number triangles in the figures are 1, 3 and 9.
a1 =I =9629.38

3=3(1) b, = I .005bi - 234.85


= l.005(9629.38)-234.85
9=3(3)
=9442.67
Therefore, the recursive formula is a,, = 3a,,_1 .
b, = 1.005bs - 234.85
 
b. = 1.005(9442.67)- 234.85
"4 = 3a, =9255.04
=3(9) b; = 1.005h6 - 234.85
=27 = l.005(9255.04)-234.85
a~ =3a4 -9066.46
=3{27) b6 - I .005hs - 234.85
.. 81 = 1.005(9066.46)- 234.85
= 8876.94
= 3(81) b., : I .005b6 - 234.85
=243 = 1.005(8876.94)- 234.85
  = 8686.48
ANSWER:   bg = I .005h7 - 234.85
a. a n = 3a n – 1; a 1 =1 = 1.005(8686.48)- 234.85
  = 8495.06
b. 243  
 
ANSWER:  
44. FINANCIAL LITERACY  Miguel’s monthly car $9815.15, $9629.38, $9442.67, $9255.04, $9066.46,
payment is $234.85. The recursive formula b n = $8876.94, $8686.48, $8495.06
 
1.005b n – 1 – 234.85 describes the balance left on the
loan after n payments. Find the balance of the 45. CONSERVATION Suppose a lake is populated
$10,000 loan after each of the first eight payments. with 10,000 fish. A year later, 80% of the fish have
  Manual - Powered by Cognero
eSolutions died or been caught, and the lake is replenishedPage
with12
10,000 new fish. If the pattern continues, will the
SOLUTION:   lake eventually run out of fish? If not, will the
ANSWER:  
ANSWER:   No; the population of fish will reach 12,500. Each
$9815.15, $9629.38, $9442.67, $9255.04, $9066.46, year, 20% of 12,500 or 2500 fish, plus 10,000
10-5$8876.94,
Recursion$8686.48, $8495.06
and Iteration additional fish, yields 12,500 fish.
   

45. CONSERVATION Suppose a lake is populated 46. GEOMETRY Consider the pattern.


with 10,000 fish. A year later, 80% of the fish have  
died or been caught, and the lake is replenished with
10,000 new fish. If the pattern continues, will the
lake eventually run out of fish? If not, will the
population of the lake converge to any particular
value? Explain.
•v
 
a. Write a sequence of the total number of triangles
 
in the first six figures.
SOLUTION:    
No; the population of fish will reach 12,500. Each b. Write a recursive formula for the number of
year, 20% of 12,500 or 2500 fish, plus 10,000 triangles.
additional fish, yields 12,500 fish.  
  c. How many triangles will be in the tenth figure?
 
ANSWER:  
No; the population of fish will reach 12,500. Each SOLUTION:  
year, 20% of 12,500 or 2500 fish, plus 10,000 a. a 1 = 1, a 2 = 4, a 3 = 10,
additional fish, yields 12,500 fish.
  4-1+3(2-1)
10=-4+3(3-1)
46. GEOMETRY Consider the pattern.
 
«: = a,,_1 + 3 ( 11 - I)
 
Therefore:
a~= 10+3(4-1) or 19
  a5 =19+3(5-1) or31
a. Write a sequence of the total number of triangles a6 =31+3{6-1) or46
in the first six figures.
 
 
b. The recursive formula is a n = a n – 1 + 3(n – 1)
b. Write a recursive formula for the number of
triangles.  
  c.
c. How many triangles will be in the tenth figure? a1 =46+3(7-1) or 64
  a8 =64~ 3(8-1) or85
SOLUTION:   a9 = 85 + 3 { 9 - I) or I 09
a. a 1 = 1, a 2 = 4, a 3 = 10, a10=109+3(10-1) or 136
 
The number of triangles in the tenth figure is 136.
 

  ANSWER:  
Therefore: a. 1, 4, 10, 19, 31, 46
 
b. a n = a n – 1 + 3(n – 1)
 
c. 136
   
b. The recursive formula is a n = a n – 1 + 3(n – 1)
  Manual - Powered by Cognero
eSolutions 47. SPREADSHEETS Consider the sequence with x0 13
Page =
c. 20,000 and f (x) = 0.3x + 5000.
 
c. Sample answer: They make it easier to analyze
b. a n = a n – 1 + 3(n – 1)
recursive sequences because they can produce the
  first 100 terms instantaneously; it would take a long
10-5c.Recursion
136 and Iteration time to calculate the terms by hand.
   

47. SPREADSHEETS Consider the sequence with x0 = 48. VIDEO GAMES The final monster in Helena’s
20,000 and f (x) = 0.3x + 5000. video game has 100 health points. During the final
  battle, the monster regains 10% of its health points
after every 10 seconds. If Helena can inflict damage
a. Enter x0 in cell A1 of your spreadsheet. Enter “=
to the monster that takes away 10 health points every
(0.3)*(A1) + 5000” in cell A2. What answer does it 10 seconds without getting hurt herself, will she ever
provide? kill the monster? If so, when?
   
b. Copy cell A2, highlight cells A3 through A70, and  
paste. What do you notice about the sequence?
  SOLUTION:  
c. How do spreadsheets help analyze recursive Let’s assume that the monster dies when its health
sequences? points reach 0 and it starts the battle with all 100 of
  its health points. After the first 10-seconds, the
monster would have (100 - 10) + 0.1(100 - 10) health
SOLUTION:   points, which is 1.1(100 - 10) or 99. After 20-
a. 11,000 seconds, the monster would have 1.1(99 - 10) or 97.9
  health points. Organize this information into a table
and complete the table through 50 seconds. Then use
b. It converges to 7142.857.
the table to write a recursive formula to represent the
 
number of health points the monster has after n 10-
c. Sample answer: They make it easier to analyze second time periods.
recursive sequences because they can produce the  
first 100 terms instantaneously; it would take a long
Time
time to calculate the terms by hand. Monster’s Health
(10-second time
  Points
periods)
ANSWER:   1 1.1(100 - 10) or 99
2 1.1(99 - 10) or 97.9
a. 11,000
3 1.1(97.9 - 10) or 96.69
 
1.1(96.69 - 10) or
b. It converges to 7142.857. 4
95.359
  1.1(95.359 - 10) or
c. Sample answer: They make it easier to analyze 5
93.849
recursive sequences because they can produce the M M
first 100 terms instantaneously; it would take a long a n = 1.1(a n - 1 - 10)
n
time to calculate the terms by hand.
  M M
25 ≈ 1.65
48. VIDEO GAMES The final monster in Helena’s 26 ≈-9.18
video game has 100 health points. During the final  
battle, the monster regains 10% of its health points Between n = 25 and n = 26, the monster’s health
after every 10 seconds. If Helena can inflict damage points drop to zero. Since n represents the number of
to the monster that takes away 10 health points every 10-second time periods, the monster’s health points
10 seconds without getting hurt herself, will she ever drop to zero and the monster is defeated between 25
kill the monster? If so, when? (10) and 26(10) seconds or between 250 and 260
  seconds.
   
SOLUTION:   ANSWER:  
Let’s assume that the monster dies when its health Yes; between 250 and 260 seconds
points reach 0 and it starts the battle with all 100 of  
its health points. After the first 10-seconds, the
monster
eSolutions would
Manual haveby(100
- Powered - 10) + 0.1(100 - 10) health
Cognero 49. CCSS CRITIQUE  Marcus and Armando arePage 14
points, which is 1.1(100 - 10) or 99. After 20- finding the first three iterates of f (x) = 5x – 3 for an
seconds, the monster would have 1.1(99 - 10) or 97.9 initial value of x0 = 4. Is either of them correct?
health points. Organize this information into a table
 
Armando; Marcus included x0 with the iterates and
ANSWER:   only showed the first 2 iterates.
10-5Yes; betweenand
Recursion 250Iteration
and 260 seconds  
   

49. CCSS CRITIQUE  Marcus and Armando are 50. CHALLENGE Find a recursive formula for 5, 23,
finding the first three iterates of f (x) = 5x – 3 for an 98, 401, … .
initial value of x0 = 4. Is either of them correct?  
Explain. SOLUTION:  
 
a1 =5
t-\~YCl.\S 23=4(5)+3(1)
~(4) ::- $"(4) -) CV \1 98 = 4(23) + 3(2)
~c 11) ::- $"( 11) - , ov n 401 =4(98)+3(3)
-r\.\e ~iv-s+- \·\,wee H·ev-"'+-es "'v-e a,,., = 4a,, + 311
4, 11, "'~J. n  
ANSWER:  
a n + 1 = 4a n + 3n; a1 =5
Ar~D  
f(4)= 5(4) - 3 or 17 51. REASONING Is the statement “If the first three
/(17) = 5(17) - 3 or 82 terms of a sequence are identical, then the
sequence is not recursive” sometimes, always, or
/(82) = 5(82)- 3 or4D7 never true? Explain your reasoning.
Tfu fa'« tJrne iorab! are  
17, 82, AJUi. 407. SOLUTION:  
- Sample answer: Sometimes; the recursive formula
  could involve the first three terms. For example, 2, 2,
  2, 8, 20,… . is recursive with a n + 3 = a n + a n + 1 +
SOLUTION:   2a n + 2.
Armando; Marcus included x0 with the iterates and  
only showed the first 2 iterates. ANSWER:  
  Sample answer: Sometimes; the recursive formula
  could involve the first three terms. For example, 2, 2,
ANSWER:   2, 8, 20, . . . is recursive with a n + 3 = a n + a n + 1 +
Armando; Marcus included x0 with the iterates and 2a n + 2.
only showed the first 2 iterates.  
 
  52. OPEN ENDED Write a function for which the first
three iterates are 9, 19, and 39.
50. CHALLENGE Find a recursive formula for 5, 23,  
98, 401, … . SOLUTION:  
 
Sample answer: f (x) = 2x + 1, x0 = 4
SOLUTION:    
ANSWER:  
Sample answer: f (x) = 2x + 1, x0 = 4
 
eSolutions Manual - Powered by Cognero Page 15
53. WRITING IN MATH Why is it useful to represent
  a sequence with an explicit or recursive formula?
 
   
ANSWER:   ANSWER:  
10-5Sample answer:
Recursion andf Iteration
(x) = 2x + 1, x0 = 4 C
   

53. WRITING IN MATH Why is it useful to represent 55. EXTENDED RESPONSE Bill launches a model
a sequence with an explicit or recursive formula? rocket from ground level. The rocket’s height h in
2
  meters is given by the equation h = –4.9t + 56t,
where t is the time in seconds after the launch.
SOLUTION:    
Sample answer: In a recursive sequence, each term a. What is the maximum height the rocket will reach?
is determined by one or more of the previous terms.  
A recursive formula is used to produce the terms of  
the recursive sequence.
b. How long after it is launched will the rocket reach
  its maximum height? Round to the nearest tenth of a
ANSWER:   second.
 
Sample answer: In a recursive sequence, each term
is determined by one or more of the previous terms. c. How long after it is launched will the rocket land?
A recursive formula is used to produce the terms of Round to the nearest tenth of a second.
the recursive sequence.  
  SOLUTION:  
54. GEOMETRY In the figure shown, a + b + c = ? a. Substitute 0 for h and find the vertex of the
  equation.
-4.9/2 + 561 =0
a
The vertex of a quadratic equation is

( -~ 1(-~))
2a · 2a
.

h 56
- 2(1 =- 2( 4.9) or 5.7

  f (- 2:,) = -4.9( 5.7)2 + 56(5.7)


A 180°
  = 160
B The vertex of the equation is (5.7, 160).
Therefore, the rocket will reach the maximum height
 
of 160 m.
C
 
  b. The vertex of the equation is (5.7, 160).
D 450° Therefore, it will take 5.7 s to reach the maximum
  height.
SOLUTION:    
The sum of the exterior angles of a polygon is 360°. c. The rocket will land in 5.7 × 2 or 11.4 s after it is
Therefore, option C is the correct answer. launched.
   

ANSWER:   ANSWER:  
C a. 160 m
   
b. 5.7 s
55. EXTENDED RESPONSE Bill launches a model  
rocket from ground level. The rocket’s height h in c. 11.4 s
2
meters is given by the equation h = –4.9t + 56t,  
where t is the time in seconds after the launch.
eSolutions Manual - Powered by Cognero 56. Which of the following is true about the graphs Page
of y 16
=
 
2 2
a. What is the maximum height the rocket will reach? 3(x – 4) + 5 and y = 3(x + 4) + 5?
   
   
b. 5.7 s
  ANSWER:  
10-5c.Recursion
11.4 s and Iteration G
   

56. Which of the following is true about the graphs of y = 57. Which factors could represent the length times the
2 2 width?
3(x – 4) + 5 and y = 3(x + 4) + 5?
 
 
F Their vertices are maximums.
  A = 16x~ - 25y2
G The graphs have the same shape with different
vertices.
 
 
A (4x – 5y)(4x – 5y)
H The graphs have different shapes with different
vertices.  
  B (4x + 5y)(4x – 5y)
J One graph has a vertex that is a maximum, while  
the other graph has a vertex that is a minimum. C (4x2 – 5y)(4x2 + 5y)
   
2 2
SOLUTION:   D (4x + 5y)(4x + 5y)
The graphs have the same shape with different  
vertices. SOLUTION:  
Therefore, option G is the correct answer. Factor the area of the rectangle.
 
 
ANSWER:   16,l -25/ ={4,Y2f-(5y)2
G
  = { 4x2 -Sy){ 4x2 + Sy)
 
57. Which factors could represent the length times the
Option C is the correct answer.
width?
 
 
ANSWER:  
C
 

  Write each repeating decimal as a fraction.


A (4x – 5y)(4x – 5y)  
  58. 
B (4x + 5y)(4x – 5y)  
 
SOLUTION:  
C (4x2 – 5y)(4x2 + 5y)
 
2 2
D (4x + 5y)(4x + 5y)
 
 
SOLUTION:   Find the value of r.
Factor the area of the rectangle.  
 

 
eSolutions
OptionManual
C is- Powered by Cognero
the correct answer. Page 17
   
  ANSWER:  

ANSWER:  
10-5CRecursion and Iteration  
 

Write each repeating decimal as a fraction. 59.  5.126


   
58.  0.7
  SOLUTION:  
The number 5.126  can be written as 
SOLUTION:  
5.126 = 5 + 0.126+0.000126+ 0.000000126 + ... .
0. 7 = 0. 7 + 0.07 +0.007 + ...  
7 7 7 ,.. _ ~ 126 126
--+-+--+ ... LCl,> - + + + ...
10 100 1000 1000 1000000 1000000000
   
Find the value of r. Find the value of r.
   
7 126
r = 100 r = 1000000
7 126
10 1000
I l
=- =--
10 1000
   
 
s = ..:!.L
I- r
--126
7 S= 1000
I
=_J_Q_ 1-
I 1000
I-
10 126
=-
7 999
=-  
9
  Therefore:
126
5.126=5+
ANSWER:   999
7 • 14
=::>-
9 111
   
ANSWER:  
59.  • 14
  )-
III
SOLUTION:    
The number  can be written as 
60. 
.
 
 
SOLUTION:  
The number  can be written as 
 
.
Find the value of r.
   

eSolutions Manual - Powered by Cognero Page 18


 
Find the value of r.
 
ANSWER:   a. Write the first five terms of a sequence describing
his training schedule.
 
10-5 Recursion and Iteration
  b. When will he exceed 26 miles in one run?
 
60.  6.259 c. When will he have run 100 total miles?
   

SOLUTION:   SOLUTION:  
a. This forms a geometric sequence.
The number 6.259  can be written as 
 
6.259 = 6 + 0.259+ 0.000259 + 0.000000259 + . Given a 1 = 2, and r = 1.5
 
 
Lei S = 259 + 259 + 259 + .. _
1000 1000000 1000000000
a2 = 1.5(2) = 3
  a3 = 1.5(3) =4.5
Find the value of r. "~ = 1.5( 4.5) = 6.75
 
259 (1$ =l.5(6.75)=10.125
,. _ 1000000  
259 b. Given a n = 26
1000  
l Find n.
=-
1000
 
259 -'6 -- .,- (I .)If-I
.:,
S= 1000 1.5"·1 ;;; 13
I- I II= 7.3
1000  
259 He will exceed 26 miles in one run on the eighth
=-
999 session.
   
Therefore: c. Given S n = 100
r9
6.259=6+~  
999 Find n.
7  
aa6-
27 ~ _ a1 -a1r"
  • u -
1-r
ANSWER:   100= 2-2(1.S)"
1-1.5
6.2. -25-1-1.5"
27
  1.5" =26
II= 8.04
61. SPORTS Adrahan is training for a marathon, about
26 miles. He begins by running 2 miles. Then, when  
he runs every other day, he runs one and a half times He will have run 100 total miles during the ninth
the distance he ran the time before. session.
   
a. Write the first five terms of a sequence describing
ANSWER:  
his training schedule.
  a. 2, 3, 4.5, 6.75, 10.125
b. When will he exceed 26 miles in one run?  
  b. the eighth session
c. When
eSolutions will
Manual he have
- Powered by run 100
Cognero total miles?   Page 19
  c. during the ninth session
 
SOLUTION:  
ANSWER:   ANSWER:  
a. 2, 3, 4.5, 6.75, 10.125 2
y + 7y + 12
10-5  Recursion and Iteration  
b. the eighth session
 
65. (x – 2)(x + 6)
c. during the ninth session
 
 
SOLUTION:  
State whether the events are independent or
(x 2)(x+6)=x2 +6x 2x 12
dependent.
  =x2 +4x-12
62. tossing a penny and rolling a number cube  
 
ANSWER:  
SOLUTION:   2
x + 4x – 12
They are independent events.  
 
66. (a – 8)(a + 5)
ANSWER:    
independent
  SOLUTION:  
(a 8 )( a + 5) = a2 + Sa - Sa - 40
63. choosing first and second place in an academic = a2 - 3a - 40
competition  
 
ANSWER:  
SOLUTION:  
2
They are dependent events. a – 3a – 40
   

ANSWER:   67. (4h + 5)(h + 7)


dependent  
  SOLUTION:  
Find each product. ( 41, + 5)(11 + 7) = 4112 + 28/H 5/H 35
  = 4/,2 + 331, + 35
64. (y + 4)(y + 3)  
 
ANSWER:  
SOLUTION:  
2
4h + 33h + 35
(y+ 4)(y + 3) =/ + 3y + 4y+ 12
 
-y2+7y+l2
  68. (9p – 1)(3p – 2)
 
ANSWER:  
2
SOLUTION:  
y + 7y + 12
(9p-1){3p-2)=27 / -18p-3p+ 2
 
- 27 p1 - 2 Ip+ 2
65. (x – 2)(x + 6)  
  ANSWER:  
SOLUTION:   2
27p – 21p + 2
 

69. (2g + 7)(5g – 8)
  Manual - Powered by Cognero
eSolutions   Page 20

ANSWER:   SOLUTION:  
2
 
ANSWER:  
2
10-527p – 21p +and
Recursion 2 Iteration
 

69. (2g + 7)(5g – 8)
 
SOLUTION:  
(2g + 7)( Sg -8) = I Og2 - 16g + 3Sg - 56
=10g2+19g-56
 
ANSWER:  
2
10g + 19g – 56
 

eSolutions Manual - Powered by Cognero Page 21


ANSWER:  
7 6 5 2 4 3 3 4 2 5
g + 7g h + 21g h + 35g h + 35g h + 21g h +
6 7
10-6 The Binomial Theorem 7gh + h
 

Expand each binomial. 6


3. (x – 4)
   
5
1. (c + d)
SOLUTION:  
 
Replace n with 6 in the Binomial Theorem. 
SOLUTION:    
Replace n with 5 in the Binomial Theorem. (x -4)6 = x6 + jjC1x5( -4) + 6C2x4c-4)2 + jjCµ3( -4)3
  + jjC4x2( -4)4 + 6C5X( -4)5 + ( -4)6
(c + d)S =cs+ sC1c 4d + sC2c3d2 =X6 + ~x5(-4) + ~x4(-4)2 + ...§Lx3(-4)3
5! 2!4! 3!3!
+ ~x2( -4)4 + ~x( -4)5 + ( -4)6
+ sCJ:"2d3 + sC4cd4 + ds   4!2! 5!   
= x6 - 24x5 + 240x 4 -1280x3
=cs+ S!c4d+-1Lc3d2 + 3840x2 -6144x + 4096
4! 2!3!  
+ -1L 2d3 + 5! d4 +ds ANSWER:  
3!2!c 4!c
6 5 4 3 2
  x – 24x + 240x – 1280x + 3840x – 6144x +
=cs+ Sc4d + 10c3d2
4096
+ 1 Oc2d3 + Scd4 + ds  

ANSWER:   5
4. (2y – z)
5 4 3 2 2 3 4 5  
c + 5c d + 10c d + 10c d + 5cd + d
 
SOLUTION:  
Replace n with 5 in the Binomial Theorem.
7
2. (g + h)  
  (2y -z)5 = (2y)5 + sC1(2y)4c-z) + sC2(2y)3( -zi
+ 5C3(2y)2( -z)3 + 5C4(2y)( -z)4 + ( -z)5
SOLUTION:  
Replace n with 7 in the Binomial Theorem. 
= 32y5 + :t16y4c-z) + :ii~!8y3( -z)2
  + 3~~14/( -z)3 + :t(2y)( -z)4 + ( -z)5
     
(g + h)7 = g 7 + 7Cig 6h + 7C2,g 5h2 + 7Cig 4h3
= 32y5 -soy4z + soy3z2 -40y2z3
+1C4g3h4 +1Csg2h5 +1Ccgh6 +h7 +10yz -z -z5
=g7 + 1lg6h + ...ll...g5h2 + ...ll...g4h3  
6! 2!5! 3!4!
+ ...ll...g3h4 + ...ll...g2h5 + 1lgh6 +h7 ANSWER:  
4!3! 5!2! 6! 5 4 3 2 2 3 4 5
     32y – 80y z + 80y z – 40y z + 10yz – z
=g7 +7g6h+21g5h2 +35g4h3
2h5  
+ 35g 3h4 + 2 lg + 7gh6 +h7
  5. (x + 3)
5

ANSWER:    
7 6 5 2 4 3 3 4 2 5 SOLUTION:  
g + 7g h + 21g h + 35g h + 35g h + 21g h +
6 7 Replace n with 5 in the Binomial Theorem.
7gh + h
   

6
3. (x – 4)
 
SOLUTION:  
Replace n with 6 in the Binomial Theorem. 
eSolutions Manual - Powered by Cognero Page 1
 

 
   
ANSWER:   ANSWER:  
5 4 3 2 2 3 4 5 4 3 2 2 3 4
The –Binomial
10-632y 80y z + 80y z – 40y z + 10yz – z
Theorem y – 16y z + 96y z – 256yz + 256z
   
5 7. GENETICS If a woman is equally as likely to have
5. (x + 3)
  a baby boy or a baby girl, use binomial expansion to
determine the probability that 5 of her 6 children are
SOLUTION:   girls. Do not consider identical twins.
Replace n with 5 in the Binomial Theorem.  
 
SOLUTION:  
(x + 3)5 = x5 + 5C1x43 + 5C2x332 6
Find the probability by expanding (g + b) .
+ 5C3X233 + 5C4x34 + 35  
(g +b)6 =g6 + 6g5b + 15g4b2
= x5 + 2x4" +-1Lx3 .. 2
4! ~ 2!3! ~ + 20g3b3 + 15g2b4 + 6gb5 +b6
 
+ -1Lx2 .. 3 + 5!x ..4 + .. 5
3!2! ~ 4! ~ ~ By adding the coefficients of the polynomial, we
determine that there are 64 combinations of girls and
=x5 + 15x4 + 90x3   boys.
 
+ 270x2 + 405x + 243 5
6g b represents the number of combinations with 5
 
girls and 1 boy.
ANSWER:    
5 4 3 2 6 3
x + 15x + 90x + 270x + 405x + 243
64 32
 
-0.09375
4  
6. (y – 4z)
Therefore, the probability that 5 of her 6 children are
  girls is 0.09375.
SOLUTION:    
Replace n with 4 in the Binomial Theorem.
ANSWER:  
 
(y -4z)4 = y4 + 4Cl)l3( -4z) 3
or 0.09375
+4C2/( -4z)2 + 4C1)1( -4z)3 + ( -4z)4
 
= y4 + ~y3( -4z) + __i!_y2(-4z)2
31 2121

+ *y( -4z)3 + ( -4z)4 Find the indicated term of each expression.


~1
   
= y4 -16y3z + 96y2z2 9
8. fourth term of (b + c)
+ 256yz3 + 256z4
 
 
SOLUTION:  
ANSWER:   Use the Binomial Theorem to write the expansion in
4 3 2 2 3 4
y – 16y z + 96y z – 256yz + 256z sigma notation.
   

7. GENETICS If a woman is equally as likely to have


a baby boy or a baby girl, use binomial expansion to
determine the probability that 5 of her 6 children are  
girls. Do not consider identical twins. For the fourth term k = 3.
   

SOLUTION:  
6
Find the probability by expanding (g + b) .
  Manual - Powered by Cognero
eSolutions
 
Page 2

ANSWER:  
 
ANSWER:  
ANSWER:  
or 0.09375 4 4
10-6 The Binomial Theorem 5670x y
   

Find the indicated term of each expression. 6


10. third term of (a – 4b)
   
9
8. fourth term of (b + c)
SOLUTION:  
 
Use the Binomial Theorem to write the expansion in
SOLUTION:   sigma notation.
Use the Binomial Theorem to write the expansion in  
sigma notation.
=L
I, I,

( a - 4h) 6! t. t
  'uk!{6-k}t (-4h)'
( b+c)
')
=""" k!(9-k)!
')
L,
, n
9! b'l A A
c
 
For the third term k = 2.
   
For the fourth term k = 3. 6!
  k!(6-k)1 · c/'t{-4 b )t 6'•
= 2!{6-2)!u I••'(· -4b)",

= 2-IOa'h:
 
ANSWER:  
  4 2
240a b
ANSWER:    
6 3
84b c 8
  11. sixth term of (2c – 3d)
 

9. fifth term of (x + 3y)


8 SOLUTION:  
  Use the Binomial Theorem to write the expansion in
sigma notation.
SOLUTION:    
Use the Binomial Theorem to write the expansion in
sigma notation.
8! ±
(2<:-3df = A ok!{8-k)!(2c) ~,
(-3d}1
 
 
' .
( x+3r) = L 8! For the sixth term k = 5.
. ,uk!(8-k)(~'(3y)'
 
  8!
k!(8-k)!(2c}8 I (-3df 8!
= 5!(8-5)! (?-C)g S (-3c/) S
For the fifth term k = 4.
  = 56{2c}' (-Jdt
8! = -108.864c'd'
k•(8
. -k ) ! xs-'c' -- 4!(8-4)('-"'
8! (3.rf  
= 5670.t'y"' ANSWER:  
  3 5
–108,864c d
ANSWER:    
4 4
5670x y 12. last term of (5x + y)
5
   
6 SOLUTION:  
10. third term of (a – 4b)
  Use the Binomial Theorem to write the expansion in
sigma notation.
SOLUTION:  
eSolutions Manual - Powered by Cognero   Page 3
Use the Binomial Theorem to write the expansion in
sigma notation.
 
   
ANSWER:   ANSWER:  
3 5 5
10-6–108,864c d
The Binomial Theorem 243a
   
5 14. CCSS MODELING  The color of a particular
12. last term of (5x + y)
  flower is determined by the combination of two
genes, also called alleles. If the flower has two red
SOLUTION:   alleles r, the flower is red. If the flower has two
Use the Binomial Theorem to write the expansion in white alleles w, the flower is white. If the flower has
sigma notation. one allele of each color, the flower will be pink. In a
  lab, two pink flowers are mated and eventually
5! produce 1000 offspring. How many of the 1000
( 5x + ,·)' =~ (5x)i .t ( l')1 offspring will be pink?
. f-'k!(5-k)! .
 
 
For the last term k = 5.
 
5! (5 )} t ( )t 5! (5 )' ~ ( )'
k!(5-k)! x y = 5!(5-5)! x y
= ,, (
 
ANSWER:  
5
y
 
5
13. first term of (3a + 8b) SOLUTION:  
 
Here n = 1000.
SOLUTION:    
Use the Binomial Theorem to write the expansion in Find the term for which the powers or r and w are
sigma notation. equal.
   
s 51 11-k=k
(3a+8bf =L _ · (3a)' '{8b)' 2k =n
4 uk!(:>-k)!
  2k = 1000
For the first term k = 0. k =500
   
5! (3at' (8b)' = 5! (3at0(8h}0 Therefore, there will be 500 pink flowers in 1000
k!(5-k)! 0!(5-0)! offspring.
= 243a~  
 
ANSWER:  
ANSWER:   500
243a
5  
  Expand each binomial.
 
14. CCSS MODELING  The color of a particular 6
flower is determined by the combination of two 15. (a – b)
genes, also called alleles. If the flower has two red  
alleles r, the flower is red. If the flower has two
white alleles w, the flower is white. If the flower has SOLUTION:  
one allele of each color, the flower will be pink. In a Replace n with 6 in the Binomial Theorem.
lab, two pink flowers are mated and eventually  
produce
eSolutions 1000
Manual offspring.
- Powered How many of the 1000
by Cognero Page 4
offspring will be pink?
   
  ANSWER:  
7 6 5 2 4 3 3 4 2 5
ANSWER:   c – 7c d + 21c d – 35c d + 35c d – 21c d +
6 7
10-6500
The Binomial Theorem 7cd – d
   

Expand each binomial. 6


17. (x + 6)
   
6
15. (a – b)
SOLUTION:  
 
Replace n with 6 in the Binomial Theorem.
SOLUTION:    
Replace n with 6 in the Binomial Theorem. (x + 6)6 = x6 + 6C1x5c6) + 6C2x\6)2
  + 6CJX3(6)3 + 6C4x2(6)4 + 6Csx(6)5 + 66
(a -b)6 =a6 + !Cia5( -b) + !C:ia\-b)2
= x6 + §!x5(6) + ...§!_x\6)2 + __§!_x3(6)3
+ ij(:y}( -b)3 + ij(:4a2( -b)4 + !Csa(-b)s + ( -b)6 51 2!4! 3!3!
=a6 + £la5( -b) + -2La4(-b)2 + -2La3(-b)3 + 46~1x\6)4 + ~x(6)5 + 66
51 2!41 381       
+-2La2( -b)4 + £la( -b)5 + ( -b)6
4121 51 =x6 + 36x5 + 540x4 +4320x3
=a6 -6asb+ ! 5a4b2 -2Cxi3b3+ !5a2b4 -6ab5 +b6
+ 19, 440x2 + 46, 656x + 46, 656
   
ANSWER:  
ANSWER:  
6 5 4 2 3 3 2 4 5 6
a – 6a b + 15a b – 20a b + 15a b – 6ab + b 6 5 4 3 2
x + 36x + 540x + 4320x + 19,440x + 46,656x +
  46,656
 
7
16. (c – d) 7
  18. (y – 5)
 
SOLUTION:  
Replace n with 7 in the Binomial Theorem. SOLUTION:  
  Replace n with 7 in the Binomial Theorem.
(c -d}1 = c1 + 7C1c6( -d) + ?C2c5( -d)2 + 7C}':\-d)3  
+?C4cl(-d)4 + 7c~2(-d)5 +7Cfl:(-d)6 + (-d)7 (y -5)7 = y7 + 7CtY6( -5) + ?C2Ys( -5)2 + ?ClY\-5)3

= c1 + l!c6( -d) + ..1Lc5( -d)2 + ..1Lc4( -d)3 + 7C4y3( -5)4 +?Csr2< -5)5 + ?C6Y( -5)6 + < -5)1
6! 2!5! 3!4!
=/ + 1lv6< -5) + ..1Ly5( -5)2 + ..1Ly4( -5)3
+ ..1Lc3(-d)4 + .11..c2(-d)5 + 1lc(-d)6 + (-d)7 sr 215! 3!41
   4131 5121 61   
   + ..lLyJ( -5)4 + ..1Ly2( -5)5 + llyc -5)6 + C -5)7   
=C? -7c6d+Z.:5d2-35c4d3 413! 5!21 6!

+35c3d4-zr::2d5+7cd6-d1 =/-35y6+525y5-4375y4 +zis75y3


  -65, 625y2 + I 09, 375y - 78, 125
 
ANSWER:  
7 6 5 2 4 3 3 4 2 5 ANSWER:  
c – 7c d + 21c d – 35c d + 35c d – 21c d +
7 6 5 4 3 2
6 7 y – 35y + 525y – 4375y + 21,875y – 65,625y +
7cd – d
  109,375y – 78,125
 
6
17. (x + 6) 4
  19. (2a + 4b)
 
SOLUTION:  
SOLUTION:  
Replace n with 6 in the Binomial Theorem.
  Replace n with 4 in the Binomial Theorem.
 

    
eSolutions Manual - Powered by Cognero   Page 5

   
ANSWER:   ANSWER:  
5 4 3 2 2 3 4
7 6 5 4 3 2 243a – 1620a b + 4320a b – 5760a b + 3840ab
y – 35y + 525y – 4375y + 21,875y – 65,625y +
5
10-6109,375y – 78,125
The Binomial Theorem – 1024b
   
4 21. COMMITTEES If an equal number of men and
19. (2a + 4b)
  women applied to be on a community planning
committee and the committee needs a total of 10
SOLUTION:   people, find the probability that 7 of the members will
Replace n with 4 in the Binomial Theorem. be women. Assume that committee members will be
  chosen randomly.
(2a +4b)4 = (2a)4 +4C1(2a)3(4b)
 
+ 4C2(2a)2(4b)2 + 4C3(2a)(4b)3 + (4b)4
SOLUTION:  
= 16a4 + ri(a:i3)c4b) + 2~! (4a2)(16b2)
10
Find the probability by expanding (m + w) .
+ ~(2a)(64b3) + (2s6b4)
 
=16a4 +128a3b+384a2b2
  (m +w)10 =m10 + 10m9w +45m8w2 + 120m1w3

+512ab3+256b4 + 210m6iv4 + 252m5w5 + 21 om4w6


+ 120m\v7 +45m2w8 + 10mw9 +w10
 
 
ANSWER:   By adding the coefficients of the polynomial, we
4 3 2 2 3 4 determine that there are 1024 combinations of men
16a + 128a b + 384a b + 512ab + 256b and women.
  3 7
120m w represents the number of combinations with
5 7 women and 3 men.
20. (3a – 4b)
 
 
Therefore, the probability that 7 of the members are
SOLUTION:   120 15
women is ----:::0.117 .
Replace n with 5 in the Binomial Theorem. 1024 128
   
(3a -4b)5 = (3a)5 + sC1(3a)4< ---4b) + sC2(3a)3( ---4b)2
+ 5C3(3a)2( ---4b)3 + 5C4(3a)( ---4b)4 + ( ---4b)s ANSWER:  
= 243a5 -~(sn4)(4b) + i~i (21a 3)(16b2) 120 15
----~0.117
- 3~1 (9a2)(64b3) + ~(3a)(2s6b4)-1024b5 1024 128
= 243a5 -1620a4b +4320a3b2  
-5160a2b3+3840ab4 -J024b5
      22. BASEBALL If a pitcher is just as likely to throw a
  ball as a strike, find the probability that 11 of his first
12 pitches are balls.
ANSWER:    
5 4 3 2 2 3 4
243a – 1620a b + 4320a b – 5760a b + 3840ab
5 SOLUTION:  
– 1024b 12
  Find the probability by expanding (b + p ) .
 
21. COMMITTEES If an equal number of men and
women applied to be on a community planning
committee and the committee needs a total of 10
people, find the probability that 7 of the members will  
be women. Assume that committee members will be By adding the coefficients of the polynomial, we
chosen randomly. determine that there are 4096 combinations of balls
  and pitches.
11
SOLUTION:   12b p represents the number of combinations with
10 11 balls.
Find the probability by expanding (m + w) .  
  Therefore, the probability that 11 of his first 12
eSolutions Manual - Powered by Cognero Page 6
pitches are balls is .
 
ANSWER:   ANSWER:  

10-6 The Binomial Theorem


   

22. BASEBALL If a pitcher is just as likely to throw a Find the indicated term of each expression.
ball as a strike, find the probability that 11 of his first  
12 pitches are balls. 7
23. third term of (x + 2z)
   
SOLUTION:   SOLUTION:  
12
Find the probability by expanding (b + p ) . Use the Binomial Theorem to write the expansion in
  sigma notation.
(b + p)12 =b12 + 12b11P + 66b1op2 + 220b9p3  
+495b8p4 + 792b7p5 + 924b6p6 + 792b5/ • 71
(.,. + ?-)'
-- -- ""
L, ( . ) ( .\·)~ l (',.._
>·)t
+495b4p8 + 220b3p9 + 66b2pl0 + 12bpll + pl2 ,0 k!7-k!
   
By adding the coefficients of the polynomial, we For the third term k = 2.
determine that there are 4096 combinations of balls  
and pitches.
11 7! ( )' i (' )' 7! ( )' .: (2 ):
12b p represents the number of combinations with k!(7-k)! x c: =2!(7-2)! x =
11 balls. = 84xs=:
   
Therefore, the probability that 11 of his first 12
12 3 ANSWER:  
pitches are balls is -- -- ~ 0.00293 . 5 2
-W96 1024 84x z
   
ANSWER:  
., .., 6
I
--- ;::; _.,_~ 0.00293 24. fourth term of (y – 3x)
-W96 I 02-t  
  SOLUTION:  
Find the indicated term of each expression. Use the Binomial Theorem to write the expansion in
  sigma notation.
7  
23. third term of (x + 2z) 6'
=L ·
t,
  (.r-3.r)" (J')" '(-3x)'
, uk!{6-k)!
SOLUTION:    
Use the Binomial Theorem to write the expansion in For the fourth term k = 3.
sigma notation.  
 
6! (,·)& t {-Jr)t = 6! ( ·)& '(-Jr)'
k!(6-k)! . . 3!(6-3)! J .
= -540y'x'
   
For the third term k = 2.
  ANSWER:  
3 3
–540y x
 
8
  25. seventh term of (2a – 2b)
 
ANSWER:  
5 2 SOLUTION:  
84x z
Use the Binomial Theorem to write the expansion in
  Manual - Powered by Cognero
eSolutions Page 7
sigma notation.
 
6
   
ANSWER:   ANSWER:  
3 3 5
10-6–540y x
The Binomial Theorem 75,000xy
   
8 9
25. seventh term of (2a – 2b) 27. fifth term of (x – 4)
   
SOLUTION:   SOLUTION:  
Use the Binomial Theorem to write the expansion in Use the Binomial Theorem to write the expansion in
sigma notation. sigma notation.
   
91
(2a-2b) l! =L...
~
(
8! ) (2a )H t (
-2b )' (x-4f
'I
=I, ~k!{9-k)!
· (xf '(-4}1
, ,k! 8-k !
   
For the seventh term k = 6. For the fifth term k = 4.
   
t: (-4r = 4!(9-4)! ( r (-4)'
8! 8!
(2u}8t(-2b)'= (2a)"..,,(-2bt 9! ( 9!
k!(8-k)! 6!(8-6)! k!(9-k)! x .r
= 7168a:b 0
=32.256.l
 
 
ANSWER:  
ANSWER:  
2 6
7168a b 5
32,256x
 
 
6
26. sixth term of (4x + 5y) 8
28. fourth term of (c + 6)
 
 
SOLUTION:  
SOLUTION:  
Use the Binomial Theorem to write the expansion in
Use the Binomial Theorem to write the expansion in
sigma notation.
sigma notation.
 
 
, , 8! ( )' t ( 6 )'
(c:+6) = ~k!{8-k)! c
 
 
For the sixth term k = 5.
For the fourth term k = 3.
 
6' 6'  
k!{6-k)! (4 x )"'(" )_I')' = 5!(6-5)! (4 x )°\5 J'f 8! ( t: ( 6)' - 3!(8-3)!
8! ( )tt-1 (6)'
k!(8-k)! c c
= 75. 000.,J·'
  = 12.096c~
 
ANSWER:  
5 ANSWER:  
75,000xy
5
  12,096c
 
9
27. fifth term of (x – 4)
  Expand each binomial.
 
SOLUTION:  
Use the Binomial Theorem to write the expansion in 29. 
sigma notation.
   
eSolutions Manual - Powered by Cognero SOLUTION:   Page 8
Replace n with 5 in the Binomial Theorem.
   
  ANSWER:  
ANSWER:  
5
10-612,096c
The Binomial Theorem  
 

Expand each binomial.


  30. 

29.   

  SOLUTION:  
Replace n with 4 in the Binomial Theorem.
SOLUTION:    
Replace n with 5 in the Binomial Theorem.
 

   
ANSWER:  
~-1,2, 4 I
  x --x· +-x· --x+-
3 3 27 81
 
ANSWER:  

31. 
 
 
SOLUTION:  
30.  Replace n with 5 in the Binomial Theorem.
 
 
('2h + 41)s = ('2h) s + 5C1('2h) 4(1) + 5C2('2h) 3(1)2
4 4
SOLUTION:  
+sC3('2h)2(±)3 +5C4('2h)(±)4 + (±)5
Replace n with 4 in the Binomial Theorem.
  = 3'2h5 + 2(16b4
~ )(l)4 + -1L(8b3)(J_)
2~ 16  
+ 3~!(4b2)(6~)+*'('2h)(2~6)+ 10~4
= ~'2hs + 20?4 + 5b3 + 2b2 + _s_b +-1-
, 8 128 1024
 
ANSWER:  
 
~ 4 -, 5, 5 I
32.h +20h +)/,· +-!,· +- h+- -
8 128 1024
 
 
32. 
ANSWER:  
 
SOLUTION:  
 
eSolutions Manual - Powered by Cognero Replace n with 5 in the Binomial Theorem. Page 9
 
31. 
SOLUTION:  
ANSWER:  
a. Substitute n = 10, k = 9, p = 0.7 and q = 0.3 in

10-6 The Binomial Theorem .


 
 

32.  ( 3c-+ I )5 IO! (0.7)" (0.3) = ~(0.7}9 (0.3)


3 9!(10 9)! 9!1!
  = 1 o(o.1f (o.3)
SOLUTION:   ::::0.121
Replace n with 5 in the Binomial Theorem. b. Substitute n = 10, k = 8, p = 0.6 and q = 0.4 in
  n! s n A

m
(3c + 1r = (3c)5 + 5C1(3c)4 + 5C2(3c)3(1Y
---pq
k!(11-k )!
.

+ 5C3(3c)2(1)3 + 5C4(3c)(ff + (1)5


 

= 243c5 + *'(sr4)(1) + ii~1 (27c3)(i)   IO!


8!(10-8)!
(0.6}8(0.4}2 =~(0.6}8{0.4}1
8!2!
+ 3~1 (9c2) (2~) + *'(3c) (ti)+ 2!3 = 10·9 (0.6)8{0.4)1
= 243c5 + ! '5c4 + 'Oc3 +.!Qc2 +_1__c +-1 2· I
, , 3 27 243
  = 45(0.6)\o.-1l
=:0.121
ANSWER:   c. Substitute n = 5, k = 2, p = 0.3 and q = 0.7 in
~ • ~ , 10 , 5 I n! ,
243c· + 13:,,· +30c· +-,· · +-,·+- ---pq
n A
.
3 27 243 k!(11-k )!
   
 
S! {0.3)1 (0.7)' - S! {0.3)2 (0.7f
2!(5-2)! 2!3!
33. CCSS SENSE-MAKING  In --,--p n! ' qn A , let
k!(11-k)! = ~(o.3t (o.7)'
2· I
p represent the likelihood of a success and q
represent the likelihood of a failure. = 10(0.3}' (0.7}1
  =: 0.309
a. If a place-kicker makes 70% of his kicks within 40
yards, find the likelihood that he makes 9 of his next ANSWER:  
10 attempts from within 40 yards. a. 0.121
   
b. If a quarterback completes 60% of his passes, b. 0.121
find the likelihood that he completes 8 of his next 10
attempts.  
  c. 0.309
c. If a team converts 30% of their two-point  
conversions, find the likelihood that they convert 2 of
their next 5 conversions. 34. CHALLENGE Find the sixth term of the expansion
  of . Explain your reasoning.
SOLUTION:    
a. Substitute n = 10, k = 9, p = 0.7 and q = 0.3 in
SOLUTION:  
n! , ,, •
---pq . Use the Binomial Theorem to write the expansion in
k!(n-k)!
sigma notation.
   

 
For the sixth term k = 5.
eSolutions Manual - Powered by Cognero Page 10
 
b. Substitute n = 10, k = 8, p = 0.6 and q = 0.4 in
.
b. 0.121 Sample answer: While they have the same terms, the
n
  signs for (x + y) will all be positive, while the signs
n
10-6cThe
. 0.309
Binomial Theorem for (x – y) will alternate.
   

34. CHALLENGE Find the sixth term of the expansion 36. REASONING Determine whether the following
statement is true or false. Explain your reasoning.
of . Explain your reasoning.  
  The eighth and twelfth terms of (x + y)
20
have the
SOLUTION:   same coefficients.
 
Use the Binomial Theorem to write the expansion in
sigma notation. SOLUTION:  
  th
Sample answer: False; a binomial to the 20 power
( [;; + ./b)•: = ~ 12 ! ( /;i)i: ' ( lb)' will have 21 terms. The eleventh term will be in the
~k!{l2-k)!
  middle and the rest of the terms will be symmetric.
For the sixth term k = 5. The tenth term corresponds with the twelfth term.
   
12!
k!(12-k)!
(,/(It• ( Jbr; 12!
5!(12-5}!
( ,/(It s(Ji,r
ANSWER:  
; 792a 'b1 J;;b th
Sample answer: False; a binomial to the 20 power
 
will have 21 terms. The eleventh term will be in the
ANSWER:   middle and the rest of the terms will be symmetric.
The tenth term corresponds with the twelfth term.
   

35. REASONING Explain how the terms of (x + y) 37. OPEN ENDED Write a power of a binomial for
n 4
and (x – y) are the same and how they are which the second term of the expansion is 6x y.
different.  
 
SOLUTION:  
SOLUTION:  
Sample answer: While they have the same terms, the Sample answer:
n
signs for (x + y) will all be positive, while the signs  
n
for (x – y) will alternate.
  ANSWER:  

ANSWER:   Sample answer: ( x 6 )~


Sy
Sample answer: While they have the same terms, the
 
n
signs for (x + y) will all be positive, while the signs
n
for (x – y) will alternate. 38. WRITING IN MATH Explain how to write out the
  terms of Pascal’s triangle.
 
36. REASONING Determine whether the following SOLUTION:  
statement is true or false. Explain your reasoning.
Sample answer: The first row is a 1. The second row
  is two 1s. Each new row begins and ends with 1.
20
The eighth and twelfth terms of (x + y) have the Each coefficient is the sum of the two coefficients
same coefficients. above it in the previous row.
   
SOLUTION:   ANSWER:  
th Sample answer: The first row is a 1. The second row
Sample answer: False; a binomial to the 20 power
eSolutions Manual - Powered by Cognero
will have 21 terms. The eleventh term will be in the is two 1s. Each new row begins and ends with 1. Page 11
Each coefficient is the sum of the two coefficients
middle and the rest of the terms will be symmetric. above it in the previous row.
ANSWER:   Sample answer: The first row is a 1. The second row
is two 1s. Each new row begins and ends with 1.
Sample answer: Each coefficient is the sum of the two coefficients
10-6 The Binomial Theorem above it in the previous row.
   

38. WRITING IN MATH Explain how to write out the 39. PROBABILITY A desk drawer contains 7
terms of Pascal’s triangle. sharpened red pencils, 5 sharpened yellow pencils, 3
  unsharpened red pencils, and 5 unsharpened yellow
pencils. If a pencil is taken from the drawer at
SOLUTION:   random, what is the probability that it is yellow, given
Sample answer: The first row is a 1. The second row that it is one of the sharpened pencils?
is two 1s. Each new row begins and ends with 1.  
Each coefficient is the sum of the two coefficients 5
above it in the previous row. A
12
   
ANSWER:   7
B
Sample answer: The first row is a 1. The second row 20
is two 1s. Each new row begins and ends with 1.  
Each coefficient is the sum of the two coefficients 5
C
above it in the previous row. 8
   
39. PROBABILITY A desk drawer contains 7 D -
5
sharpened red pencils, 5 sharpened yellow pencils, 3
unsharpened red pencils, and 5 unsharpened yellow  
pencils. If a pencil is taken from the drawer at SOLUTION:  
random, what is the probability that it is yellow, given Favorable outcomes: {5 sharpened yellow pencils}
that it is one of the sharpened pencils? Possible outcomes: {7 sharpened red pencils, 5
  sharpened yellow pencils}
A 5
Probability  
12
   
B A is the correct option.
 
 
ANSWER:  
C
A
   
D 40. GRIDDED RESPONSE Two people are 17.5 miles
  apart. They begin to walk toward each other along a
straight line at the same time. One walks at the rate
SOLUTION:   of 4 miles per hour, and the other walks at the rate of
Favorable outcomes: {5 sharpened yellow pencils} 3 miles per hour. In how many hours will they meet?
Possible outcomes: {7 sharpened red pencils, 5  
sharpened yellow pencils}
SOLUTION:  
Probability   Let the distance traveled by one person be x.
So, the distance traveled by the other person be 17.5
  – x.
A is the correct option.  
  The equation that represents this situation is
ANSWER:   .
A
 
 
eSolutions Manual - Powered by Cognero Page 12

40. GRIDDED RESPONSE Two people are 17.5 miles


apart. They begin to walk toward each other along a
   
ANSWER:   ANSWER:  
10-6AThe Binomial Theorem 2.5
   

40. GRIDDED RESPONSE Two people are 17.5 miles 41. GEOMETRY Christie has a cylindrical block that
apart. They begin to walk toward each other along a she needs to paint for an art project.
straight line at the same time. One walks at the rate  
of 4 miles per hour, and the other walks at the rate of
3 miles per hour. In how many hours will they meet?
  I
20 in.
SOLUTION:  
Let the distance traveled by one person be x.
_!
So, the distance traveled by the other person be 17.5
– x.
 
 
What is the surface area of the cylinder in square
The equation that represents this situation is
inches rounded to the nearest square inch?  
.\· 17.5-x
.  
-t 3 F 1960
   
x 17.5 x G 2413
-=
4 3  
Jx-4(17.5-x} H 5127
3x=10-4x  
x=10 J 6634
   
Find the time taken by substituting 10 for distance
SOLUTION:  
and 4 for speed.
  The radius of the cylinder is 12 in.
 
Distance
Time taken = --- Substitute 20 for h and 12 for r in the formula to find
peed the volume V of the cylinder.
10  
=-
4 SA= 2m·1 + Tnrh
=2.5
;:;; 2,T(12)1 + 2;r(12){20)
 
So, the two people will meet after 2.5 hrs. - 288;r - 480;r - 768;r
  ::: 2-t I Jin:

ANSWER:    
G is the correct option.
2.5  
 
ANSWER:  
41. GEOMETRY Christie has a cylindrical block that G
she needs to paint for an art project.
 
 
42. Which of the following is a linear function?
 
A
 
B
   
What
eSolutions is the- Powered
Manual surfaceby
area of the
cylinder in square
Cognero Page 13
inches rounded to the nearest square inch?   C
   
   
ANSWER:   ANSWER:  
10-6GThe Binomial Theorem C
   

42. Which of the following is a linear function? Find the first five terms of each sequence.
 
A
,. x+3 _  
43. a 1 = –2, a n + 1 = a n + 5
• X r2
 
 
SOLUTION:  
B
a,,.1 = a., + S
 
C .
,. x+J
2
_ {I ••

(I,
= "1 + 5
-2+5 or 3
II - I
_.,
"•
  a,= 3+5 or 8 a. -3
D y-'3.rj +2 ", -8+5 or 13 a, 8
  (I~ = 13 + 5 orl8 a, 13
 
SOLUTION:  
x+3 The first five terms of the sequence are –2, 3, 8, 13
The graph of the function .
\':::--
2  is a straight line. and 18.
 
 
x+3 ANSWER:  
So, .
,·=---2  is a linear function.
–2, 3, 8, 13, 18
   
C is the correct option.
 
44. a 1 = 3, a n + 1 = 4a n – 10
ANSWER:    
C
  SOLUTION:  
a,,., - 4a,, - IO
Find the first five terms of each sequence. t11•1 = 4a1 - IO II= 1
 
": -4(3)-10 or1 "• 3
43. a 1 = –2, a n + 1 = a n + 5
01 - 4 ( 2) - IO or - 2 {I. - '.!
 
a, =4(-2)-10 or -18 a1- 2
SOLUTION:  
a, -4(-18)-10 or -82 a ·-18
 
The first five terms of the sequence are 3, 2, –2, –18
and –82.
 
ANSWER:  
3, 2, –2, –18, –82
 
 
The first five terms of the sequence are –2, 3, 8, 13
and 18.
45. a 1 = 4, a n + 1 = 3a n – 6
 
 
ANSWER:  
SOLUTION:  
–2, 3, 8, 13, 18
 
eSolutions Manual - Powered by Cognero Page 14

44. a 1 = 3, a n + 1 = 4a n – 10
 
   
ANSWER:   ANSWER:  
2, –2,
10-63,The –18, –82Theorem
Binomial 4, 6, 12, 30, 84
   

45. a 1 = 4, a n + 1 = 3a n – 6 Find the sum of each infinite geometric series,


  if it exists.
 
SOLUTION:   3
46.  -6+3--+ ...
a,,.1 = 3a,, - 6 2
a1•1 = 3a1 - 6 II= I  
a1 -3( 4)-6 or 6 SOLUTION:  
a1 =3(6)-6 or 12 a, =6 Find the value of r to determine if the sum exists.
 
a, = 3 ( 12 )- 6 or 30 a, = 12 .
.)

u,.-3(30)-6 or84 o~-30 r=-


-6
  I
The first five terms of the sequence are 4, 6, 12, 30 =--
2
and 84.  
 
Since , the sum exists.
ANSWER:  
4, 6, 12, 30, 84  
  Use the formula to find the sum.
 
Find the sum of each infinite geometric series, (I
S=-
if it exists. 1 - ,.
  -6
46.  1-(- ~)
  -6
SOLUTION:   =-r
Find the value of r to determine if the sum exists. 2
  =-4
 
ANSWER:  
–4
   

Since , the sum exists.


47. 
 
Use the formula to find the sum.  
  SOLUTION:  
Find the value of r to determine if the sum exists.

eSolutions Manual - Powered by Cognero


  Page 15

  Since , the sum exists.


ANSWER:   ANSWER:  
–4
10-6  The Binomial Theorem
 

3
47.  -+-+-+
I I 48.  Jj +3+ Jn + ...
4 4 12 ...
 
 
SOLUTION:  
SOLUTION:   Find the value of r to determine if the sum exists.
Find the value of r to determine if the sum exists.  
I 3
r--+-
4 4
I 4
=-x-
4 3  
I
3 Since IJJl> 1 , the series diverges and the sum does
  not exist.
 
Since , the sum exists.
ANSWER:  
 
No sum exists.
Use the formula to find the sum.
   

S=~ 49. TRAVEL A trip between two towns takes 4 hours


1-r under ideal conditions. The first 150 miles of the trip
3 is on an interstate, and the last 130 miles is on a
highway with a speed limit that is 10 miles per hour
,_,
=-4-
less than on the interstate.
3  
a. If x represents the speed limit on the interstate,
write expressions for the time spent at that speed and
for the time spent on the other highway.
 
b. Write and solve an equation to find the speed
limits on the two highways.
 
9
SOLUTION:  
8
a.
I
=1-
8
 
Distance traveled on the interstate is 150 miles.
ANSWER:  
So, the time spent on the interstate is .
I _!.
8 Distance traveled on highways is 130 miles.
  Speed on highways is x – 10.
So, the time spent on the highways is .
48. 
   
b.
SOLUTION:   Total time taken for the trip is 4 hrs.
Find the value of r to determine if the sum exists.
.
 
Solve for x.
eSolutions Manual - Powered by Cognero Page 16
 
 
ANSWER:  
b. ; 75 mph, 65 mph
10-6No
Thesum exists. Theorem
Binomial
   

49. TRAVEL A trip between two towns takes 4 hours State whether each statement is true or false
under ideal conditions. The first 150 miles of the trip when n = 1. Explain.
is on an interstate, and the last 130 miles is on a  
highway with a speed limit that is 10 miles per hour (11+ 1){11+ I)
less than on the interstate. 50.  -'------'--''--.....:.
2
- ?
-
 
 
a. If x represents the speed limit on the interstate,
write expressions for the time spent at that speed and SOLUTION:  
for the time spent on the other highway. Substitute n = 1 in the equation and simplify.
   
b. Write and solve an equation to find the speed (11 + l){n-+ 1)
limits on the two highways. 2 =2
 
(1+1)(1+1) _.,
SOLUTION:   2 --
a. ...
- -?
. Distance 2 --
TI mc taken = ---
Speed 2=2 /
Distance traveled on the interstate is 150 miles.  
150 So, given statement is true for n = 1.
So, the time spent on the interstate is .
.\'  
Distance traveled on highways is 130 miles.
Speed on highways is x – 10. ANSWER:  
130 ..:..(1_+.....:..l)..:....(1_+...:...I)
_?
So, the time spent on the highways is . true;
.,· 10 2 -
   
b.
Total time taken for the trip is 4 hrs.
150 130 51. 3n + 5 is even.
-+---4 .  
x x IO
Solve for x. SOLUTION:  
150(.\'- IO)+ I 30x = 4x(.,· -I 0) Substitute n = 1 in the expression and simplify.
150x - I 500 + I 30x - 4x: - 40x  
4.,.: - 320.,· + I 500 =0 311 + 5 = 3(1) + 5
=8
.\' = 5 or 75
 
So, the speed on the interstate is 75 mph.
Since 8 is an even number, given statement is true
 
for n = 1.
Thus, the speed on the highway is 75 – 10 = 65 mph.
 
 
ANSWER:  
ANSWER:  
150 130 true; 3(1) + 5 = 8, which is even
a. -·--  
x 'x 10
  2
150 130 52. n – 1 is odd.
b. -+---4 ; 75 mph, 65 mph  
x x 10
  SOLUTION:  
Substitute n = 1 in the expression and simplify.
State whether each statement is true or false
eSolutions Manual - Powered by Cognero   Page 17
when n = 1. Explain.
 
 
ANSWER:  
10-6true;
The3(1) + 5 = 8,Theorem
Binomial which is even
 
2
52. n – 1 is odd.
 
SOLUTION:  
Substitute n = 1 in the expression and simplify.
 
,,2_l=(t)i_,
=0
 
Since 0 is not an odd number, given statement is false
for n = 1.
 
ANSWER:  
2
false; 1 – 1 = 0, which is not odd
 

eSolutions Manual - Powered by Cognero Page 18


10-7 Proof by Mathematical Induction Prove that each statement is true for all natural
numbers.
 
Prove that each statement is true for all natural 2
1. 1 + 3 + 5 + … + (2n – 1) = n
numbers.  
 
1. 1 + 3 + 5 + … + (2n – 1) = n
2 SOLUTION:  
  Step 1: When n = 1, the left side of the given
2
equation is 1. The right side is 1 or 1, so the equation
SOLUTION:   is true for n = 1.
Step 1: When n = 1, the left side of the given  
2 2
equation is 1. The right side is 1 or 1, so the equation Step 2: Assume that 1 + 3 + 5 + . . . + (2k – 1) = k
is true for n = 1. for some natural number k.
   
2
Step 2: Assume that 1 + 3 + 5 + . . . + (2k – 1) = k Step 3: 1 + 3 + 5 + . . . + (2k – 1) + (2(k + 1) – 1)
for some natural number k.  
  2
= k + (2(k + 1) – 1)
Step 3: 1 + 3 + 5 + . . . + (2k – 1) + (2(k + 1) – 1) 2
  = k + (2k + 2 – 1)
2
2 = k + 2k + 1
= k + (2(k + 1) – 1) 2
2 = (k + 1)
= k + (2k + 2 – 1)
2  
= k + 2k + 1 The last expression is the right side of the equation to
2
= (k + 1) be proved, where n = k + 1.
  Thus, the equation is true for n = k + 1.
The last expression is the right side of the equation to  
be proved, where n = k + 1. 2
Therefore, 1 + 3 + 5 + . . . + (2n – 1) = n for all
Thus, the equation is true for n = k + 1. natural numbers n.
   
2
Therefore, 1 + 3 + 5 + . . . + (2n – 1) = n for all
natural numbers n. ANSWER:  
  Step 1: When n = 1, the left side of the given
2
equation is 1. The right side is 1 or 1, so the equation
ANSWER:   is true for n = 1.
Step 1: When n = 1, the left side of the given  
2 2
equation is 1. The right side is 1 or 1, so the equation Step 2: Assume that 1 + 3 + 5 + . . . + (2k – 1) = k
is true for n = 1. for some natural number k.
   
2
Step 2: Assume that 1 + 3 + 5 + . . . + (2k – 1) = k Step 3: 1 + 3 + 5 + . . . + (2k – 1) + (2(k + 1) – 1)
for some natural number k.  
  2
= k + (2(k + 1) – 1)
Step 3: 1 + 3 + 5 + . . . + (2k – 1) + (2(k + 1) – 1) 2
  = k + (2k + 2 – 1)
2
2 = k + 2k + 1
= k + (2(k + 1) – 1) 2
2 = (k + 1)
= k + (2k + 2 – 1)
2  
= k + 2k + 1 The last expression is the right side of the equation to
2
= (k + 1) be proved, where n = k + 1. Thus, the equation is
  true for n = k + 1. Therefore, 1 + 3 + 5 + . . . + (2n
The last expression is the right side of the equation to 2
– 1) = n for all natural numbers n.
be proved, where n = k + 1. Thus, the equation is  
true for n = k + 1. Therefore, 1 + 3 + 5 + . . . + (2n
eSolutions 2 - Powered by Cognero
– 1)Manual
= n for all natural numbers n. Page 1
  2. 
The last expression is the right side of the equation to
be proved, where n = k + 1. Thus, the equation is Step 2: Assume that  for 
true for n = k + 1. Therefore, 1 + 3 + 5 + . . . + (2n some natural number k.
2
10-7–Proof
1) = nbyfor all natural numbers
Mathematical n.
Induction  
  Step 3: 1 + 2 + 3 + . . . + k + (k + 1)
 
11(11+1) - k(k+l) +(k+I)
2.  1+2+3+ ... +11 2
2
  k(k + 1)+ 2(k + 1)
=
2
SOLUTION:   (k+l)(k+2)
Step 1: When n = 1, the left side of the given = ")
1(1 + I}  
equation is 1. The right side is  or 1, so the
2 The last expression is the right side of the equation to
equation is true for n = 1. be proved, where n = k + 1. Thus, the equation is
  true for n = k + 1. Therefore,
k(k + I} 11(11+1)
Step 2: Assume that 1+2+3+ ... +k=......;..- . . . .  for  I +2+3+ ... +11---'----  for all natural numbers n.
2 2
some natural number k.  
 
Step 3: 1 + 2 + 3 + . . . + k + (k + 1) 3. NUMBER THEORY A number is triangular if it
  can be represented visually by a triangular array.
 
- k(k+l) +(k+I)
2

• •
=
k(k + I}+ 2(k + 1) • • •
2 • •10 • •
(k+l)(k+2)  
=
2 a. The first triangular number is 1. Find the next 5
  triangular numbers.
The last expression is the right side of the equation to  
be proved, where n = k + 1. Thus, the equation is b. Write a formula for the nth triangular number.
true for n = k + 1. Therefore,  
11(11+1) c. Prove that the sum of the first n triangular
1+2+3+ ... +11  for all natural numbers n.
2 11(11+ 1)(11 + 2)
numbers equals .
  6
 
ANSWER:  
Step 1: When n = 1, the left side of the given SOLUTION:  
1(1 + 1) a.
equation is 1. The right side is  or 1, so the
2 •
••
equation is true for n = 1. •••
  ••••
•••••
Step 2: Assume that 1+2+3+
k(k + 1)
... +k=---'-- . . . .  for  ••••••
• ••••••
2
some natural number k.  
  The next 5 triangular numbers are 3, 6, 10, 15, 21.
Step 3: 1 + 2 + 3 + . . . + k + (k + 1)  
  11(11+1)
b. Cl -
" 2
 
c. Step 1: When n = 1, the left side of the given
1(1 + I}
equation is  or 1. The right side is 
2
eSolutions Manual - Powered by Cognero Page 2
 or 1, so the equation is true for n = 1.
 
The last expression is the right side of the equation to  
b.  for some 
  natural number k.
c. Step 1: When n = 1, the left side of the given  
10-7 Proof by Mathematical Induction
equation is  or 1. The right side is  Step 3:
k(k + l)(k + 2) (k + l)(k + 1 + 1)
1(1 + 1)(1 + 2) = 6 + 2
 or 1, so the equation is true for n = 1.
6
_ k(k + l)(k + 2) 3(k + l)(k + 1 + 1)
  - 6 + 6
Step 2: Assume that (k + l)(k + 2)(k + 3)
1+3+6+ ... + k(k+l)_k(k+l)(k+2)  for some  6
2 6 = (k+l)((k+l)+l)[(k+1)+2]
natural number k. 6
   
  The last expression is the right side of the equation to
be proved, where n = k + 1. Thus, the equation is
Step 3: I +J+6+ ... + k(k+ 1) + (k+ l)(k+ I+ 1) true for n = k + 1. Therefore,
2 2
_ k(k + l)(k + 2) (k + l)(k + 1 + 1) 11(11 + 1) _ 11(11 + 1)(11 + 2)
- 6 + 2
I + 3 + 6 + ... +--------  for all 
2 2
_ k(k + l)(k + 2) 3(k + l)(k + 1 + 1) natural numbers n.
- 6 + 6  
(k + l)(k + 2)(k + 3)
6 Prove that each statement is true for all natural
= (k+l)((k+l)+l)[(k+1)+2] numbers.
6  
  n
4. 10 – 1 is divisible by 9.
The last expression is the right side of the equation to
 
be proved, where n = k + 1. Thus, the equation is
true for n = k + 1. Therefore, SOLUTION:  
11(11+l)_11(11+1)(n+2) 1
I + 3 + 6 + ... + ---------  for all  Step 1: 10 – 1 = 9, which is divisible by 9. The
natural numbers n.
-
'} '}
. statement is true for n = 1.
 
  k
Step 2: Assume 10 – 1 is divisible by 9 for some
k
ANSWER:   natural number k. This means that 10 – 1 = 9r for
a. 3, 6, 10, 15, 21 some whole number r.
   
11(11+1) Step 3:
b. a,, - 1ot - I -9r
2
  10'-9r+I
c. Step 1: When n = 1, the left side of the given I 01•1 = 90r + 10
1(1 + 1) IO' .. , - I = 90r + 9
equation is  or 1. The right side is 
2
I 01•1 - I =9(10r + 1)
1(1 + 1)(1 + 2)
 or 1, so the equation is true for n = 1.  
6
Since r is a whole number, 10r + 1 is a whole
  k +1
Step 2: Assume that number. Thus, 10 – 1 is divisible by 9, so the
n
1+3+6+ ... + k(k+l) _ k(k+l)(k+2)  for some  statement is true for n = k + 1. Therefore, 10 – 1 is
2 6 divisible by 9 for all natural numbers n.
natural number k.  
 
ANSWER:  
I .., k(k+I) k(k+l){k+2)
Step 3: +., + 6 + ... + = ------------- 1
2 6 Step 1: 10 – 1 = 9, which is divisible by 9. The
statement is true for n = 1.
 
eSolutions Manual - Powered by Cognero k Page 3
Step 2: Assume 10 – 1 is divisible by 9 for some
k
natural number k. This means that 10 – 1 = 9r for
some whole number r.
  for all natural numbers n.
 
ANSWER:  
1 ANSWER:  
10-7Step 1: 10
Proof – 1 = 9, which is
by Mathematical divisible by 9. The
Induction 1
statement is true for n = 1. Step 1: 4 – 1 = 3, which is divisible by 3. The
  statement is true for n = 1.
k
Step 2: Assume 10 – 1 is divisible by 9 for some  
k
k
natural number k. This means that 10 – 1 = 9r for Step 2: Assume that 4 – 1 is divisible by 3 for some
k
some whole number r. natural number k. This means that 4 – 1 = 3r for
  some whole number r.
Step 3:  
1ot - I -9r Step 3:
10' -9r+ I 44 - 3r
I=
41 -3r+ I
I 01•1 = 90r + 10
.,11•'=12r+4
I 01 • 1 - I = 90r + 9
41•'-1=12r+3
I 01 ' - I = 9 ( I Or+ I)
  4'•1 1=3(-tr +-1)
Since r is a whole number, 10r + 1 is a whole  
k +1 Since r is a whole number, 4r + 1 is a whole number.
number. Thus, 10 – 1 is divisible by 9, so the
k +1
n
statement is true for n = k + 1. Therefore, 10 – 1 is Thus, 4 – 1 is divisible by 3, so the statement is
n
divisible by 9 for all natural numbers n. true for n = k + 1. Therefore, 4 – 1 is divisible by 3
  for all natural numbers n.
 
n
5. 4 – 1 is divisible by 3. Find a counterexample to disprove each
  statement.
 
SOLUTION:   n
1 6. 3 + 1 is divisible by 4.
Step 1: 4 – 1 = 3, which is divisible by 3. The  
statement is true for n = 1.
  SOLUTION:  
k Test different values of n.
Step 2: Assume that 4 – 1 is divisible by 3 for some
k  
natural number k. This means that 4 – 1 = 3r for
some whole number r. II 3" + I Divisible by 4?
  I 31 +I Yes
Step 3: 2 3: + I No
44 - I -3r
 
41 -3r + I The value n = 2 is a counterexample for the
4'•1 = 12r +4 statement.
 
41•1 - +3
I= 12r
41•1 -I =3(-tr + 1) ANSWER:  
  n =2
Since r is a whole number, 4r + 1 is a whole number.  
k +1
Thus, 4 – 1 is divisible by 3, so the statement is
n
true for n = k + 1. Therefore, 4 – 1 is divisible by 3 7. 2n + 3n is divisible by 4.
for all natural numbers n.  
  SOLUTION:  
ANSWER:   Test different values of n.
1  
Step 1: 4 – 1 = 3, which is divisible by 3. The
statement is true for n = 1.
  Manual - Powered by Cognero
eSolutions Page 4
k  
Step 2: Assume that 4 – 1 is divisible by 3 for some
natural number k. This means that 4 – 1 = 3r for
k The value n = 1 is a counterexample for the
ANSWER:    
n =2 The last expression is the right side of the equation to
10-7 Proof by Mathematical Induction
be proved, where n = k + 1. Thus, the equation is
true for n = k + 1. Therefore,
7. 2n + 3n is divisible by 4. I I I I I
-+-+-+ ~--1--
  2 2z 2' ··· 2" 2"  for all natural numbers
SOLUTION:   n.
Test different values of n.  
 
ANSWER:  
11 2n -1 311 Divisible b) 4?
Step 1: When n = 1, the left side of the given
2(1)+3{1) 0

  equation is . The right side is 1 _ _!_ or  , so the 


2 2 2
The value n = 1 is a counterexample for the
statement. equation is true for n = 1.
   
Step 2: Assume that
ANSWER:   I I I I 1
-+-+-+ +--1--
n =1 2 2= 2' ... 2t 21  for some natural 
 
number k.
CCSS ARGUMENTS  Prove that each  
statement is true for all natural numbers. I I I 1 I
  Step 3: -+--.-+~+ ... +-t +-.-,
2 2· 2· 2 2· •
1 I I I 1 I I
8.  -+-+-+ ... +-=1-- =l--+-
2 22 2J 2n 2" 2t 2t•I
  2 1
;;;I--T-
SOLUTION:  
2hl 2'·'
Step 1: When n = 1, the left side of the given =1-- ., ..I . ,
equation is . The right side is or  , so the   
2
The last expression is the right side of the equation to
equation is true for n = 1.
be proved, where n = k + 1. Thus, the equation is
 
true for n = k + 1. Therefore,
Step 2: Assume that
I I I I I
I I I I 1 -+-+~+ ... ~--1--  for all natural numbers
-+-+-+ +--1--
2 2= 2' ... 2t 21  for some natural  2 2: 2' 2" 2"
n.
number k.
   
I I I 1 I
Step 3: -+--.-+~+ ... +-t+-,-, (Ju• J)
2 2· 2. 2 2 • II
9.  2+5+8+ ... +(311-I)- 2
I I
=l--+-
2t z':'  
SOLUTION:  
Step 1: When n = 1, the left side of the given
1
=1--
2 ...'
equation is 2. The right side is  or 2, so 
  2
The last expression is the right side of the equation to the equation is true for n = 1.
be proved, where n = k + 1. Thus, the equation is  
true for n = k + 1. Therefore, Step 2: Assume that
eSolutions Manual - Powered by Cognero Page 5
 for all natural numbers   for some natural 

n. number k.
equation is 2. The right side is  or 2, so   for some natural 

the equation is true for n = 1. number k.


10-7 Proof by Mathematical Induction  
Step 2: Assume that Step 3:
k(3k + 1)
2+5+8+ ... •(3k-1)=
2
  for some natural  = k(3k2 + 1) + [ 3(k+l)-1 ]
number k. k(3k + 1)+ 2[3(k + 1)-1]
 
=------------
  3k1 +k-t-6k+6-2
=-----
Step 3: 2
2 + 5 + 8 + ... + ( 3k - 1) + [ 3 ( k + I) - 1] 3e +1k 1-4
=---
2
= k(3k + I) +[3(k+1)-1] (k+1)(3k+4)
2 = 2
= k(3k+1)+2[3(k+1)-1]
_ (k + 1)[3(k + 1)]+ 1
2 ..,
3k: +k+6k+6-2
=-----  
2
The last expression is the right side of the equation to
3k: + 7k 1-4
=--- be proved, where n = k + 1. Thus, the equation is
2
(k+1)(3k+4) true for n = k + 1. Therefore,
=-------- 2 2 +5+8+ ... +(311-1)-------
11(311-..-1)
 for all natural 
= (k + 1)[3(k + 1)]+ I 2
numbers n.
2
   
The last expression is the right side of the equation to
10.  I+ 2 + 4 + ... + 2"
1
= 2" - I
be proved, where n = k + 1. Thus, the equation is  
true for n = k + 1. Therefore,
SOLUTION:  
11(311 ... 1)
2 + 5 + 8-t- •.. +(311- I)- -------  for all natural  Step 1: When n = 1, the left side of the given
2
1
numbers n. equation is 1. The right side is 2 – 1 or 1, so the
  equation is true for n = 1.
 
ANSWER:   k –1 k
Step 2: Assume that 1 + 2 + 4 + . . . + 2 =2 – 1
Step 1: When n = 1, the left side of the given for some natural number k.
 
equation is 2. The right side is  or 2, so 
Step 3:
the equation is true for n = 1. I + 2 + 4 + ... + 2•-I + 2'-1•1
  :;;; 2' - I+ 2i-1+1
Step 2: Assume that
= 2' + 2· -I
k(3k + 1)
2 + 5 + 8+ ... •(Jk-1)---------  for some natural  -2(i)-l
2
number k.
   
Step 3: 2 + 5 + 8 + ... + ( 3k - I) + [ 3 ( k + I) - I] The last expression is the right side of the equation to
be proved, where n = k + 1. Thus, the equation is
n–
true for n = k + 1. Therefore, 1 + 2 + 4 + . . . + 2
1 n
eSolutions Manual - Powered by Cognero = 2 – 1 for all natural numbers n. Page 6
 
= 2k – k + 4k + 4 – 3
  2
= 2k + 3k + 1
The last expression is the right side of the equation to = (k + 1)(2k + 1)
10-7be
Proof by Mathematical
proved, where n = k + 1. Induction
Thus, the equation is = (k + 1)[2(k + 1) – 1]
n–  
true for n = k + 1. Therefore, 1 + 2 + 4 + . . . + 2
1 n The last expression is the right side of the equation to
= 2 – 1 for all natural numbers n.
  be proved, where n = k + 1. Thus, the equation is
ANSWER:   true for n = k + 1. Therefore, 1 + 5 + 9 + . . . + (4n
Step 1: When n = 1, the left side of the given – 3) = n (2n – 1) for all natural numbers n.
1  
equation is 1. The right side is 2 – 1 or 1, so the
equation is true for n = 1. ANSWER:  
  Step 1: When n = 1, the left side of the given
k –1 k
Step 2: Assume that 1 + 2 + 4 + . . . + 2 =2 – 1 equation is 1. The right side is 1[2(1) – 1] or 1, so the
for some natural number k. equation is true for n = 1.
   
Step 3: Step 2: Assume that 1 + 5 + 9 + . . . + (4k – 3) = k
I +2+4+ ... +2,\-1 +2l-l+I (2k – 1) for some natural number k.
- 2' - I + 2'-1+1  
""2' + 2' - I Step 3: 1 + 5 + 9 + . . . + (4k – 3) + [4 (k + 1) – 3]
= k (2k – 1) + [4(k + 1) – 3]
2
= 2k – k + 4k + 4 – 3
2
  = 2k + 3k + 1
The last expression is the right side of the equation to = (k + 1)(2k + 1)
be proved, where n = k + 1. Thus, the equation is = (k + 1)[2(k + 1) – 1]
true for n = k + 1. Therefore, 1 + 2 + 4 + . . . + 2
n–  
1 n The last expression is the right side of the equation to
= 2 – 1 for all natural numbers n.
  be proved, where n = k + 1. Thus, the equation is
true for n = k + 1. Therefore, 1 + 5 + 9 + . . . + (4n
11.  I+ 5 + 9 + ... + ( 411-3) ""u(211 - I) – 3) = n (2n – 1) for all natural numbers n.
   
SOLUTION:  
II (311- J)
Step 1: When n = 1, the left side of the given 12.  I + 4 + 7 + ... + (311 - 2) =
2
equation is 1. The right side is 1[2(1) – 1] or 1, so the  
equation is true for n = 1.
SOLUTION:  
 
Step 1: When n = 1, the left side of the given
Step 2: Assume that 1 + 5 + 9 + . . . + (4k – 3) = k
(2k – 1) for some natural number k. 1[3(1)+1]
equation is 1. The right side is  or 1, so 
  2
Step 3: 1 + 5 + 9 + . . . + (4k – 3) + [4 (k + 1) – 3] the equation is true for n = 1.
 
= k (2k – 1) + [4(k + 1) – 3]
2 Step 2: Assume that
= 2k – k + 4k + 4 – 3
2 ._ ., k(3k-1)
= 2k + 3k + 1 I + 4 + 7 + ... + ( .,k - - ) - --'--..a. for some natural
2
= (k + 1)(2k + 1) number k.
= (k + 1)[2(k + 1) – 1]  
  Step 3: 1 + 4 + 7 + . . . + (3k – 2) + [3(k + 1) – 2]
The Manual
eSolutions last expression is Cognero
- Powered by the right side of the equation to Page 7

be proved, where n = k + 1. Thus, the equation is


true for n = k + 1. Therefore, 1 + 5 + 9 + . . . + (4n
Step 2: Assume that
for some natural
for some natural
number k.
10-7number
Proof by
k. Mathematical Induction  
  Step 3: 1 + 4 + 7 + . . . + (3k – 2) + [3(k + 1) – 2]
Step 3: 1 + 4 + 7 + . . . + (3k – 2) + [3(k + 1) – 2] _k(Jk-1)
- +[J(k+l)-2]
_k(Jk-1) 2
- +[3(k+l)-2]
2 k(Jk-1)
k{Jk-1)
= +3k+ I
2
= .., + 3k + I _k(3k-1) 6k+2
- +--
= k(3k-1) + 6k+2 2 2
2 2 _ 3k:. + Sk + 2
_ 3k: + Sk t-2 2
2 =(k+1)(3k+2)
=(k+1)(3k+2) 2
2 = (k+1)[3(k+1)]-1
= (k+1)[3(k+1)]-1 2
2  
  The last expression is the right side of the equation to
The last expression is the right side of the equation to be proved, where n = k + 1. Thus, the equation is
be proved, where n = k + 1. Thus, the equation is true for n = k + 1. Therefore,
true for n = k + 1. Therefore,
1+4+7+ ... +(311-2)- "(311-l)  for all natural 
I + 4 + 7 + ... + ( 311 _ 2) _ II ( 311 - I)  for all natural  2
2 numbers n.
numbers n.  
 
13.  3 + 7 + I I + ... + ( 411 - I) = 211~ + 11
ANSWER:    
Step 1: When n = 1, the left side of the given
SOLUTION:  
equation is 1. The right side is  or 1, so  Step 1: When n = 1, the left side of the given
2
the equation is true for n = 1. equation is 4(1) – 1 or 3. The right side is 2(1) + 1
  or 3, so the equation is true for n = 1.
Step 2: Assume that  
Step 2: Assume that 3 + 7 + 11 + . . . + (4k – 1) =
1+4+7+ ... +(3k-2)- k(3k-1) for some natural 2
2 2k + k for some natural number k.
number k.  
  Step 3: 3 + 7 + 11 + . . . + (4k – 1) + [4(k + 1) – 1]
2
Step 3: 1 + 4 + 7 + . . . + (3k – 2) + [3(k + 1) – 2] = 2k + k + [4(k + 1) – 1]
2
= 2k + k + 4k + 3
2
= 2k + 5k + 3
2
= 2k + 4k+2+k+1
2
= [2(k + 1) ] + (k + 1)
 
The last expression is the right side of the equation to
be proved, where n = k + 1. Thus, the equation is
true for n = k + 1. Therefore, 3 + 7 + 11 + . . . + (4n
2
eSolutions Manual - Powered by Cognero – 1) = 2n + n for all natural numbers n. Page 8
 
ANSWER:  
= [2(k + 1) ] + (k + 1)
for some natural
 
The last expression is the right side of the equation to number k.
10-7be
Proof by Mathematical
proved, where n = k + 1. Induction
Thus, the equation is  
true for n = k + 1. Therefore, 3 + 7 + 11 + . . . + (4n Step 3:
2 I I I I I
– 1) = 2n + n for all natural numbers n. 2+6 + 12 + ... + k(k + I) + (k + l)(k + I+ I)
  k 1
= (k + 1) + (k + l)(k + I+ I)
ANSWER:   k(k +2) 1
(k + l)(k + 2) + (k + l)(k + I+ 1)
Step 1: When n = 1, the left side of the given
2 k2+2k+1
equation is 4(1) – 1 or 3. The right side is 2(1) + 1 (k + l)(k +2)
or 3, so the equation is true for n = 1. (k + l)(k + 1)
(k + l)(k +2)
 
(k + 1)
Step 2: Assume that 3 + 7 + 11 + . . . + (4k – 1) = (k +I)+ I
2
2k + k for some natural number k.  
   
Step 3: 3 + 7 + 11 + . . . + (4k – 1) + [4(k + 1) – 1] The last expression is the right side of the equation to
2 be proved, where n = k + 1. Thus, the equation is
= 2k + k + [4(k + 1) – 1]
2 true for n = k + 1. Therefore,
= 2k + k + 4k + 3
2 I I I I II
= 2k + 5k + 3 -+-·-· .... =--  for all natural 
2 6 12 ... 11(11+1) 11+1
2
= 2k + 4k+2+k+1
2 numbers n.
= [2(k + 1) ] + (k + 1)
   
The last expression is the right side of the equation to ANSWER:  
be proved, where n = k + 1. Thus, the equation is Step 1: When n = 1, the left side of the given
true for n = k + 1. Therefore, 3 + 7 + 11 + . . . + (4n l I
equation is or -  The right side is
2 1(1 + 1) 2
– 1) = 2n + n for all natural numbers n.
  I I
-or- so the equation is true for n = 1.
11- I 2
I I I I II
 
14.  -..&.-+-+....... =--
2 6 12 11(11+1) n+I Step 2: Assume that
  I I I I k
-+-+-+....... =--  for some natural
SOLUTION:   2 6 12 k(k+I) k+I
Step 1: When n = 1, the left side of the given number k.
I I  
equation is --or- . The right side is
1(1 + 1) 2 Step 3:
I 1 I 1 I
I I 2+6+12+ .. + k(k+l) + (k+l)(k+l+l)
-or- , so the equation is true for n = 1.
1+1 2 k 1
= (k + 1) + (k + l)(k + I+ I)
 
k(k +2) I
Step 2: Assume that (k + l)(k + 2) + (k + l)(k + I+ I)
I I I I k k2+2k+1
-+-+-+....... =-- for some natural
2 6 12 k(k+I) k+I (k + l)(k +2)
(k + l)(k +1)
number k. (k + l)(k +2)
  (k + I)
(k + I) +1
Step 3:
 
The last expression is the right side of the equation to
eSolutions Manual - Powered by Cognero be proved, where n = k + 1. Thus, the equation isPage 9
true for n = k + 1. Therefore,
 
The last expression is the right side of the equation to
10-7 Proof by Mathematical Induction be proved, where n = k + 1. Thus, the equation is
The last expression is the right side of the equation to true for n = k + 1. Therefore, 1 + 3 + 5 + . . . +
2 2 2

be proved, where n = k + 1. Thus, the equation is 2 11(211-1)(211 + 1)


(2n – 1) =  for all natural numbers
true for n = k + 1. Therefore, 3
n.
 for all natural 
 
numbers n. ANSWER:  
 
Step 1: When n = 1, the left side of the given
2
15.  12 +J2 +s-' + ... +(211-I) 2
= 11(211-1)(211 + 1) equation is 1 or 1. The right side is
3 1[2(1)-1][2(1)+ 1]
   or 1, so the equation is true for
3
SOLUTION:   n = 1.
Step 1: When n = 1, the left side of the given  
2 2 2 2 2
equation is 1 or 1. The right side is Step 2: Assume that 1 + 3 + 5 + . . . + (2k – 1)
k(2k-1)(2k + 1)
 or 1, so the equation is true for = 3  for some natural number k.

n = 1.  
2 2 2 2
  Step 3: 1 + 3 + 5 + . . . + (2k – 1) + [2(k + 1) –
2 2 2 2 2
Step 2: Assume that 1 + 3 + 5 + . . . + (2k – 1) 1]
k(2k-1)(2k + 1) _ k ( 2k - I) ( 2k + 1) + [ 2 ( k + I)- Ir
 for some natural number k. 3
3
  k(2k-1)(2k + 1)+3{2k + 1)2
2 2 2
Step 3: 1 + 3 + 5 + . . . + (2k – 1) + [2(k + 1) –
2 = 3
2 ( 2 k + I) [ k ( 2k - I) + 3 ( 2k + I)] .
1]
.,
_ k(2k-1)(2k + 1) +[2(k + 1)-lr .)

.,
.) (2k + 1)( 2k1 - k + 6k + 3)
k(2k-1)(2k + t)+3(2k + 1)2 = 3
= 3 (2k + t)(2k1 + Sk +3)
(2k + 1)[ k (2k-1) + 3(2k + t)]. = 3
= .,
.) (2k + l)(k + 1)(2k +3)
(2k + t)( 2k1 - k + 6k + 3) 3
3 (k+1)[2{k+l)-1][2(k+l)+1]
(2k + t)(2k1 + Sk +3) 3
= 3  
(2k + l)(k + 1)(2k +3) The last expression is the right side of the equation to
= ..
.) be proved, where n = k + 1. Thus, the equation is
2 2 2
(k + 1)[2(k + l)-1][2(k + 1) + 1] true for n = k + 1. Therefore, 1 + 3 + 5 + . . . +
=
3 2 11(211-1)(211 + 1)
(2n – 1) = ....  for all natural numbers
  .)

The last expression is the right side of the equation to n.


be proved, where n = k + 1. Thus, the equation is  
2 2 2
true for n = k + 1. Therefore, 1 + 3 + 5 + . . . + 16. GEOMETRY According to the Interior Angle Sum
2 Formula, if a convex polygon has n sides, then the
(2n –Manual
eSolutions 1) - Powered by Cognero  for all natural numbers Page 10
sum of the measures of the interior angles of a
n. polygon equals 180(n – 2). Prove this formula for
 using mathematical induction and geometry.
(2n – 1)  for all natural numbers polygon has 3 sides or 3 vertices, there are 180
degrees. The statement is true for n = 3.
n.
 
10-7 Proof by Mathematical Induction Step 2: Assume that the statement is true for .
16. GEOMETRY According to the Interior Angle Sum  
Formula, if a convex polygon has n sides, then the Step 3: Consider a convex polygon with n + 1
sum of the measures of the interior angles of a vertices. Since II+ ( <'!: 4 , if we take one vertex x
polygon equals 180(n – 2). Prove this formula for
11 <'!: 3  using mathematical induction and geometry.
there is another vertex y such that there is one vertex
  between x and y in one direction, and n – 2 in the
other direction. Join x and y by a new edge, dividing
our original polygon into two polygons. The new
polygons’ interior angles summed together make up
the sum of the original polygon’s interior angles. One
  of the new polygons is a triangle and the other a
SOLUTION:   polygon of n vertices (all but the one isolated
Step 1: When n = 3, 180(3 – 2) = 180. When a between x and y). The triangle has interior angle sum
polygon has 3 sides or 3 vertices, there are 180 180°, and by the inductive hypothesis the other 
degrees. The statement is true for n = 3. polygon has interior angle sum (11-2) · 180° .
  Summing these we get ( II + l - 2) · 180° , and the
Step 2: Assume that the statement is true for 11 <'!: 3.
theorem is proved.
   
Step 3: Consider a convex polygon with n + 1
vertices. Since II+ 1 <'!: 4 , if we take one vertex x Prove that each statement is true for all natural
numbers.
there is another vertex y such that there is one vertex
 
between x and y in one direction, and n – 2 in the n
17. 5 + 3 is divisible by 4.
other direction. Join x and y by a new edge, dividing  
our original polygon into two polygons. The new
SOLUTION:  
polygons’ interior angles summed together make up 1
Step 1: 5 + 3 = 8, which is divisible by 4. The
the sum of the original polygon’s interior angles. One statement is true for n = 1.
of the new polygons is a triangle and the other a  
k
polygon of n vertices (all but the one isolated Step 2: Assume 5 + 3 is divisible by 4 for some
k
between x and y). The triangle has interior angle sum natural number k. This means that 5 + 3 = 4r for
180°, and by the inductive hypothesis the other  some natural number r.
 
polygon has interior angle sum (11-2)·180 . Step 3:
Summing these we get (11+1-2)·180 , and the
theorem is proved.
 
ANSWER:  
Step 1: When n = 3, 180(3 – 2) = 180. When a
polygon has 3 sides or 3 vertices, there are 180  
degrees. The statement is true for n = 3. Since r is a natural number, 5r – 3 is a natural
k +1
  number. Thus, 5 + 3 is divisible by 4, so the
n
Step 2: Assume that the statement is true for . statement is true for n = k + 1. Therefore, 5 + 3 is
  divisible by 4 for all natural numbers n.
 
Step 3: Consider a convex polygon with n + 1
eSolutions Manual - Powered by Cognero Page 11
vertices. Since , if we take one vertex x ANSWER:  
1
there is another vertex y such that there is one vertex Step 1: 5 + 3 = 8, which is divisible by 4. The
theorem is proved. Since r is a natural number, 5r – 3 is a natural
k +1
  number. Thus, 5 + 3 is divisible by 4, so the
n
statement is true for n = k + 1. Therefore, 5 + 3 is
10-7Prove
Proof that each statement
by Mathematical is true for all natural
Induction divisible by 4 for all natural numbers n.
numbers.
 
 
n
17. 5 + 3 is divisible by 4. n
  18. 9 – 1 is divisible by 8.
 
SOLUTION:  
1 SOLUTION:  
Step 1: 5 + 3 = 8, which is divisible by 4. The 1
statement is true for n = 1. Step 1: 9 – 1 = 8, which is divisible by 8. The
  statement is true for n = 1.
k  
Step 2: Assume 5 + 3 is divisible by 4 for some k
k Step 2: Assume that 9 – 1 is divisible by 8 for some
natural number k. This means that 5 + 3 = 4r for k
some natural number r. natural number k. This means that 9 – 1 = 8r for
  some whole number r.
Step 3:  
5i +3=4r Step 3:
5A -4r-)
9• - I= 8r
9' -Sr+ 1
= 20r - IS
5'·1
9l•I : 72,- + 9
5t.• + 3 = 20r -12
9'·1 -I= 72r+8
5t·1+3=4{5,· 3)
9' 1 -1=8(9r+-1)
 
Since r is a natural number, 5r – 3 is a natural  
k +1 Since r is a whole number, 9r + 1 is a whole number.
number. Thus, 5 + 3 is divisible by 4, so the k +1
n Thus, 9 – 1 is divisible by 8, so the statement is
statement is true for n = k + 1. Therefore, 5 + 3 is n
divisible by 4 for all natural numbers n. true for n = k + 1. Therefore, 9 – 1 is divisible by 8
  for all natural numbers n.
 
ANSWER:  
1 ANSWER:  
Step 1: 5 + 3 = 8, which is divisible by 4. The 1
statement is true for n = 1. Step 1: 9 – 1 = 8, which is divisible by 8. The
  statement is true for n = 1.
k  
Step 2: Assume 5 + 3 is divisible by 4 for some k
k Step 2: Assume that 9 – 1 is divisible by 8 for some
natural number k. This means that 5 + 3 = 4r for k
some natural number r. natural number k. This means that 9 – 1 = 8r for
  some whole number r.
Step 3:  
5i + J - 4r Step 3:
5A a:a4r-) 9t - I= 8r

5t.1 = :Wr - I S 9' -Sr+ 1


9l•I : 72,- + 9
5'·1 • 3 = 20r - 12
9'•1 -I= 72r+8
5t1+3=4{5r 3)
1
  9' -I =8{9r t I)
Since r is a natural number, 5r – 3 is a natural  
number. Thus, 5
k +1
+ 3 is divisible by 4, so the Since r is a whole number, 9r + 1 is a whole number.
n k +1
statement is true for n = k + 1. Therefore, 5 + 3 is Thus, 9 – 1 is divisible by 8, so the statement is
n
divisible by 4 for all natural numbers n. true for n = k + 1. Therefore, 9 – 1 is divisible by 8
  for all natural numbers n.
 
eSolutions Manual - Powered by Cognero Page 12
n
18. 9 – 1 is divisible by 8. n
19. 12 + 10 is divisible by 11.
   
 
Since r is a whole number, 9r + 1 is a whole number.
k +1
Thus, 9 – 1 is divisible by 8, so the statement is
10-7true
Proof n
forby
n =Mathematical Induction
k + 1. Therefore, 9 – 1 is divisible by 8  
for all natural numbers n.
Since r is a natural number, 12r – 10 is a natural
  k +1
number. Thus, 12 + 10 is divisible by 11, so the
n n
19. 12 + 10 is divisible by 11. statement is true for n = k + 1. Therefore, 12 + 10
  is divisible by 11 for all natural numbers n.
SOLUTION:    
1
Step 1: 12 + 10 = 22, which is divisible by 11. The n
20. 13 + 11 is divisible by 12.
statement is true for n = 1.  
 
k SOLUTION:  
Step 2: Assume that 12 + 10 is divisible by 11 for 1
k Step 1: 13 + 11 = 24, which is divisible by 12. The
some natural number k. This means that 12 + 10 =
statement is true for n = 1.
11r for some natural number r.
 
  k
Step 3: Step 2: Assume that 13 + 11 is divisible by 12 for
k
12' +IO - I Ir some natural number k. This means that 13 + 11 =
12r for some natural number r.
124 -1 lr-10
 
12 .. 1 = 132r- l 20
Step 3:
121•1 +IO= I 32r -110
134 + I 1 - I 2r
I 21 ' + I O = I I ( I 2r IO} 13' -12r -11
  IJl•I =156r-J4J
Since r is a natural number, 12r – 10 is a natural
k +1
I 31•1 + I 1 = I S6r - 132
number. Thus, 12 + 10 is divisible by 11, so the
n 13tt• .. 11=12(13r II}
statement is true for n = k + 1. Therefore, 12 + 10
 
is divisible by 11 for all natural numbers n.
Since r is a natural number, 13r – 11 is a natural
  k +1
number. Thus, 13 + 11 is divisible by 12, so the
ANSWER:   statement is true for n = k + 1. Therefore, 13 + 11
n
1
Step 1: 12 + 10 = 22, which is divisible by 11. The is divisible by 12 for all natural numbers n.
statement is true for n = 1.  
 
k ANSWER:  
Step 2: Assume that 12 + 10 is divisible by 11 for 1
k Step 1: 13 + 11 = 24, which is divisible by 12. The
some natural number k. This means that 12 + 10 =
statement is true for n = 1.
11r for some natural number r.
 
  k
Step 3: Step 2: Assume that 13 + 11 is divisible by 12 for
k
12t +IO - I Ir some natural number k. This means that 13 + 11 =
12r for some natural number r.
124 -1 lr-10
 
12 .. ' =132r-120
Step 3:
121•1 +10=132r-1 IO
1
121 +- IO= 11 ( I 2r IO)
 
Since r is a natural number, 12r – 10 is a natural
k +1
number.
eSolutions Thus,
Manual 12 by Cognero
- Powered + 10 is divisible by 11, so the Page 13
n
statement is true for n = k + 1. Therefore, 12 + 10
 
is divisible by 11 for all natural numbers n.
Step 2: Assume that 13 + 11 is divisible by 12 for  
k
some natural number k. This means that 13 + 11 = ANSWER:  
12r for some natural number r.
10-7 Proof by Mathematical Induction n =3
 
Step 3:
134 + 11 - I 2r 2
23. n – n + 15 is prime.
13' .::; I 2r - 11  
= I 56r-143
1J4•1 SOLUTION:  
131+• + 11 = I 56r - 132 When n = 1, the value of the expression is 15 which
is not a prime number, so the expression is not true
13t.a t I I = 12 ( I 3r 1 I)
for n = 1.
   
Since r is a natural number, 13r – 11 is a natural
k +1
ANSWER:  
number. Thus, 13 + 11 is divisible by 12, so the
n n =1
statement is true for n = k + 1. Therefore, 13 + 11  
is divisible by 12 for all natural numbers n.
  2
24. n + n + 23 is prime.
 
Find a counterexample to disprove each
statement. SOLUTION:  
  When n = 1, the value of the expression is 25 which
2 is not a prime number, so the expression is not true
21. 1 + 2 + 3 + … + n = n
  for n = 1.
 
SOLUTION:  
When n = 2, the left side of the given equation is 2. ANSWER:  
2 n =1
The right side is 2 or 4, so the equation is not true
for n = 2.  
 
25. NATURE The terms of the Fibonacci sequence are
ANSWER:   found in many places in nature. The number of
n =2 spirals of seeds in sunflowers is a Fibonacci number,
  as is the number of spirals of scales on a pinecone.
The Fibonacci sequence begins 1, 1, 2, 3, 5, 8, … 
Each element after the first two is found by adding
3 2
22. 1 + 8 + 27 + … + n = (2n + 2) the previous two terms. If f n stands for the nth
  Fibonacci number, prove that f 1 + f 2 + … + f n = f n +
SOLUTION:   2 – 1.
3
When n = 3, the left side of the given equation is 3  
2
or 27. The right side is 8 or 64, so the equation is not SOLUTION:  
true for n = 3.
Step 1: When n = 1, the left side of the given
 
equation is f 1. The right side is f 3 – 1. Since f 1 = 1
ANSWER:   and f 3 = 2 the equation becomes 1 = 2 – 1 and is true
n =3 for n = 1.
   
2 Step 2: Assume that f 1 + f 2 + … + f k = f k + 2 – 1 for
23. n – n + 15 is prime.
  some natural number k.
 
SOLUTION:  
Step 3: f 1 + f 2 + … + f k + f k + 1
When n = 1, the value of the expression is 15 which
is notManual
a prime number, so the expression is not true = f k + 2  – 1 + f k + 1
eSolutions - Powered by Cognero Page 14
for n = 1. =f k + 1 +f k + 2 – 1
  = f k + 3 – 1, since Fibonacci numbers are produced
  Step 1: 7 + 5 = 12, which is divisible by 6. The
Step 2: Assume that f 1 + f 2 + … + f k = f k + 2 – 1 for statement is true for n = 1.
some natural number k.  
10-7 Proof by Mathematical Induction k
Step 2: Assume that 7 + 5 is divisible by 6 for some
Step 3: f 1 + f 2 + … + f k + f k + 1 k
natural number k. This means that 7 + 5 = 6r for
= f k + 2  – 1 + f k + 1 some whole number r.
=f k + 1 +f k + 2 – 1  
= f k + 3 – 1, since Fibonacci numbers are produced Step 3:
by adding the two previous Fibonacci numbers. The 7' + 5 = 6r
last expression is the right side of the equation to be 7' = 6r-5
proved, where n = k + 1. Thus, the equation is true 7t.1 = 7 ( 6r - 5)
for n = k + 1. Therefore, f 1 + f 2 + … + f n = f n + 2  – 7'•1 =-12r-35
1 for all natural numbers n. 7'•1 + 5 = -12r 30
  71+1 +5-6{7r-5)
ANSWER:    
Since r is a natural number, 7r – 5 is a natural
Step 1: When n = 1, the left side of the given
k +1
equation is f 1. The right side is f 3 – 1. Since f 1 = 1 number. Thus, 7 + 5 is divisible by 6, so the
n
and f 3 = 2 the equation becomes 1 = 2 – 1 and is true statement is true for n = k + 1. Therefore, 7 + 5 is
divisible by 6 for all natural numbers n.
for n = 1.
   
Step 2: Assume that f 1 + f 2 + … + f k = f k + 2 – 1 for ANSWER:  
some natural number k. 1
Step 1: 7 + 5 = 12, which is divisible by 6. The
  statement is true for n = 1.
Step 3: f 1 + f 2 + … + f k + f k + 1  
= f k + 2  – 1 + f k + 1 k
Step 2: Assume that 7 + 5 is divisible by 6 for some
=f k + 1 +f k + 2 – 1 k
natural number k. This means that 7 + 5 = 6r for
= f k + 3 – 1, since Fibonacci numbers are produced some whole number r.
by adding the two previous Fibonacci numbers. The  
last expression is the right side of the equation to be Step 3:
proved, where n = k + 1. Thus, the equation is true 7' + 5 = 6r
for n = k + 1. Therefore, f 1 + f 2 + … + f n = f n + 2  – 7' =6r-5
1 for all natural numbers n. 7t.1 = 7 ( 6r - 5)
  7'-• = -12r-35

Prove that each statement is true for all natural 7l•I +5=-12r-)0
numbers or find a counterexample. 1'•1 +5-6{7r-5)
   
n
26. 7 + 5 is divisible by 6. Since r is a natural number, 7r – 5 is a natural
  k +1
number. Thus, 7 + 5 is divisible by 6, so the
SOLUTION:   statement is true for n = k + 1. Therefore, 7 + 5 is
n
1
Step 1: 7 + 5 = 12, which is divisible by 6. The divisible by 6 for all natural numbers n.
statement is true for n = 1.  
 
k n
Step 2: Assume that 7 + 5 is divisible by 6 for some 27. 18 – 1 is divisible by 17.
k
natural number k. This means that 7 + 5 = 6r for  
some whole number r. SOLUTION:  
eSolutions Manual - Powered by Cognero Page 15
  1
Step 1: 18 – 1 = 17, which is divisible by 17. The
Step 3:
statement is true for n = 1.
 
 
10-7 n
Proof
27. 18 – 1by Mathematical
is divisible by 17. Induction Since r is a natural number, 18r + 1 is a natural
  k +1
number. Thus, 18 – 1 is divisible by 17, so the
SOLUTION:   n
statement is true for n = k + 1. Therefore, 18 – 1 is
1
Step 1: 18 – 1 = 17, which is divisible by 17. The divisible by 17 for all natural numbers n.
statement is true for n = 1.  
 
2
k
Step 2: Assume that 18 – 1 is divisible by 17 for 28. n + 21n + 7 is a prime number.
k  
some natural number k. This means that 18 – 1 =
17r for some natural number r. SOLUTION:  
  When n = 6, the value of the expression is 169 which
is not a prime number, so the expression is not true
Step 3:
for n = 6.
181 -1 :ee: I 7r  
181 = I 7r + I
ANSWER:  
1st-• =18{17r+l)
n =6
I 81•1 "'306r + 18  
I 81 •1 - I = 306r + 17
2
I 81 •1 - I = I 7 ( I 8r + I ) 29. n + 3n + 3 is a prime number.
 
 
Since r is a natural number, 18r + 1 is a natural SOLUTION:  
k +1 When n = 3, the value of the expression is 21 which
number. Thus, 18 – 1 is divisible by 17, so the
n
is not a prime number, so the expression is not true
statement is true for n = k + 1. Therefore, 18 – 1 is for n = 3.
divisible by 17 for all natural numbers n.  
 
ANSWER:  
ANSWER:   n =3
1  
Step 1: 18 – 1 = 17, which is divisible by 17. The
statement is true for n = 1.
  30.  500 + 1 oo+ 20 + ... + 4. 51-"::; 625(1-_!_)
k
5"
Step 2: Assume that 18 – 1 is divisible by 17 for  
k
some natural number k. This means that 18 – 1 =
SOLUTION:  
17r for some natural number r.
  Step 1: When n = 1, the left side of the given
Step 3: equation is 4 · 5~ I  or 500. The right side is 
181 -1 :ee: I 7r
625( I- ~1)  or 500, so the equation is true for n = 1.
181 = I 7r + I
 
181•1 =18(17r+I)
Step 2: Assume that
I 8t.t = 306r + 18
500+100+20+ ... +4·5~-· =625(1-5'1)  for some 
I 81 •1 - I = 306r +. 17
1 81 •1 - I - I 7 (I s- + I ) natural number k.
   
Since r is a natural number, 18r + 1 is a natural Step 3:
k +1
number. Thus, 18 – 1 is divisible by 17, so the
n
statement
eSolutions Manual is
- Powered Therefore, 18 – 1 is
true forbynCognero
= k + 1. Page 16
divisible by 17 for all natural numbers n.
 
 for some  that    for 
natural number k. some natural number k.
10-7 Proof by Mathematical Induction  
Step 3: Step 3:
500+100+20+ ... +4·5"' l +4·5l-(t+I) 500 + 100 + 20+ ........... · 5"' '+4 .5•-(hl)

=625(1-
51,
)+4·5~ <'·11 =625(1-
51,
)+4·5~ <'·11

=625(5'5~')+4·5'' =625(5'5~1)+4·5''

5.t.'-5) 4.5• 5.l+l _5) 4.5•


= 625 ( 51+1 + 51+ = 625 ( 51+1 + 51+

- 5''(5t•I -5)+4·5• - 5''(5t+I -5)+4·5•


- 5' 1 - 5' 1
_ ?
-6 .. 5
( 5t+I -
-.hi
:>
I) _- 625 ($1+ 1 -
-hi
:>
I)
=62s(1--1-)
5h1
=62s(1--1-)
5h1

   
The last expression is the right side of the equation to The last expression is the right side of the equation to
be proved, where n = k + 1. Thus, the equation is be proved, where n = k + 1. Thus, the equation is
true for n = k + 1. Therefore, true for n = k + 1. Therefore,

500+ 100+20+ ... +4·5J-n =625(1- sin) 500 + 100 + 20 + ... + 4 · SJ-" = 625( I -
51n)
 for all 

for all natural numbers n. natural numbers n.


   

ANSWER:   _1_+_1_+_1_+
1·2·3 2·3·4 3.4.5 .
Step 1: When n = 1, the left side of the given 31. + 1 _ n(n+3)
equation is 4 · 5~ I or 500. The right side n(n+l)(n+2) - 4(n+l)(n+2)
 
is 625(1-;1) or 500, so the equation is true for n =
SOLUTION:  
1. Step 1: When n = 1, the left side of the given
  I 1
equation is ----or- . The right side is
Step 2: Assume 1(1+1)(1+2) 6
that   500 + 100 + 20 + ... + 4 · 5J-t = 625( 1 - 51, )  for  1(1 +3) 1
----or- , so the equation is true for n =
4(1+1)(1+2) 6
some natural number k.
1.
 
 
Step 3:
Step 2: Assume that

1.l.3 + 2·3·4 + 3.~.5 + ...


1 k(k + 3)  
+ k(k + l)(k + 2) 4(k + l)(k + 2)
for some natural number k.
 
Step 3:
eSolutions Manual - Powered by Cognero Page 17
natural number k.
   
10-7for
Proof bynatural
Mathematical Step 3:
some number k.Induction
 
 
Step 3: k(k+3) I
--,-----,,....,..-----,-+-~,----,-~~
_1_+_1_+_1_+ .. 4(k+1)(k+2) (k+l}(k+2)(k+3)
1·2·3 2·3·4 3·4·5
1 1 _ k ( k + 3) ( k + 3) + --,----,--,----4----,-,,-----~
+ k(k + l)(k + 2) + (k + l)(k + 2)(k + 3)
- 4(k+l}(k+2)(k+3) 4(k+l)(k+2)(k+3)
k(k +3) 1
= 4(k+l)(k+2) + (k+l)(k+2)(k+3) k' + 6k2 + 9k + 4
k(k + 3)(k + 3) 4 - -'(k+l)(k+2)(k+3)
= 4(k + l)(k + 2)(k + 3) + 4(k + l)(k + Z)(k + 3)
(k+l)(k2+5k+4)
k3+6k2+9k+4
4(k + l)(k + 2)(k + 3)
= 4(k + l)(k + 2)(k + 3)
_ (k+l){k+4)
(k + 1)(k2 + 5k +4)
= 4(k + l)(k + 2)(k + 3)
- 4(k+2)(k+3)
_ (k + l)(k +4) (k+l)[(k+1)+3]
- 4(k + 2)(k+ 3) = 4[(k+1)+1J[(k+l)+2]
(k + 1) [ (k + 1) + 3]
= 4[(k+l)+l)[(k+1)+2]  
  The last expression is the right side of the equation to
The last expression is the right side of the equation to be proved, where n = k + 1. Thus, the equation is
be proved, where n = k + 1. Thus, the equation is true for n = k + 1. Therefore,
true for n = k + 1. Therefore, _l_+_l_+_l_+ + 1
1·2·3 2·3·4 3·4·5 · n(n+l)(n+2)
  or all  n(n+3)
= 4(n +l)(n +2)

natural numbers n. for all natural numbers n.


   

ANSWER:   32. CCSS PERSEVERANCE  Refer to the figures


below.
Step 1: When n = 1, the left side of the given
 
I I
equation is  or  . The right side is
1(1-r 1)(1 + 2) 6
1 (I -t-3)
 or  , so the equation is true for n =
4(1+1)(1+2) 6
1.  
  a. There is a total of 5 squares in the second figure.
Step 2: Assume that How many squares are there in the third figure?
I I I I k(k+3}   for some   
1·2·3 + 2·3-4 + 3,4.5 + ... + k(k+l)(k+2} 4(k+l)(k+2)
b. Write a sequence for the first five figures.
natural number k.  
  c. How many squares are there in a standard
Step 3:  checkerboard?
  I I I I
 
-+--+
1·2·3 2·3·4 ... + k(k+l)(k+2} + (k+I )( k+2 )( k+3 ) d. Write a formula to represent the number of
squares in an   grid.
 
SOLUTION:  
a. 14
 
b. 1, 5, 14, 30, 55
eSolutions Manual - Powered by Cognero   Page 18

c. 204
 
 

33. CHALLENGE Suggest a formula to represent 2 +


4 + 6 + … + 2n, and prove your hypothesis using
10-7for all natural
Proof numbers n. Induction
by Mathematical mathematical induction.
 
 
32. CCSS PERSEVERANCE  Refer to the figures SOLUTION:  
below.
Formula to represent 2 + 4 + 6 + … + 2n:
 
2 + 4 + 6 + … + 2n = 2(1 + 2 + 3 + … + n)

Figure 1
Figure 2
Figure 3
= n(n + 1)
   
a. There is a total of 5 squares in the second figure. Step 1: When n = 1, the left side of the given
How many squares are there in the third figure? equation is 2(1) or 2. The right side is 1(1 + 1) or 2,
  so the equation is true for n = 1.
b. Write a sequence for the first five figures.
 
 
Step 2: Assume that 2 + 4 + 6 + . . . + 2k = k (k + 1)
c. How
8x8 many squares are there in a standard
 checkerboard? for some natural number k.
   
d. Write a formula
II XIIto represent the number of Step 3: 2 + 4 + 6 + . . . + 2k + 2(k + 1)
squares in an   grid. = k(k + 1) + 2(k + 1)
 
= (k + 1)(k + 2)
SOLUTION:   = (k + 1)[(k + 1) + 1]
a. 14  
  The last expression is the right side of the equation to
b. 1, 5, 14, 30, 55 be proved, where n = k + 1. Thus, the equation is
  2
true for n = k + 1. Therefore, 2 + 4 + 6 + . . . + n =
c. 204 n (n + 1) for all natural numbers n.
  II ( II + I) ( 211 + I)  
d. 6
ANSWER:  
  n(n + 1)
 
ANSWER:  
Step 1: When n = 1, the left side of the given
a. 14
equation is 2(1) or 2. The right side is 1(1 + 1) or 2,
 
b. 1, 5, 14, 30, 55 so the equation is true for n = 1.
   
c. 204 Step 2: Assume that 2 + 4 + 6 + . . . + 2k = k (k + 1)
  for some natural number k.
11(11+ 1)(211+ 1)
 
d. 6
Step 3: 2 + 4 + 6 + . . . + 2k + 2(k + 1)
  = k(k + 1) + 2(k + 1)
= (k + 1)(k + 2)
33. CHALLENGE Suggest a formula to represent 2 +
4 + 6 + … + 2n, and prove your hypothesis using = (k + 1)[(k + 1) + 1]
mathematical induction.  
  The last expression is the right side of the equation to
eSolutions Manual - Powered by Cognero be proved, where n = k + 1. Thus, the equationPage
is 19
SOLUTION:  
2
Formula to represent 2 + 4 + 6 + … + 2n: true for n = k + 1. Therefore, 2 + 4 + 6 + . . . + n =
= k(k + 1) + 2(k + 1) false, but it is more difficult to prove that it is true.
= (k + 1)(k + 2) Statements need to be proven true by mathematical
= (k + 1)[(k + 1) + 1] induction, geometrically, or by another method.
10-7 Proof by Mathematical Induction  
The last expression is the right side of the equation to
be proved, where n = k + 1. Thus, the equation is 35. If a statement is true for n = k and n = k + 1, then it
2 is also true for n = 1.
true for n = k + 1. Therefore, 2 + 4 + 6 + . . . + n =  
n (n + 1) for all natural numbers n.
  SOLUTION:  
Sample answer: False; assume k = 2, just because a
REASONING Determine whether the following statement is true for n = 2 and n = 3 does not mean
statements are true or false . Explain.
  that it is true for n = 1.
34. If you cannot find a counterexample to a statement,  
then it is true.
  ANSWER:  
Sample answer: False; assume k = 2, just because a
SOLUTION:  
statement is true for n = 2 and n = 3 does not mean
Sample answer: False; Even if a counterexample
that it is true for n = 1.
cannot immediately be found, it does not mean that
 
one doesn’t exist. A statement can easily be proven
false, but it is more difficult to prove that it is true.
Statements need to be proven true by mathematical
36. CHALLENGE Prove Il' =(11(11+1))
1. I 2
1 .

induction, geometrically, or by another method.  


 
SOLUTION:  
ANSWER:   Step 1: When n = 1, the left side of the given
Sample answer: False; Even if a counterexample
3
cannot immediately be found, it does not mean that equation is1 or 1. The right side is or 1,
one doesn’t exist. A statement can easily be proven so the equation is true for n = 1.
false, but it is more difficult to prove that it is true.  
Statements need to be proven true by mathematical Step 2: Assume that
induction, geometrically, or by another method.
  1+8+27+ ... +k \ = (k(k + 1)):  for some natural 
2
number k.
35. If a statement is true for n = k and n = k + 1, then it
is also true for n = 1.  
 
SOLUTION:  
Step 3:
1+8+. +k3+(k+l)3=( k(k/1)

2 2
r +(k+l)3

Sample answer: False; assume k = 2, just because a k (k + 1) (k 1)3


4 + +
statement is true for n = 2 and n = 3 does not mean k2(k + 1)2 4(k + 1)3
that it is true for n = 1. 4 + 4

  Ck+ o2[k2 +4Ck + o]


4
ANSWER:   Ck+ o2(k2 +4k +4)
4
Sample answer: False; assume k = 2, just because a
(k + 1)2(k + 2)2
statement is true for n = 2 and n = 3 does not mean
that it is true for n = 1.
 
4

=( (k+l)[(k/1)+1)2 r
eSolutions Manual - Powered by Cognero   Page 20

The last expression is the right side of the equation to


36. CHALLENGE Prove .
be proved, where n = k + 1. Thus, the equation is
true for n = k + 1. Therefore,

 for all natural 
10-7 Proof by Mathematical Induction
numbers n.
 
 
The last expression is the right side of the equation to 3
37. REASONING Find a counterexample to x + 30 >
be proved, where n = k + 1. Thus, the equation is 2
x + 20x.
true for n = k + 1. Therefore,  

I+ 8 + 27 + ... + 11
1
= (11(11+ 1)):  for all natural  SOLUTION:  
Substitute x = 3 in the given inequality.
2
 
numbers n.
xl + 30 > x: + 20x
 
3' +30>3: +20(3}
ANSWER:   57 >69X
Step 1: When n = 1, the left side of the given  
So, the inequality is not true for x = 3.
3  
equation is1 or 1. The right side is or 1,

so the equation is true for n = 1. ANSWER:  


  x =3
Step 2: Assume that
 

I+ 8 + 27 + ... + k =
1 (k(k + ')):
 for some natural 
38. OPEN ENDED Write a sequence, the formula that
produces it, and determine the formula for the sum of
2
the terms of the sequence. Then prove the formula
number k. with mathematical induction.
   
Step 3: 1+8+ ... +k·'+(k+lf =(k(k/l)r +(k+ty1 SOLUTION:  

1+8+ +k3+(k+1)3=(k(k/1) r +(k+1)3


Sample answer: 6 + 10 + 14 + . . . . The sequence is
produced by a n = 4n + 2. The sum of the sequence is
2
k (k + I)
2
(k 1)3
represented by 2n(n + 2).
4 + +  
k2(k+1)2 4(k+1)3 Step 1: When n = 1, the left side of the given
4 + 4
equation is 4(1) + 2 or 6. The right side is 2(1)(1 + 2)
Ck+ o2[k 2 +4Ck + o] or 6, so the equation is true for n = 1.
4
 
(k+1)2(k2+4k+4) Step 2: Assume that 6 + 10 + 14 + . . . 4k + 2 = 2k(k
4 + 2) for some natural number k.
(k +1)2(k +2)2  
4

= ( (k + I)[ (k/ I) + I)
2
r Step 3: 6 + 10 + 14 + . . . 4k + 2 + 4(k + 1) + 2
= 2k(k + 2) + 4(k + 1) + 2
2
= 2k + 4k + 4k + 4 + 2
  2
The last expression is the right side of the equation to = 2k + 8k + 6
= 2(k + 1)(k + 3)
be proved, where n = k + 1. Thus, the equation is = 2(k + 1)[(k + 1) + 2]
true for n = k + 1. Therefore,  

I+ 8 + 27 + ...
, ( 11(11 +I)
+n =
2
l:  for all natural 
The last expression is the right side of the equation to
be proved, where n = k + 1. Thus, the equation is
true for n = k + 1. Therefore, 6 + 10 + 14 + . . . (4n
numbers n. + 2) = 2n(n + 2) for all natural numbers n.
 
 
eSolutions Manual - Powered by Cognero Page 21
3
ANSWER:  
37. REASONING Find a counterexample to x + 30 > Sample answer: 6 + 10 + 14 + . . . . The sequence is
2
x + 20x.
 
The last expression is the right side of the equation to
The last expression is the right side of the equation to
be proved, where n = k + 1. Thus, the equation is
be proved, where n = k + 1. Thus, the equation is
true for n = k + 1. Therefore, 6 + 10 + 14 + . . . (4n
true for n = k + 1. Therefore, 6 + 10 + 14 + . . . (4n
10-7 Proof by Mathematical Induction + 2) = 2n(n + 2) for all natural numbers n.
+ 2) = 2n(n + 2) for all natural numbers n.
 
 
ANSWER:   39. WRITING IN MATH Explain how the concept of
dominoes can help you understand the power of
Sample answer: 6 + 10 + 14 + . . . . The sequence is
mathematical induction.
produced by a n = 4n + 2. The sum of the sequence is  
represented by 2n(n + 2).
  SOLUTION:  
Step 1: When n = 1, the left side of the given Sample answer: When dominoes are set up, after the
equation is 4(1) + 2 or 6. The right side is 2(1)(1 + 2) first domino falls, the rest will fall as well. With
or 6, so the equation is true for n = 1.
  induction, once it is proved that the statement is true
Step 2: Assume that 6 + 10 + 14 + . . . 4k + 2 = 2k(k for n = 1 (the first domino), n = k (the second
+ 2) for some natural number k. domino), and n = k + 1 (the next domino), it will be
 
true for any integer value (any domino).
Step 3: 6 + 10 + 14 + . . . 4k + 2 + 4(k + 1) + 2
= 2k(k + 2) + 4(k + 1) + 2  
2
= 2k + 4k + 4k + 4 + 2 ANSWER:  
2
= 2k + 8k + 6 Sample answer: When dominoes are set up, after the
= 2(k + 1)(k + 3)
= 2(k + 1)[(k + 1) + 2] first domino falls, the rest will fall as well. With
  induction, once it is proved that the statement is true
The last expression is the right side of the equation to for n = 1 (the first domino), n = k (the second
be proved, where n = k + 1. Thus, the equation is
true for n = k + 1. Therefore, 6 + 10 + 14 + . . . (4n domino), and n = k + 1 (the next domino), it will be
+ 2) = 2n(n + 2) for all natural numbers n. true for any integer value (any domino).
   

39. WRITING IN MATH Explain how the concept of 40. WRITING IN MATH Provide a real-world
dominoes can help you understand the power of example other than dominoes that describes
mathematical induction. mathematical induction.
   
SOLUTION:   SOLUTION:  
Sample answer: When dominoes are set up, after the Sample answer: Climbing a ladder; each step leads to
first domino falls, the rest will fall as well. With the next step.
induction, once it is proved that the statement is true  
for n = 1 (the first domino), n = k (the second
ANSWER:  
domino), and n = k + 1 (the next domino), it will be
Sample answer: Climbing a ladder; each step leads to
true for any integer value (any domino).
the next step.
   
ANSWER:  
41. Which of the following is a counterexample to the
Sample answer: When dominoes are set up, after the statement below?
first domino falls, the rest will fall as well. With  
2
induction, once it is proved that the statement is true n + n – 11 is prime.
for n = 1 (the first domino), n = k (the second  
A n = -6
domino), and n = k + 1 (the next domino), it will be
 
true for any integer value (any domino). B n =4
  Manual - Powered by Cognero
eSolutions   Page 22
C n =5
40. WRITING IN MATH Provide a real-world  
example other than dominoes that describes
 
ANSWER:  
Sample answer: Climbing a ladder; each step leads to ANSWER:  
10-7the nextby
Proof step.
Mathematical Induction B
   

41. Which of the following is a counterexample to the 42. PROBABILITY Latisha wants to create a 7-


statement below? character password. She wants to use an
  arrangement of the first 3 letters of her first name
2 (lat), followed by an arrangement of the 4 digits in
n + n – 11 is prime.
  1986, the year she was born. How many possible
passwords can she create in this way?
A n = -6
 
 
F 72
B n =4
 
 
G 144
C n =5
 
 
H 288
D n =6
 
 
J 576
SOLUTION:    
When n = 4, the value of the expression is 9 which is
SOLUTION:  
not a prime number, so the expression is not true for
n = 4. Number of po sible pas words = 3!· 4 !
B is the correct option. =6·24
  = 144
 
ANSWER:  
G is the correct option.
B  
 
ANSWER:  
42. PROBABILITY Latisha wants to create a 7- G
character password. She wants to use an  
arrangement of the first 3 letters of her first name
(lat), followed by an arrangement of the 4 digits in 43. GRIDDED RESPONSE A gear that is 8 inches in
1986, the year she was born. How many possible diameter turns a smaller gear that is 3 inches in
passwords can she create in this way? diameter. If the larger gear makes 36 revolutions,
  how many revolutions does the smaller gear make in
F 72 that time?
   
G 144 SOLUTION:  
  Let x be the number of revolutions that small gear
H 288 makes.
   
J 576 8 x
  The equation that represents the situation is .
3 36
SOLUTION:    
3x = 288
x-96
 
  ANSWER:  
G is the correct option. 96
 
 
ANSWER:  
eSolutions Manual - Powered by Cognero 44. SHORT RESPONSE  Write an equation for the Page 23
G
nth line. Show how it fits the pattern for each given
 
line in the list.
 
 
ANSWER:   Line 4: 4(4 - 1) = 4(3) or 12 and 16 - 4 = 12;
 
10-796
Proof by Mathematical Induction Line 5: 5(5 - 1) = 5(4) or 20 and 25 - 5 = 20
   

44. SHORT RESPONSE  Write an equation for the Find the indicated term of each expansion.
nth line. Show how it fits the pattern for each given  
line in the list. 45. fourth term of (x + 2y)
6
   
Line 1: }XQ=}-}
  SOLUTION:  
Line 2: 2X}=4-2 Use the Binomial Theorem to write the expansion in
  sigma notation.
Line 3:  
 
Line 4: 4X3=16-4 I, ~ 6! <, 1. ( )t
(
x+2y )
= Hk!(6-k)!x 2y
 
Line 5: 5X4=25-5  
  For the fourth term k = 3.
 
SOLUTION:  
2 6! 1>-A (' )4 6! c,-l (? )'
n(n-1) =n -n ;  k!(6-k)( -)' = 3!(6-3)!°'" -."
 
Line 1: 1(1 - 1) = 1(0) or 0 and 1 - 1 = 0;
= 20x'(8r')
  = l60x 1/

Line 2: 2(2 - 1) = 2(1) or 2 and 4 - 2 = 2;  


 
Line 3: 3(3 - 1) = 3(2) or 6 and 9 - 3 = 6; ANSWER:  
  3 3
160x y
Line 4: 4(4 - 1) = 4(3) or 12 and 16 - 4 = 12;
   
Line 5: 5(5 - 1) = 5(4) or 20 and 25 - 5 = 20
  46. fifth term of (a + b)
6

ANSWER:    
2
n(n-1) =n -n ;  SOLUTION:  
  Use the Binomial Theorem to write the expansion in
Line 1: 1(1 - 1) = 1(0) or 0 and 1 - 1 = 0; sigma notation.
   
Line 2: 2(2 - 1) = 2(1) or 2 and 4 - 2 = 2; (, ~ 6! t, 'bJ
  (a+ b) =~ a
t uk!(6-k}!
Line 3: 3(3 - 1) = 3(2) or 6 and 9 - 3 = 6;
   
Line 4: 4(4 - 1) = 4(3) or 12 and 16 - 4 = 12; For the fifth term k = 4.
   
Line 5: 5(5 - 1) = 5(4) or 20 and 25 - 5 = 20 6!
---(I
1>-t, t
) ""
6! (I
h-~h~
  k!(6-k)! 4!(6-4)!

Find the indicated term of each expansion. ""I Sa:b1


   
6
45. fourth term of (x + 2y) ANSWER:  
  2 4
15a b
SOLUTION:    
Use the Binomial Theorem to write the expansion in
9
sigma notation. 47. fourth term of (x – y)
   
eSolutions Manual - Powered by Cognero Page 24
SOLUTION:  
Use the Binomial Theorem to write the expansion in
   
ANSWER:   ANSWER:  
2 4 6 3
10-715a b by Mathematical Induction
Proof –84x y
   
9 48. BIOLOGY In a particular forest, scientists are
47. fourth term of (x – y)
  interested in how the population of wolves will
change over the next two years. One model for
SOLUTION:   animal population is the Verhulst population model,
Use the Binomial Theorem to write the expansion in p n + 1 = p n + rpn (1 – p n), where n represents the
sigma notation.
number of time periods that have passed, p n
 
represents the percent of the maximum sustainable
population that exists at time n, and r is the growth
factor.
   
For the fourth term k = 3. a. To find the population of the wolves after one
  year, evaluate p 1 = 0.45 + 1.5(0.45)(1 – 0.45).
9! 'J-.1 ( )'- 9! .,_, ( )'
 
k!(9-k)( -_I' = 3!(9-J)r"' -y
b. Explain what each number in the expression in
--8-l.l'/ part a represents.
   
c. The current population of wolves is 165. Find the
ANSWER:  
new population by multiplying 165 by the value in part
6 3
–84x y a.
   

48. BIOLOGY In a particular forest, scientists are SOLUTION:  


interested in how the population of wolves will a.
change over the next two years. One model for p1 =0..15+1.5(0.45)(1-0.45)
animal population is the Verhulst population model, -0.45 + 0.675(0.55)
p n + 1 = p n + rpn (1 – p n), where n represents the
=0.82125
number of time periods that have passed, p n  
represents the percent of the maximum sustainable b.
population that exists at time n, and r is the growth 0.45 represents the maximum sustainable population
factor. of 45% and 1.5 is the growth factor.
   
a. To find the population of the wolves after one c.
year, evaluate p 1 = 0.45 + 1.5(0.45)(1 – 0.45). New population = 165 × 0.82125
≈136 wolves
  The new population of wolves is about 136.
b. Explain what each number in the expression in  
part a represents.
  ANSWER:  
c. The current population of wolves is 165. Find the a. 0.82125
new population by multiplying 165 by the value in part  
a.
b. 0.45 represents the maximum sustainable
 
population of 45% and 1.5 is the growth factor.
SOLUTION:  
 
a.
c. about 136 wolves
 

Find the exact solution(s) of each system of


  equations.
eSolutions Manual - Powered by Cognero
b.   Page 25

0.45 represents the maximum sustainable population


49. 
of 45% and 1.5 is the growth factor.
c. about 136 wolves
   
c. about 136 wolves
10-7 Proof by Mathematical Induction Find the exact solution(s) of each system of
equations.
Find the exact solution(s) of each system of  
equations. x2 + y2 - I 8x + 24 y + 200 =0
  49. 
4x+3y=O
49.   

  SOLUTION:  
4.t+3y = 0
SOLUTION:  
3y--4x
4
)'=- x
. 3
 
4
  Replace y by --x  in the equation 
3
Replace y by  in the equation  x! + / - I 8x + 24 J' + 200 - 0 .
 
.
  x1 +(-~xr-18x+24(-~x)+200=0

.r2 +~x1 -18x-32.u200=0


9
x2 + ~ x2 - 50x + 200 - 0
9
9x1-16xl-450x+l800 _0
9
9.r2 + I 6.r1 - 450,\' + 1800 = 0
25x1 -450x+ 1800= 0
x2-18x+7'2-0
(x-12)(.r-6)=0
x = 12 or x - 6
 
4
  Substitute the values of x in the equation .r=---x
3  to
Substitute the values of x in the equation  to find the corresponding y values.
 
find the corresponding y values. When x = 12:
  4
When x = 12: y=-~(12)
.)

::.-16
When x = 6:
4
When x = 6: y--~(6)
.)

=-8
 
Therefore, the exact solutions of the system of
  equations are (12, –16) and (6, –8).
Therefore, the exact solutions of the system of  
equations are (12, –16) and (6, –8).
  ANSWER:  
(6, –8), (12, –16)
ANSWER:  
 
(6, –8), (12, –16)
eSolutions Manual - Powered by Cognero Page 26
 
50. 
   

ANSWER:   ANSWER:  
10-7(6, –8), by
Proof –16)
(12,Mathematical Induction
   

Evaluate each expression.


4x2+/,,,,16  
50. 
.\'? + 2y2 = -t 51. P(8, 2)
   

SOLUTION:   SOLUTION:  
Multiply the second equation by –4.
 
-4 ( x:' + 2y-' = 4 ) -+ -4.,·-' - Sy-' = -16 8!
= 6!
 
Add both the equations. -8·7
  ,,,,56
-tx1 + / =16
 

--tr2 -8/ =-16 ANSWER:  


-7/-0 56
." - 0
 
 
Substitute y = 0 in any one of the original equation 52. P(9, 1)
and solve for x.  
  SOLUTION:  
4x' + \., = 16
p - 91.
4x2 +0-16 ~ ,- (9-1)!
.\'? =4 9!
x=±2 8!
  =9
The solution is (:!:2.0) .  
  ANSWER:  
ANSWER:   9
(:!:2.0)  
  53. P(12, 6)
Evaluate each expression.  
  SOLUTION:  
51. P(8, 2)
 
SOLUTION:  

 
ANSWER:  
  665,280
 
eSolutions Manual - Powered by Cognero Page 27
ANSWER:  
56 54. C(5, 2)
 
   
ANSWER:   ANSWER:  
10-79Proof by Mathematical Induction 70
   

53. P(12, 6) 56. C(20, 17)


   
SOLUTION:   SOLUTION:  

r;
p -
,.-(12-6)!
I ')l

't)
(' -
,,-(20-17)!17!
-.
'>QI

12! 20!
=- =
6! 3!17!
=12·11·10·9·8·7 = 1140
= 665.280  
  ANSWER:  
ANSWER:   1140
665,280  
  57. P(12, 2)
 
54. C(5, 2)
  SOLUTION:  
SOLUTION:   ,P ,= 12!
C -
-,
).
,. • (12-2)!
' ' - (5-2)!2! 12!
5! 10!
=- -12-11
3!2!
=10 =132
   

ANSWER:   ANSWER:  
10 132
   

55. C(8, 4) 58. P(7, 2)
   
SOLUTION:   SOLUTION:  
c = 8! P - 7!
~ "' (8-4)!4! ' ,-(7-2)!
8! 7!
= =-
4!4! 5!
= 70 -1·6
  -42
 
ANSWER:  
70 ANSWER:  
  42
 
56. C(20, 17)
  59. C(8, 6)
 
SOLUTION:  
eSolutions Manual - Powered by Cognero Page 28
SOLUTION:  
   
ANSWER:   ANSWER:  
10-742
Proof by Mathematical Induction 24
   
59. C(8, 6) 62.  C{IO.S)·C(8.4)
   
SOLUTION:  
SOLUTION:  
8!
10! 8!
~c.. = (8-6)!6! ,oc\. i.C4 = (10-5)!5!. (8-4)!4!
8!
= 10! 8!
2!6! =-·-
5!5! -t!4!
=28
=252 · 70
 
=17.6-tO
ANSWER:    
28
ANSWER:  
 
17,640
60.  C(9.4)·C(S.3)  
 
SOLUTION:  
9! 5!
9 C4. )(\ = (9--1)!4! (S-3)!3!
9! 5!
=-·-
5!4! 2!3!
-126·10
=1260
 
ANSWER:  
1260
 

61.  C( 6.1) · C( 4.1)


 
SOLUTION:  

6! 4!
=-·-
-6·4
""24
 
ANSWER:  
24
 

62. 
eSolutions Manual - Powered by Cognero Page 29
 
SOLUTION:  
 
ANSWER:  
Practice Test - Chapter 10 129
 

1. Find the next 4 terms of the arithmetic sequence 81, MULTIPLE CHOICE What is the eighth term in
72, 63, … . the arithmetic sequence that begins 18, 20.2, 22.4,
  24.6, …?
 
SOLUTION:  
72-81 = 9 A 26.8
63-72- -9  
B 29
   
The common difference is –9.
C 31.2
To find the next term, add –9 to the previous term.
   
D33.4
63 – 9 = 54
54 – 9 = 45  
45 – 9 = 36 SOLUTION:  
36 – 9 = 27
Given a 1 = 18 and n = 8.
 
The next 4 terms are 54, 45, 36 and 27.  
  The common difference is 20.2 – 18 = 2.2.
 
ANSWER:   a., - a + (n
1 l}d
54, 45, 36, 27
"~ 18+(8-1)2.2
 
-33A
2. Find the 25th term of an arithmetic sequence for
which a 1 = 9 and d = 5. Option D is the correct answer.
   

SOLUTION:   ANSWER:  
Substitute 9, 25 and 5 for a 1, n and d respectively in D
"11 = "1 + (11 l)d .  

  4. Find the four arithmetic means between –9 and 11.


(I~~ -9-t-(25-1)5  
-9+24(5) SOLUTION:  
-9+ 120 Given a 1 = –9 and a 6 = 11.
-129  
  a,, - a1 + (11 l)d
11- 9-( 6 l)d
ANSWER:  
129
Sd 20
  d 4
 
MULTIPLE CHOICE What is the eighth term in The four arithmetic means between –9 and 11 are –
the arithmetic sequence that begins 18, 20.2, 22.4, 5, –1, 3 and 7.
24.6, …?  
 
A 26.8 ANSWER:  
  –5, –1, 3, 7
B 29  
 
C 31.2
eSolutions Manual - Powered by Cognero 5. Find the sum of the arithmetic series for which aPage
1= 1
  11, n = 14, and a n = 22.
D33.4  
   
ANSWER:   ANSWER:  
–5, –1,
Practice 3, 7- Chapter 10
Test 231
   

5. Find the sum of the arithmetic series for which a 1 = 6. MULTIPLE CHOICE What is the next term in
the geometric sequence below?
11, n = 14, and a n = 22.
 
  10 2. 5 5
·2·s·32···
SOLUTION:  
 
13
F "")
->-
 
5
G
   
5
ANSWER:   H 128
231
   
5
J 8
6. MULTIPLE CHOICE What is the next term in
the geometric sequence below?  
 
SOLUTION:  
Find the common ratio (r).
    )
2 _ 1
F 10 -4
  5
8 _
1
5 --
G - 4
2
5
  32 _ 1
5 --
H 8 4

   
J The common ratio is .
   
Find the next term.
SOLUTION:    
Find the common ratio (r). 2_ I 5
" X--
  ->2 4 128
 
Option H is the correct answer.
 
ANSWER:  
H
 

7. Find the three geometric means between 6 and 1536.


   
eSolutions Manual - Powered by Cognero Page 2
The common ratio is . SOLUTION:  
  Given a 1 = 5 and a 6 = 1536.
   
ANSWER:   ANSWER:  
H Test - Chapter 10
Practice 24, 96, 384
   

7. Find the three geometric means between 6 and 1536. 8. Find the sum of the geometric series for which a 1 =
  2
15, r-- , and n = 5.
SOLUTION:   3
Given a 1 = 5 and a 6 = 1536.  
  SOLUTION:  
Find r.
 
aII :(, 1, 1·" I

1536=61'~ 1s-1s(~r
-
I

,.i -256 I- 2
,. -- 4 3
  32)
1s-1s(
The three geometric means between 6 and 1536 are _ 243
24, 96 and 384.
  - G)
ANSWER:   32)
1s(1-
24, 96, 384 243
 
- (~)
8. Find the sum of the geometric series for which a 1 =
_
is( 243
211)
15, , and n = 5.
  - (;)

SOLUTION:  
=15(211)(3)
243
_15{211)
81
1055
=-
27
 
ANSWER:  
1055
27
 

Find the sum of each series, if it exists.


 

9. 

 
SOLUTION:  
There are 12 – 2 + 1 or 11 terms, so n = 11.
 
eSolutions Manual - Powered by Cognero Page 3

 
 
ANSWER:  
ANSWER:  
Practice Test - Chapter 10 does not exist
   

Find the sum of each series, if it exists. 11. 45 + 37 + 29 + … + –11


   
I.?
9.  I{Jk-1) SOLUTION:  
,.2 Given a 1 = 45 and a n = –11.
 
 
SOLUTION:   Find the value of d.
There are 12 – 2 + 1 or 11 terms, so n = 11. 45 37 29
  -8 -8
a1 =3(2) I or5  
a11-3{12)-1 or35 The common difference d is –8.
   
Find n.
Find the sum.
   
a,, =a, +(11- l)d
S
n
=11(a• +a,
2
.) - I I = 4 5 + ( 11 - I)( -8)
= 4 S - 811 + 8
S11 - I I(
5
+/S) -I I
811 = 64
=220 I] =8

   
I:! Find the sum.
Therefore, I{3k-1)-220 .  
l-l
  S
n
= ,, ( a, +2 a., )
ANSWER:  
220
Si. _ s( 4S ; I I )
 
=136
 
10.  ANSWER:  
136
   
SOLUTION:  
Since , the series is diverges and the sum 12. 
does not exist.  
 
SOLUTION:  
ANSWER:   Find the value of r.
does not exist  
 

11. 45 + 37 + 29 + … + –11


   
SOLUTION:  
Since , the series is convergent.
Given a 1 = 45 and a n = –11.
   
FindManual
the value of d. by Cognero Find the sum.
eSolutions - Powered Page 4
 
 
ANSWER:  
ANSWER:  
136 Test - Chapter 10
Practice
   

I 2 -t 13. Write 0.65 as a fraction.


12.  -+-+-+ ...
8 24 72  
 
SOLUTION:  
SOLUTION:  
0.65 = 0.65 + 0.0065
+ 0.000065 + ...
Find the value of r.
  65 65 65
--+--+ +...
100 10000 1000000
 
Find the value of r.
   
6.5
Since , the series is convergent. r = 10,000
6.5
  100
Find the sum. 1
= 100
 
 
s = _!!J_
1-r s = _!!J_
1-r
1
__ 8_ 6.5
-
1--
2 100
3 - 1
1-100
3
8 65
  =w
I 2 4 3
 
-+-+-+. __ ;;;;-
8 24 72 8 ANSWER:  
  65
ANSWER:   99
3
 
-
8 Find the first five terms of each sequence.
   
14. a 1 = –1, a n + 1 = 3a n + 5
13. Write as a fraction.
 
 
SOLUTION:  
SOLUTION:  

 
Find the value of r.
 

 
  The first five terms of the sequence are –1, 2, 11, 38
eSolutions Manual - Powered by Cognero and 119. Page 5
 
ANSWER:  
ANSWER:  
–1, 2, 11, 38, 119
Practice Test - Chapter 10  
 

Find the first five terms of each sequence. 15. a 1 = 4, a n + 1 = a n + n


   
14. a 1 = –1, a n + 1 = 3a n + 5
  SOLUTION:  
Cl,a I - Cl11 T II
SOLUTION:  
01 = ( 4).,.. J
a,1.. 1 = 3u,. + 5
-5
lli =3( 1)+5
a,-(5)+2
-2
-7
a, -3(2} + 5
lJ_i - (7)-.- 3
=II
=10
a, =3(11)-5 a~ = (I 0) + 4
=38
--14
0~ -3(38)+5  
=119 The first five terms of the sequence are 4, 5, 7, 10
  and 14.
The first five terms of the sequence are –1, 2, 11, 38  
and 119.
  ANSWER:  
4, 5, 7, 10, 14
ANSWER:    
–1, 2, 11, 38, 119
  16. MULTIPLE CHOICE What are the first 3 iterates
of f (x) = –5x + 4 for an initial value of x0 = 3?
15. a 1 = 4, a n + 1 = a n + n  
A 3, –11, 59
 
 
SOLUTION:   B –11, 59, –291
 
C –1, –6, –11
 
D 59, –291, 1459
 
SOLUTION:  

 
The first five terms of the sequence are 4, 5, 7, 10
and 14.
 
ANSWER:  
4, 5, 7, 10, 14
 
 
Option B is the correct answer.
eSolutions Manual - Powered by Cognero Page 6
16. MULTIPLE CHOICE What are the first 3 iterates  
of f (x) = –5x + 4 for an initial value of x0 = 3? ANSWER:  
 
 
ANSWER:  
4, 5, 7, 10, 14 ANSWER:  
Practice Test - Chapter 10
4 3 2 2 3 4
  16a – 96a b + 216a b – 216ab + 81b

16. MULTIPLE CHOICE What are the first 3 iterates 18. What is the coefficient of the fifth term of (m + 3n)
6
of f (x) = –5x + 4 for an initial value of x0 = 3? ?
   
A 3, –11, 59 SOLUTION:  
  Use the Binomial Theorem to write the expansion in
B –11, 59, –291 sigma notation.
   
C –1, –6, –11
  ( m + t = """iok!6-k!
z: ( . ) m, _, (3 )'
311
61 6
11

D 59, –291, 1459


   
For the fifth term k = 4.
SOLUTION:    
/(x) :a:: -Sx+ 4 6! Ill
1,-l ("
sn )A = ( 6! ) ,,,IH ("
.,,, )~
k!6-k
( )! 4! 6-4 !
f (.,·o )- -S.,·o + 4
= 1215111211~
/(3)=-5(3)+4  
=-II The coefficient of the fifth term is 1215.
/(/(3)) = -5(-11) +4  
-59 ANSWER:  
J (I (/(3))),., -5(59) + 4 1215
 
=-291
  19. Find the fourth term of the expansion of (c + d) .
9
Option B is the correct answer.  
 
SOLUTION:  
ANSWER:   Use the Binomial Theorem to write the expansion in
B sigma notation.
   
')
4 ( c+d )
9
= '~ 9! c9 4 cl 4
17. Expand (2a – 3b) .
i 0k!{9-k)!
 
 
SOLUTION:   For the fourth term k = 3.
(2a -3b)4 =4Co(2a)4-0C-3b)o +4C1(2a)4-1( -3b)'  
+4C2(2a)4-2(-3b)2 +4C3(2a)4-3(-3b)3
---C
9! 9-i 1"
G = 9! 9-.\
C c
,i
+4C4(2a)4-\-3b)4 k!(9-k)! 3!(9-3)!
=16a4 -96a3b+216a2b2 -2J6ab3+81b4
-84c6d'
 
 
ANSWER:   6 3
The fourth term is 84c d .
4 3 2 2 3 4  
16a – 96a b + 216a b – 216ab + 81b

18. What is the coefficient of the fifth term of (m + 3n) ANSWER:  


6 6 3
? 84c d
   
SOLUTION:   Prove that each statement is true for all
Use Manual
eSolutions the Binomial Theorem
- Powered to write the expansion in
by Cognero positive integers. Page 7
sigma notation.
 
20.  .
ANSWER:   some positive integer k.
6 3  
84c d
 
Practice Test - Chapter 10 Step 3: Show that the given equation is true for n = k
+1
Prove that each statement is true for all  
positive integers. I + 6 + 36 + ... + 6' I + 6hl-l = i( 64 - I)+ 6"1-1

20.  I +6+36+ ... +611


1
-±(611-I) . = i( 64 - I)+ 6"

  =.!.4 _.!.+61
5 5
SOLUTION:   =~-61 _ _!_
5 5
Step 1: When n = 1, the left side of the given
:.!.-6t.l _ _!_
equation is 1. The right side is 1 also, so the equation 5 5
is true for n = 1.
  =5·I (6k•I -I )

 
Step 2: Assume  for
The last expression is the right side of the equation to
some positive integer k. be proved, where n = k + 1. Therefore,
 
Step 3: Show that the given equation is true for n = k 1+6+36+ ... +6" '-±(6" -1)  for all positive 
+1 integers n.
   
I + 6 + 36 + ... + 61-1 + 61+1-1 = i( 6• - I)+ 6._1-1
n
= ~( 61 - I)+ 6t
21. 11 – 1 is divisible by 10.
::>  
I
=-·6 * --+6
I •
5 5 SOLUTION:  
1
=!·6' _ _!_ Step 1: 11 – 1 = 10, which is divisible by 10. The
5 5 statement is true for n = 1.
=.!.4+1 _ _!_  
5 5
k
-i·(6t+1 _ ,)
Step 2: Assume that 11 – 1 is divisible by 10 for
k
some positive integer k. This means that 11 – 1 =
  10r for some whole number r.
The last expression is the right side of the equation to  
be proved, where n = k + 1. Therefore, Step 3:
I-.. 6 + 36 + ... + 61'
1
-±( 611 - I)  for all positive  I I" I= I Or
I 1' - I Or+ I
integers n.
  11(1 t*)=(IOr+l)t I
ANSWER:   I I'• 1 = I I Or + I I
Step 1: When n = 1, the left side of the given I 1.t-+1 - I ~ I I Or + I I - 1
equation is 1. The right side is 1 also, so the equation
is true for n = 1. 1 lh1 - I = I I Or + IO
1-1-10(1
  11' .. lr+ 1)

Step 2: Assume  for  


Since r is a whole umber, 11r + 1 is a whole number.
some positive integer k. Thus, 11
k +1
– 1 is divisible by 10, so the statement
  n
Step 3: Show that the given equation is true for n = k is true for n = k + 1.Therefore, 11 – 1 is divisible by
+1 10 for all positive integers n.
   
ANSWER:  
eSolutions Manual - Powered by Cognero 1 Page 8
Step 1: 11 – 1 = 10, which is divisible by 10. The
statement is true for n = 1.
 
k +1
 
Thus, 11 – 1 is divisible by 10, so the statement
n
is true for n = k + 1.Therefore, 11 – 1 is divisible by ANSWER:  
10 forTest
Practice all positive integers
- Chapter 10 n. n =1
   

ANSWER:   23. SCHOOL There are an equal number of girls and


1
Step 1: 11 – 1 = 10, which is divisible by 10. The boys in Mr. Marshall’s science class. He needs to
statement is true for n = 1. choose 8 students to represent his class at the
science fair. What is the probability that 5 are boys?
 
k  
Step 2: Assume that 11 – 1 is divisible by 10 for
k SOLUTION:  
some positive integer k. This means that 11 – 1 =
10r for some whole number r. Let b represents the number of boys and g
  represents the number of girls.
Step 3:  
2 3
(b +g)8 =s Cob8 +s c,b7g +s C2b6g +s c:it,5g
111 I= IOr
+gC4b4g4 +s Csb3g5 +s C6h2g6
I It - I Or+ I 7 8
+sc1bg +s egg
11(11.1;)=(10r .... 1)11 =bs + 8!/g + Z8?6g2 + S6bsg 3
114 •1 = I I Or+ II +7Q!.i4g4 + 56b3g 5 + 28?2g 6

I 1t+I - I ::. I I Or+ l I - l +8?g7 +g7


 
I lh1 - I = I I Or+ IO
 
11'+1-1-10(1 lr+ t) Sum of the coefficients of the terms = 256
   
Since r is a whole umber, 11r + 1 is a whole number. 5 3
The term 56b g represents that 5 boys are chosen.
k +1
Thus, 11 – 1 is divisible by 10, so the statement  
n -6
is true for n = k + 1.Therefore, 11 – 1 is divisible by P(S boys) :-)-
10 for all positive integers n. 256
  :::0.219 or 21.9%
 
22. Find a counterexample for the following statement.
The probability that 5 are boys is about 21.9%.
   
n n
2 + 4 is divisible by 4.
  ANSWER:  
about 21.9%
SOLUTION:  
 
2' +41 =2 i 4
=6 24. PENDULUM Laurie swings a pendulum. The
  distance traveled per swing decreases by 15% with
Since 6 is not divisible by 4, the statement is false each swing. If the pendulum initially traveled 10
when n = 1. inches, find the total distance traveled when the
  pendulum comes to a rest.
 
ANSWER:  
n =1
 

23. SCHOOL There are an equal number of girls and


boys in Mr. Marshall’s science class. He needs to
choose 8 students to represent his class at the
science fair. What is the probability that 5 are boys?
 
 
SOLUTION:  
eSolutions Manual - Powered by Cognero SOLUTION:   Page 9
Let b represents the number of boys and g
represents the number of girls. Given a 1 = 10 in. and r = 100% – 15% = 0.85
 
 
ANSWER:  
aboutTest
Practice 21.9%
- Chapter 10
 

24. PENDULUM Laurie swings a pendulum. The


distance traveled per swing decreases by 15% with
each swing. If the pendulum initially traveled 10
inches, find the total distance traveled when the
pendulum comes to a rest.
 

 
SOLUTION:  
Given a 1 = 10 in. and r = 100% – 15% = 0.85
 
Find the sum.
 
S=~
1-r
10
=---
1-0.85
=z, 66. 7
 
The distance traveled by the pendulum is about 66.7
inches.
 
 
ANSWER:  
about 66.7 inches
 

eSolutions Manual - Powered by Cognero Page 10


 
ANSWER:  
Study Guide and Review - Chapter 10 false, sequence
 

State whether each sentence is true or false . If 5. The sum of the first n terms of a series is called the
false, replace the underlined term to make a partial sum.
true sentence.  
 
SOLUTION:  
1. An infinite geometric series that has a sum is called a
convergent series. True
   

SOLUTION:   ANSWER:  
True true
   

ANSWER:   6. The formula a n = a n – 2 + a n – 1 is a recursive


true formula.
   
SOLUTION:  
2. Mathematical induction is the process of repeatedly True
composing a function with itself.
 
 
SOLUTION:   ANSWER:  
False, Iteration. true
   

ANSWER:   7. A geometric sequence is a sequence in which every


term is determined by adding a constant value to the
false, iteration
previous term.
   
3. The arithmetic means of a sequence are the terms SOLUTION:  
between any two non-successive terms of an False, arithmetic sequence
arithmetic sequence.
 
 
SOLUTION:   ANSWER:  
True false, arithmetic sequence
   

ANSWER:   8. An infinite geometric series that does not have a sum
is called a partial sum.
true
 
 
SOLUTION:  
4. A term is a list of numbers in a particular order. False, divergent series
   
SOLUTION:  
ANSWER:  
False, sequence
false, divergent series
 
 
ANSWER:  
9. Eleven and 17 are two geometric means between 5
false, sequence
and 23 in the sequence 5, 11, 17, 23.
   
5. The sum of the first n terms of a series is called the
eSolutions Manual - Powered by Cognero SOLUTION:   Page 1
partial sum. False, arithmetic means
   
   
ANSWER:   ANSWER:  
false,
Study divergent
Guide series - Chapter 10
and Review true
   

9. Eleven and 17 are two geometric means between 5 Find the indicated term of each arithmetic
and 23 in the sequence 5, 11, 17, 23. sequence.
   
11. a 1 = 9, d = 3, n = 14
SOLUTION:  
False, arithmetic means  
  SOLUTION:  
ANSWER:   Substitute 9 for a 1, 3 for d, and 14 for n in the
false, arithmetic means formula for the nth term and simplify.
   
u,, =a, + (11- l)d
4
10. Using the Binomial Theorem, (x – 2) can be a1_. = 9 + ( 14 - I) 3
4 3 2
expanded to x – 8x + 24x – 32x + 16. =9-,-39
 
=48
SOLUTION:    
Replace n with 4 in the Binomial Theorem.
  ANSWER:  
(x -2)4 =x4 +4C1x3(-2) +4C2x2( -2)2 48
 
+4C:µ:(-2)3 + (-2)4

=x4 + ~x3(-2) + ~x2(-2)2 12. a 1 = –3, d = 6, n = 22


-'· 2!2.

+ ~x(-2)3 + 16    
-'·
SOLUTION:  
  Substitute –3 for a 1, 6 for d, and 22 for n in the
The statement is true. formula for the nth term and simplify.
   
ANSWER:  
a,, = a, + (11 - I) d
true a!:. =-3+{22-1)6
  =-3+126

Find the indicated term of each arithmetic


= 123
sequence.  
  ANSWER:  
11. a 1 = 9, d = 3, n = 14 123
   
SOLUTION:   13. a 1 = 10, d = –4, n = 9
Substitute 9 for a 1, 3 for d, and 14 for n in the
 
formula for the nth term and simplify.
  SOLUTION:  
Substitute 10 for a 1, –4 for d, and 9 for n in the
formula for the nth term and simplify.
 

 
eSolutions Manual - Powered by Cognero Page 2
ANSWER:  
48  
   
ANSWER:   ANSWER:  
123
Study Guide and Review - Chapter 10 –86
   

13. a 1 = 10, d = –4, n = 9 Find the arithmetic means in each sequence.


   
15. –12, __, __, __, 8
SOLUTION:    
Substitute 10 for a 1, –4 for d, and 9 for n in the SOLUTION:  
formula for the nth term and simplify. Substitute –12 for a 1, 5 for n, and 8 for a 5 in the
 
formula for the nth term and find d.
a,, = a1 + ( 11 - I) d
 
a., = I O + ( 9 - I)( -4) a,,=a ·(11-l)d
-10-32 <Is - -12 +(5- l)d
--22 8 =-12+4d
 
4,i = 20
ANSWER:   d =5
–22  
  The arithmetic means are (–12 + 5) or −7, (–7 + 5)
or −2 and (–2 + 5) or 3.
14. a 1 = –1, d = –5, n = 18  
 
ANSWER:  
SOLUTION:   –7, –2, 3
Substitute –1 for a 1, –5 for d, and 18 for n in the  
formula for the nth term and simplify.
  16. 15, __, __, 29
a,,= a,+ (11- l)d  
a,~ =-1+{18-1)(-5) SOLUTION:  
--1-85 Substitute 15 for a 1, 4 for  n, and 29 for a 4 in the
=-86 formula for the nth term and find d.
   
a~ =« +(11- l)d
ANSWER:  
–86 a~=l5+(4-l)d
  29= 15+3d
3d=I-I
Find the arithmetic means in each sequence.
14
  d=-
15. –12, __, __, __, 8 3
   
The arithmetic means are
SOLUTION:  
Substitute –12 for a 1, 5 for n, and 8 for a 5 in the .
formula for the nth term and find d.  
 
ANSWER:  
59 73

 
eSolutions Manual - Powered by Cognero 17. 12, __, __, __, __, –8 Page 3

   
The arithmetic means are (–12 + 5) or −7, (–7 + 5)
 
ANSWER:  
ANSWER:  
Study Guide and Review - Chapter 10 60, 48, 36
   

17. 12, __, __, __, __, –8 19. BANKING Carson saves $40 every 2 months. If he
  saves at this rate for two years, how much will he
have at the end of two years?
SOLUTION:    
Substitute 12 for a 1, 6 for n, and –8 for a 6 in the
SOLUTION:  
formula for the nth term and find d.
  Substitute 40 for a 1, 12 for n and 40 for d in the
a,, =a ..-(11-l)d
formula for the nth term and find a 12.
a = 12 ~ ( 6 - I )d
 
-8"" 12+5d "· ~a, +(11-l)d
Sd,,,,-20
"11 =-10+(12-1)40
d--l
=-10+ 440
  = 480
The arithmetic means are (12 – 4) or 8, (8 – 4) or 4,  
(4 – 4) or 0 and (0 – 4) or −4.
  ANSWER:  
$480
ANSWER:  
 
8, 4, 0, –4
  Find Sn for each arithmetic series.
18. 72, __, __, __, 24  
  20. a 1 = 16, a n = 48, n = 6
SOLUTION:    
Substitute 72 for a 1, 5 for n, and 24 for a 5 in the SOLUTION:  
formula for the nth term and find d. Substitute 16 for a 1, 48 for a n, 6 for n in the Sum
  a,, -a -(11-l)d formula.
a; 71+(5-l)tl  
24 -72 + .Jd
4d--48 6
- 2(16-+ 48)
d--12
=3(64)
  192
The arithmetic means are (72 – 12) or 60, (60 – 12)
or 48 and (48 – 12) or 36.  
 
ANSWER:  
ANSWER:   192
60, 48, 36  
 

19. BANKING Carson saves $40 every 2 months. If he 21. a 1 = 8, a n = 96, n = 20


saves at this rate for two years, how much will he  
have at the end of two years?
  SOLUTION:  
Substitute 8 for a 1, 96 for a n, 20 for n in the Sum
SOLUTION:  
formula.
Manual 40
Substitute for n and 40 for d in the
for a 1,by12Cognero
eSolutions - Powered   Page 4
formula for the nth term and find a 12.
 
 
ANSWER:  
ANSWER:  
192
Study 1040
  Guide and Review - Chapter 10
 

21. a 1 = 8, a n = 96, n = 20 22. 9 + 14 + 19 + … + 74


   
SOLUTION:  
SOLUTION:  
Substitute 9 for a 1, 5 for d, and 74 for a n in the
Substitute 8 for a 1, 96 for a n, 20 for n in the Sum
formula. formula for the nth term and find n.
   
JI a., =a +(11-l)d
S=-(a+a)
" 2 I r,
74 =9+(11-1)5
20 74=9+511-5
=2(8+96)
511 = 70
= 10(104) 11-14
= 1040  
  Substitute 9 for a 1, 74 for a n, 14 for n in the Sum
ANSWER:   formula.
 
1040
JI
  2 I +a)n
Sn =-(a

22. 9 + 14 + 19 + … + 74 14
= (9+ 74)
  2
= 7(83)
SOLUTION:  
=581
Substitute 9 for a 1, 5 for d, and 74 for a n in the
 
formula for the nth term and find n.
  ANSWER:  
581
 

23. 16 + 7 + -2 + … + –65


 
SOLUTION:  
 
Substitute 16 for a 1, –9 for d, and –65 for a n in the
Substitute 9 for a 1, 74 for a n, 14 for n in the Sum
formula. formula for the nth term and find n.
   

 
  Substitute 16 for a 1, –65 for a n, 10 for n in the Sum
ANSWER:   formula.
 
581
 
eSolutions Manual - Powered by Cognero Page 5
23. 16 + 7 + -2 + … + –65
 
   
ANSWER:   ANSWER:  
581
Study Guide and Review - Chapter 10 –245
   

23. 16 + 7 + -2 + … + –65 24. DRAMA Laura has a drama performance in 12


  days. She plans to practice her lines each night. On
the first night she rehearses her lines 2 times. The
SOLUTION:   next night she rehearses her lines 4 times. The third
Substitute 16 for a 1, –9 for d, and –65 for a n in the night she rehearses her lines 6 times. On the eleventh
formula for the nth term and find n. night, how many times has she rehearsed her lines?
   
a,. =a, +(11-l}c/ SOLUTION:  
-65-- 16+ (11-1)(-9) The arithmetic sequence that represents the situation
65-16-911 ·9 is 2, 4, 6,…
Substitute 2 for a 1, 2 for d, and 11 for n in the
911 ""90
11- IO formula for the nth term and find a 11.
   
Substitute 16 for a 1, –65 for a n, 10 for n in the Sum o, ", +(11 l}c/
formula. a11 - 2 + ( I I - I} 2
  2+20
22
 
1:
- (16 65} Substitute 2 for a 1, 22 for a n, 11 for n in the Sum
formula.
=5{-t9)
 
-245
 
S,, - 2II(a1 +a,.)
11
ANSWER:   = (2 + 22}
2
–245
  _ 1~(2-1)

24. DRAMA Laura has a drama performance in 12 :.132


days. She plans to practice her lines each night. On  
the first night she rehearses her lines 2 times. The
next night she rehearses her lines 4 times. The third ANSWER:  
night she rehearses her lines 6 times. On the eleventh 132
night, how many times has she rehearsed her lines?
 
 
SOLUTION:   Find the sum of each arithmetic series.
 
The arithmetic sequence that represents the situation
is 2, 4, 6,… 25. 
Substitute 2 for a 1, 2 for d, and 11 for n in the
formula for the nth term and find a 11.  
  SOLUTION:  

Use the formula .


 
There are 17 terms, a 1 = 3(5) – 2 or 13, and a 21 = 3
  (21) – 2
eSolutions Manual - Powered by Cognero or 61. Page 6
Substitute 2 for a 1, 22 for a n, 11 for n in the Sum
 
formula.
   
ANSWER:   ANSWER:  
132
Study Guide and Review - Chapter 10 319
   
Find the sum of each arithmetic series. I!

  27.  I(-2kt5)
11 1 J

25.  I:(Jk-2)  
"~ SOLUTION:  
 
11
SOLUTION:   Use the formula Sn --(a
2 I + Cl., ) .

Use the formula sn ;;;;!2.(,,... + a,,) .  


2 There are 9 terms, a 1 = –2(4) + 5 or –3, and a 9 = –2
  (12) + 5
There are 17 terms, a 1 = 3(5) – 2 or 13, and a 21 = 3 or –19.
(21) – 2  
or 61. 9
  s., ; ; ; -(-3 - 19)
2
• 17 =-99
•S 11 = 2 (13+61)
 
=629
  ANSWER:  
–99
ANSWER:  
 
629
  Find the indicated term for each geometric
sequence.
10
 
26.  I:(6k-1) 28. a 1 = 5, r = 2, n = 7
" 0  
 
SOLUTION:  
SOLUTION:  
Substitute 5 for a 1, 2 for r, 7 for n in the formula for
c- 11
Use the formula ,1n --(a
2 I + ti,.) . the nth term and simplify.
 
 
There are 11 terms, a 1 = 6(0) – 1 or –1, and a 11 = 6
«; =«
·r",
(10) – 1
a, = S · ( 2~-,)
or 59. = S( 64)
 
=320
11
S1, = 2 (-1 +59)  
The seventh term is 320.
=319  
 
ANSWER:  
ANSWER:  
320
319
 
 
29. a 1 = 11, r = 3, n = 3
27. 
 
  SOLUTION:  
eSolutions Manual - Powered by Cognero Page 7
SOLUTION:   Substitute 11 for a 1, 3 for r, 3 for n in the formula for
the nth term and simplify.
Use the formula .
 
ANSWER:  
ANSWER:  
320
Study 8
  Guide and Review - Chapter 10
 

29. a 1 = 11, r = 3, n = 3
31. a 8 for
 
 
SOLUTION:  
SOLUTION:  
Substitute 11 for a 1, 3 for r, 3 for n in the formula for
Find the common ratio r.
the nth term and simplify.
 
  r=-
3 I
:--
u - u . ,..... 8 8
" I

a, I I · ( 3' ) 3 8
= x
8 1
11(9) -3
=99
 
  I
The third term is 99. Substitute 8 for a 1, 3 for r, 8 for n in the formula for
 
the nth term and simplify.
ANSWER:     " - a .,. ... ,
" I
99
I .. ~
  a -·J
' 8
I 2187
30.  Gt1=d28.r --.n-5
2 8
    2187
SOLUTION:   The eighth term is 8.
Substitute 128 for a 1,  for r,
2 5 for n in the  
formula for the nth term and simplify. ANSWER:  
  2187
Cl ; (I • r" I
8
" I

( -2I)'
1

a; =128·  

=8 Find the geometric means in each sequence.


 
 
32. 6, __, __, 162
The fifth term is 8.
 
 
SOLUTION:  
ANSWER:  
Substitute 6 for a 1, 162 for a 4, 4 for n in the formula
8
for the nth term and find r.
 
 

31. a 8 for
 
SOLUTION:  
Find the common ratio r.
   
eSolutions Manual - Powered by Cognero The geometric means are 6(3) or 18 and 18(3) orPage
54.8
 
ANSWER:  
72 and 72(±3) or ±216.
ANSWER:    

Study Guide and Review - Chapter 10 ANSWER:  


 

Find the geometric means in each sequence. 34. –4, __, __, 108
   
32. 6, __, __, 162
SOLUTION:  
 
Substitute –4 for a 1, 108 for a 4, 4 for n in the
SOLUTION:  
formula for the nth term and find r.
Substitute 6 for a 1, 162 for a 4, 4 for n in the formula  
for the nth term and find r. tin = (II , r" I
 
"~ = -4. r"-1
a,, =a · r"
1

108 = --tr'
=6·/'H
{I"
-27 = r'
162-6r'
-3-r
27 = r'  
3=r The geometric means are –4(–3) or 12 and 12(–3) or
  –36.
The geometric means are 6(3) or 18 and 18(3) or 54.  
 
ANSWER:  
ANSWER:   12, –36
18, 54  
 
35. SAVINGS Nolan has a savings account with a
current balance of $1500. What would be Nolan’s
33. 8, __, __, __, 648 account balance after 4 years if he receives 5%
  interest annually?
SOLUTION:    
Substitute 8 for a 1, 648 for a 5, 5 for n in the formula SOLUTION:  
for the nth term and find r. Substitute 1500 for a 0, 1.05 for r and 4 for n in the
 
1
formula and simplify.
a,, = a, · r"
 
a; =8·r1,
a,, = u.,r"
6-ts = s,.~ a, = 1500(1.054)
81 = r4 :::::$1823.26
±3-r  
 
The geometric means are 8(±3) or ±24, ±24(±3) or ANSWER:  
72 and 72(±3) or ±216. $1823.26
   
ANSWER:  
Find Sn for each geometric series.
±24. 72.±216
 
34. –4, __, __, 108 36. a 1 = 15, r = 2, n = 4
   
SOLUTION:   SOLUTION:  
Substitute –4 for a 1, 108 for a 4, 4 for n in the Substitute 15 for a 1, 2 for r, 4 for n in the Sum
eSolutions Manual - Powered by Cognero Page 9
formula for the nth term and find r. formula.
   
   
ANSWER:   ANSWER:  
$1823.26
Study Guide and Review - Chapter 10 12,285
   

Find Sn for each geometric series. 38. 5 – 10 + 20 – … to 7 terms


   
36. a 1 = 15, r = 2, n = 4 SOLUTION:  
  Find the common ratio r.
 
SOLUTION:   10
r--
Substitute 15 for a 1, 2 for r, 4 for n in the Sum 5
- 2
formula.
   
a, - a r " Substitute 5 for a 1, –2 for r, 7 for n in the Sum
S" = I -r
formula.
15-15(21)  
a, -a,r'
1-2 s" -------
1-r
-225
-I
5-5( :f)
225 1+2
  6-45
3
ANSWER:   215
225  
 
ANSWER:  
37. a 1 = 9, r = 4, n = 6 215
   

SOLUTION:   39. 243 + 81 + 27 + … to 5 terms


Substitute 9 for a 1, 4 for r, 6 for n in the Sum  
 
formula.
  SOLUTION:  
Find the common ratio r.

1-4
36855
 
-3
-12.285 Substitute 243 for a 1,  for r, 5 for n in the Sum
  formula.
ANSWER:    
12,285
 

38. 5 – 10 + 20 – … to 7 terms


 
SOLUTION:  
eSolutions the common
FindManual ratio
- Powered r.
by Cognero Page 10
 

 
   
ANSWER:   ANSWER:  
215
Study Guide and Review - Chapter 10 363
   

39. 243 + 81 + 27 + … to 5 terms Evaluate the sum of each geometric series.


   
 
40. 
SOLUTION:  
Find the common ratio r.  
,. _ 81
--243 SOLUTION:  
Substitute 3 for a 1, –2 for r, 7 for n in the Sum
I
formula.
3
 
 
S = 01 - a.r"
Substitute 243 for a 1,  for r, 5 for n in the Sum " I r
3
formula. = 3-3(-2)'
  1+2
SJ,-- a1 -aI r" 387
1-r 3
=129
243-243-(~f
ANSWER:  
,_, 129
3
 
= 242 +3.
3
=363
  41. 

ANSWER:    
363 SOLUTION:  
 
Substitute –1 for a 1,  for r, 8 for n in the Sum
Evaluate the sum of each geometric series. formula.
   
40. 

 
SOLUTION:  
Substitute 3 for a 1, –2 for r, 7 for n in the Sum
formula.
 

 
ANSWER:  
eSolutions Manual - Powered by Cognero Page 11
ANSWER:  
129  
ANSWER:   ANSWER:  
129
Study
  Guide and Review - Chapter 10
 

42. ADVERTISING Natalie is handing out fliers to


41.  advertise the next student council meeting. She hands
out fliers to 4 people. Then, each of those 4 people
  hand out 4 fliers to 4 other people. Those 4 then hand
SOLUTION:   out 4 fliers to 4 new people. If Natalie is considered
.., the first round, how many people will have been
Substitute –1 for a 1, given fliers after 4 rounds?
3 for r, 8 for n in the Sum
 
formula.
  SOLUTION:  
s - "• - (
'1' ·" Substitute 1 for a 1, 4 for r, 4 for n in the Sum
" 1-r
formula.

=
-1+1-(~r.,  
S,,
ll -ar •
I- -... I r
., 1-1(4)'
256 I =
=-1+-+- 1-4
6561 3 1-256
= _ 6305 i..3 -3
6561 -255
6305
=---- _.,"
2187 85
   
Therefore, 85 people will have been given flier after
ANSWER:   4 rounds.
6305
 
2187
  ANSWER:  
85
42. ADVERTISING Natalie is handing out fliers to  
advertise the next student council meeting. She hands
out fliers to 4 people. Then, each of those 4 people Find the sum of each infinite series, if it exists.
hand out 4 fliers to 4 other people. Those 4 then hand  
out 4 fliers to 4 new people. If Natalie is considered
the first round, how many people will have been 43. 
given fliers after 4 rounds?
   

SOLUTION:   SOLUTION:  
Substitute 1 for a 1, 4 for r, 4 for n in the Sum Substitute 8 for a 1 and  for r in the sum formula.
formula.
 
 

eSolutions Manual - Powered by Cognero Page 12

 
   
ANSWER:   ANSWER:  
85Guide and Review - Chapter 10
Study does not exist
   

I3-2I )A-I
Find the sum of each infinite series, if it exists. 7
(
  45. 
,, I
3
43.  "• - 8.r - -  
4
  SOLUTION:  
SOLUTION:   Substitute 3 for a 1 and  for r in the sum formula.
2
"
.)
Substitute 8 for a 1 and  for r in the sum formula.  
...
  S=~
" 1-r
S=~ ..
.)
" 1-r --- I
8 I-
=-- 2
3
I- 3
4
I
8
2
I
=6
4
 
=32
  ANSWER:  
ANSWER:   6
 
32
  46. PHYSICAL SCIENCE Maddy drops a ball off of a
building that is 60 feet high. Each time the ball
~ _ 20 + 80 _ 320 bounces, it bounces back to  its previous height. If 
44.  6 18 54 162 + ...
  the ball continues to follow this pattern, what will be
the total distance that the ball travels?
SOLUTION:    
Find the value of r to determine if the sum exists.
  SOLUTION:  

Substitute 60 for a 1 and  for r in the sum formula.

4  
3
 
Since , the series diverges and the sum does
not exist.
 
ANSWER:    
does not exist Therefore, the total distance traveled by the ball is 2
(180) – 60 or 300 ft.
 
 

45.  ANSWER:  
eSolutions Manual - Powered by Cognero 300 ft Page 13
   
SOLUTION:  
   
ANSWER:   ANSWER:  
6 Guide and Review - Chapter 10
Study 300 ft
   

46. PHYSICAL SCIENCE Maddy drops a ball off of a Find the first five terms of each sequence.
building that is 60 feet high. Each time the ball  
2 47. a 1 = –3, a n + 1 = a n + 4
bounces, it bounces back to  its previous height. If 
3  
the ball continues to follow this pattern, what will be
the total distance that the ball travels? SOLUTION:  
  (IN•l:(ln~4

SOLUTION:   a1.1 = a, + 4
2 ": =-3+4 or I
Substitute 60 for a 1 and  for r in the sum formula.
3 "=·· = ": + 4
  a,= I+ 4 or 5
s =-3_ lll.l =a, +4
" 1-r
a;» 5 + 4 or 9
60
a1•1 =a, +4
-1- 2
3 "s - 9 + 4 or 13
-60x3  
""'180 The first five terms of the sequence are –3, 1, 5, 9,
  and 13.
Therefore, the total distance traveled by the ball is 2  
(180) – 60 or 300 ft. ANSWER:  
 
–3, 1, 5, 9, 13
ANSWER:    
300 ft
  48. a 1 = 5, a n + 1 = 2a n – 5
Find the first five terms of each sequence.  
  SOLUTION:  
47. a 1 = –3, a n + 1 = a n + 4
 
SOLUTION:  

 
The first five terms of the sequence are 5, 5, 5, 5 and
5.
   
The first five terms of the sequence are –3, 1, 5, 9, ANSWER:  
and 13.
  5, 5, 5, 5, 5
eSolutions Manual - Powered by Cognero   Page 14
ANSWER:  
–3, 1, 5, 9, 13 49. a 1 = 1, a n + 1 = a n + 5
 
ANSWER:  
ANSWER:  
–3, 1, 5, 9, 13
Study 1, 6, 11, 16, 21
  Guide and Review - Chapter 10
 

48. a 1 = 5, a n + 1 = 2a n – 5 50. SAVINGS Sari has a savings account with a $12,000


  balance. She has a 5% interest rate that is
compounded monthly. Every month Sari adds $500 to
SOLUTION:   the account. The recursive formula b n = 1.05b n – 1 +
a,,.1 = 2a,, - 5 500 describes the balance in Sari’s savings account
a1•1 =2a1 -5 after n months. Find the balance of Sari’s account
after 3 months. Round your answer to the nearest
a1 -2(5)-5 or S
penny.
a:., = 2az 5  
'11 =2(5)-5or5 SOLUTION:  
a,.1 = Ta, 5 a 0 = 12000
a, = 2 ( 5)- 5 or 5  
a .. , =2a, -5 h,, = I .05b,, 1 + 500
a,=2(5)-5or5 b, = 1.05(12000) + 500
  ,,,13100
The first five terms of the sequence are 5, 5, 5, 5 and
h: = 1.05 ( 13100) + 500
5.
  -14255
bl -1.05(14255)+500
ANSWER:  
= 15467.75
5, 5, 5, 5, 5  
  Sari will have $15,467.75 after 3 months.
 
49. a 1 = 1, a n + 1 = a n + 5
  ANSWER:  
$15,467.75
SOLUTION:  
 
a,,.1 = a,, + 5
lll•I = a +5
1 Find the first three iterates of each function for
<1, = 1 + 5 or 6 the given initial value.
 
"=·· - +5Cl: 51. f (x) = 2x + 1, x 0 = 3
a,= 6+ 5 or 11
 
SOLUTION:  
a~=ll+Sorl6
a1•1 =a, +S
,,, = 16 + 5 or 2 I
 
The first five terms of the sequence are 1, 6, 11, 16,
and 21.
 
ANSWER:  
1, 6, 11, 16, 21
 

50. SAVINGS Sari has a savings account with a $12,000


balance.
eSolutions She
Manual has a 5%
- Powered interest
rate that is
by Cognero Page 15
compounded monthly. Every month Sari adds $500 to
the account. The recursive formula b n = 1.05b n – 1 +
 
ANSWER:  
ANSWER:  
7, 15, 31
$15,467.75
Study Guide and Review - Chapter 10  
 

Find the first three iterates of each function for 52. f (x) = 5x – 4, x 0 = 1
the given initial value.
 
  x. - f ( '( )
51. f (x) = 2x + 1, x 0 = 3 SOLUTION:  
- /(1)
 
:= 5(1) .4
SOLUTION:   _,
.'C, - /(x,)
\",~/(.t,)
-e/(3)
:= 2(3) \ "'/(1)
:=5(1) 4
_7
:I
X,..; f(.t)
:= .f (7) x, s: J(x.)
:=2(7)-l "'/(1)
:= 5(1) .4
=IS
/( \",)
_,
\",

-.f(7)
:=2(15)+1  
..,31 The first three iterates are 1, 1, and 1.
 
 
The first three iterates are 7, 15, and 31. ANSWER:  
  1, 1, 1
 
ANSWER:  
7, 15, 31 53. f (x) = 6x – 1, x 0 = 2
   
SOLUTION:  
52. f (x) = 5x – 4, x 0 = 1
 
SOLUTION:  

 
The first three iterates are 11, 65, and 389.
eSolutions Manual - Powered by Cognero   Page 16

  ANSWER:  
The first three iterates are 1, 1, and 1.
   
ANSWER:   ANSWER:  
1, Guide
Study 1, 1 and Review - Chapter 10 11, 65, 389
   

53. f (x) = 6x – 1, x 0 = 2 54. f (x) = 3x + 1, x 0 = 4


   
SOLUTION:   SOLUTION:  
x, - J(x,) X1 - /(x 0)

=/(2) = /(4)
=6(2) I =3(4)+1
-11 -13
X1 -f(x,) x~ - /(x 1)

=/(11) = /(13)
=6(11)-1 =3(13)+1
=65 -40
x1 -.f(xJ x, - f(x~)
= /(65) = !( 40)
=6(65)-1 =3( 40)+ I
=389 -121
   
The first three iterates are 11, 65, and 389. The first three iterates are 13, 40, and 121.
   
ANSWER:   ANSWER:  
11, 65, 389 13, 40, 121
   

54. f (x) = 3x + 1, x 0 = 4 Expand each binomial.


   
3
55. (a + b)
SOLUTION:    
SOLUTION:  
Replace n with 3 in the Binomial Theorem.
 
(a+h}' =a1 + ,C1a1h+ ,C1ah:. +h'
 
=a 1 + 3clh + 3ah1 + h
1

 
ANSWER:  
3 2 2 3
a + 3a b + 3ab + b

7
56. (y – 3)
 
SOLUTION:  
  Replace n with 7 in the Binomial Theorem. 
The first three iterates are 13, 40, and 121.  
  Manual - Powered by Cognero
eSolutions Page 17

ANSWER:  
 
 
ANSWER:  
ANSWER:  
5 4 3 2
Study –32z + 240z – 720z + 1080z – 810z + 243
a Guide
+ 3a band Review
+ b - Chapter 10
3 2 2 3
+ 3ab
 
7 4
56. (y – 3) 58. (4a – 3b)
   
 
SOLUTION:  
Replace n with 7 in the Binomial Theorem.  SOLUTION:  
  Replace n with 4 in the Binomial Theorem.
(y - 3)7 = y7 + 7Cl)'6( -3) + 7C2Y5( -3)2  
+7C3Y4c-3)3 +7C4y3(-3)4 (4a -3b)4 = (4a)4 +4C1(4a)3( -3b)

+ 7Csr2( -3)5 + 7C6J?( -3)6 + ( -3) 7 +4C2(4a)2( -3b)2


=/ + 2!y6(-3) + ..1Ly5(9) +4C3(4a)( -3b)3 + ( -3b)4
6! 2!51

+? ~li13(81)
,:,!4.ly4c-27) + 4.,:,, = 25624 + 1,' (64a3)c -3b)
+ ..1Ly 2( -243) + 2!((729) - 2187
 
5!2! 6.
   + 2;~1(t 6a2) (9b2)
=/-21r6+1s9y5-945y4
+ 2835y3 - 5103y2 + 5103y - 2187 + ~;C4a)(-27b
~-
3) + (a1b4)
   
= 25624 -76&i3b
ANSWER:   + 864a2b2 -432ab3 + 81b4
7 6 5
y – 21y + 189y – 945y + 2835y – 5103y +
4 3 2  
5103y – 2187 ANSWER:  
  4 3 2 2 3 4
256a – 768a b + 864a b – 432ab + 81b
5
57. (3 – 2z)
  59. 
SOLUTION:    
Replace n with 5 in the Binomial Theorem. 
  SOLUTION:  
(3 -2z} = (3)5 + 5C1(3)\-2z) Replace n with 5 in the Binomial Theorem.
+ 5C2(3)3( -2z)2 + 5C3(3)2( -2z}  
+ 5C4(3)( -2z)4 + ( -2z)5 (x-±f =x5+scix4(-i)+se2x3 (-±)2
= 243 + 11(81)( -2z) + .1Lc21)(4z2)
41 2131 +sc3X2 (-±)3 +sc4X(-if + (-±f
+ .1L(9)(-&3)
3!21
+ 20)(162
4!
4) + (-32z5) =XS+ 1lx4(-'-) + -1Lx3(_1_)  
     41 4 2G! 16
= 243 -810z + 1080z2
+ 3~/(- ~) +~x(2~6) + (-1014)
-720z3+240z4 -32z5
5 54 53 52 5 1
  -x - 4x + 8x - 32x + 256x- 1024
 
ANSWER:  
5 4 3 2 ANSWER:  
–32z + 240z – 720z + 1080z – 810z + 243
 
4
 
58. (4a – 3b)
  Find the indicated term of each expression.
   
8
SOLUTION:   60. third term of (a + 2b)
 
Replace
eSolutions n -with
Manual 4 inby
Powered theCognero
Binomial Theorem. Page 18
  SOLUTION:  
Use the Binomial Theorem to write the expansion in
 
ANSWER:   ANSWER:  
2 5
Study Guide and Review - Chapter 10 193,536x y
   
Find the indicated term of each expression. 62. second term of (4x-St'
   
8
60. third term of (a + 2b)
  SOLUTION:  
Use the Binomial Theorem to write the expansion in
SOLUTION:   sigma notation.
Use the Binomial Theorem to write the expansion in  
sigma notation. •• , 10'
.I .)10 = '~
( -,.\"-) • (4 .\")I' 1 )•I
  , uk!{IO-k)!
(a+2bf = :tuk!(8-k)!
t
&! a' '(2b)'  
For the second term k = 1.
   
For the third term k = 2. 10! (4x)IO-k(-sl =~(4x)IO-l(-s)I
  k!(IO-k)! 1!(10-1)!

=-10!(4" x•)( -,•)


S! uu(2h)1== S! u~:(2h): 9!
k!(8-k)! 2!(8-2)! =-13,107.200.r9
=112,/b;  
 
ANSWER:  
ANSWER:   9
-13,107,200x
6 2
112a b  
 
Prove that each statement is true for all
7
positive integers.
61. sixth term of (3x + 4y)  
  11(11 + 1)(11 + 2)
63.  2 + 6 + 12 + ... + 11(11+ 1)- ......:...-~-...:..
SOLUTION:   3
Use the Binomial Theorem to write the expansion in  
sigma notation. SOLUTION:  
 
Step 1 When n = 1, the left side of the equation is
., ~ 71 1 l I
(3x+4.r) = ~ 1

(3x) (4y) equal to 2. The right side of the equation is also equal
A c,k.(7-k). 1

  to 2. So the equation is true for n = 1.


For the sixth term k = 5.  
  Step 2 Assume that
7! (3 )7. I (4 )I 7! (~ )7 S ( 4Y )S k(k + l)(k + 2)
k!(7-k)! ,,· .l' ~ 5!(7-5)! sx 2+6+ 12+ ... +k(k + !)=--------..:.  for some 
3
= 21{9x:){1024/)
positive integer k.
= 193.536.rz/
 
 
Step 3
ANSWER:   I· 2 + 2 ·3 + 3·42+ ... +k(k + l)~(k + l)(k + 2)
2 5
193,536x y
 

62. second term of
 
SOLUTION:  
eSolutions Manual - Powered by Cognero Page 19

Use the Binomial Theorem to write the expansion in


sigma notation.
positive integer k.  
  The last expression is the right side of the equation to
Study Guide and Review - Chapter 10 be proved, where n = k + 1. Thus, the equation is
Step 3
true for n = k + 1. Therefore,
k(k i- l)(k + 2) 3(k + l)(k i-2) 11(11 + 1 )(11 + 2)
=-- + - 2 + 6 + I 2 + ... + 11 ( 11 + 1 ) - --'-----''-"------  for all 
3 3 3
= (k + 1)[ k(k.,.. 2) + 3(k + 2)] positive integers n.
3  
= (k+l)(k+2)(k+3)
n
3 64. 7 – 1 is divisible by 6.
{k+l}((kTl)+l){(k+1)+2)  
= 3 SOLUTION:  
  1
The last expression is the right side of the equation to Step 1: When n = 1, 7 – 1 = 7 – 1 or 6. Since 6
be proved, where n = k + 1. Thus, the equation is divided by 6 is 1, the statement is true for n = 1.
true for n = k + 1. Therefore,  
n(11+l)(11+2) k
Step 2: Assume that 7 – 1 is divisible by 6 for some
2+6+12+ ... +11(11+1)= "  for all 
.> k
positive integer k. This means that 7 – 1 = 6r for
positive integers n.
  some whole number r.
 
ANSWER:   Step 3:
Step 1 When n = 1, the left side of the equation is 7' -1 =6r
equal to 2. The right side of the equation is also equal 7' =6r + I
to 2. So the equation is true for n = 1. 7l•I = 42r + 7
 
71•1 -1 = 42r + 7 - I
Step 2 Assume that
71•1 - 1 - 42r + 6
k(k + l)(k + 2)
2+6+ 12+ ... +k(k + 1)  for some  7'•1-1=6(7r+l)
3
positive integer k  
  Since r is a whole number, 7r + 1 is a whole number.
k +1
Step 3: I· 2 + 2 ·3+ 3 ·42+ ... + k(k + 1)-r(k + l)(k + 2) Thus, 7 – 1 is divisible by 6, so the statement is
n
_ k(k + l){k + 2) +(k + l}(k + 2) true for n = k + 1. Therefore, 7 – 1 is divisible by 6
3 for all positive integers n.
k(k t- l)(k + 2) 3(k + l)(k 1- 2)  
= 3 + -----------------
3
ANSWER:  
= ( k +I)[ k ( k.,.. 2) + 3 ( k + 2)] 1
Step 1: When n = 1, 7 – 1 = 7 – 1 or 6. Since 6
3
divided by 6 is 1, the statement is true for n = 1.
= (k+l)(k+2)(k+3)
 
3 k
(k+l}((k-1)+1){(k+l)+2) Step 2: Assume that 7 – 1 is divisible by 6 for some
= 3 positive integer k. This means that 7 – 1 = 6r for
k

  some whole number r.


The last expression is the right side of the equation to  
be proved, where n = k + 1. Thus, the equation is Step 3:
true for n = k + 1. Therefore,

 for all 
eSolutions Manual - Powered by Cognero Page 20
positive integers n.
 
Step 2: Assume that 7 – 1 is divisible by 6 for some  
k
positive integer k. This means that 7 – 1 = 6r for
ANSWER:  
some whole number r.
1
Study
  Guide and Review - Chapter 10 Step 1: When n = 1, 5 – 1 = 5 – 1 or 4. Since 4
Step 3: divided by 4 is 1, the statement is true for n = 1.
 
7'' I= 6r
k
Step 2: Assume that 5 – 1 is divisible by 4 for some
7t - 6r + I k
positive integer k. This means that 5 – 1 = 4r for
7h1 =42r+ 7
some whole number r.
7" •I - I - 42r + 7 - I
 
i•1 1=42r+6 Step 3:
i-•1-1-6(7r+l) 5i - I= 4r
  s' = -lr + I
Since r is a whole number, 7r + 1 is a whole number. 5t•I : 20r T 5
k +1
Thus, 7 – 1 is divisible by 6, so the statement is 5t. - I - 20r + 5 - I
n
true for n = k + 1. Therefore, 7 – 1 is divisible by 6 5hl -1- 20,..... 4
for all positive integers n.
s'• -1=-l(Sr+l)
 
 
n Since r is a whole number, 5r + 1 is a whole number.
65. 5 – 1 is divisible by 4.
k +1
  Thus, 5 – 1 is divisible by 4, so the statement is
n
SOLUTION:   true for n = k + 1. Therefore, 5 – 1 is divisible by 4
1 for all positive integers n.
Step 1: When n = 1, 5 – 1 = 5 – 1 or 4. Since 4
 
divided by 4 is 1, the statement is true for n = 1.
  Find a counterexample for each statement.
k  
Step 2: Assume that 5 – 1 is divisible by 4 for some
k 66. 8n + 3 is divisible by 11.
positive integer k. This means that 5 – 1 = 4r for
 
some whole number r.
  SOLUTION:  
Step 3: Substitute n = 2 in the given expression.
 
5i - I = -lr
8(2)+3=19
5i = 4r + I  
5t+t : 201' T 5 19 is not divisible by 11, so the expression is not true
5t.1 - I ""20r + 5- I for n = 2.
 
54•1 - I - 20r - 4
St.1-1=-l(Sr+l) ANSWER:  
  n =2
Since r is a whole number, 5r + 1 is a whole number.  
k +1
Thus, 5 – 1 is divisible by 4, so the statement is
n n+1
true for n = k + 1. Therefore, 5 – 1 is divisible by 4 67. 6 – 2 is divisible by 17.
for all positive integers n.  
  SOLUTION:  
Substitute n = 2 in the given expression.
ANSWER:    
1
Step 1: When n = 1, 5 – 1 = 5 – 1 or 4. Since 4
divided by 4 is 1, the statement is true for n = 1.
  Manual - Powered by Cognero
eSolutions   Page 21
k 214 is not divisible by 17, so the expression is not true
Step 2: Assume that 5 – 1 is divisible by 4 for some
k for n = 2.
positive integer k. This means that 5 – 1 = 4r for
ANSWER:  
n =2
Study
  Guide and Review - Chapter 10

n+1
67. 6 – 2 is divisible by 17.
 
SOLUTION:  
Substitute n = 2 in the given expression.
 
6"·1 2=6' 2
-214
 
214 is not divisible by 17, so the expression is not true
for n = 2.
 
ANSWER:  
n =2
 
2
68. n + 2n + 4 is prime.
 
SOLUTION:  
When n = 2, the value of the expression is 12 which
is not a prime number, so the expression is not true
for n = 2.
 
ANSWER:  
n =2
 

69. n + 19 is prime.
 
SOLUTION:  
When n = 1, the value of the expression is 20 which
is not a prime number, so the expression is not true
for n = 1.
 
ANSWER:  
n =1
 

eSolutions Manual - Powered by Cognero Page 22

You might also like